You are on page 1of 245

PEARSON VUE 1000 FILE CORRECTED BY "PEARSON FIGHTERS" GROUP.

I THANK ONE ALL OF THIS GROUP WHO HELPED ME IN CORRECTION. PLEASE FEEL FREE TO
POST ANY WRONG ANSWERS IN THIS FILE. THE GROUP DID EXCELLENT JOB AND TRIED
MAXIMUM TO CHOOSE CORRECT ANSWERS.

1) 25 year teacher have fear attack and worry before enter the class ( I forgot all the scenario)
what is the initial treatment:
a. Selective serotonin reuptake inhibitor
b. Tricyclic depressant
c. Beta blocker
This is a case of social phobia and the initial treatment is answer A (SSRIs). SSRI is the initial
treatment for any phobia and depression
………………………………………………………………………………………………………………
2) The best initial TTT for depression
is: a. SSRIs
b. Tricyclic depressant
c. MAO inhibitors
d. Beta blocker
The correct answer is A (SSRIs).
3) 50 year old Man presented to ER with sudden headache, blurred of vision, and eye pain. The
diagnosis is:
a. Acute glaucoma
b. Acute conjunctivitis
c. Corneal ulcer Answer: A
Acute angle-closure glaucoma present with Severe headache, Eye pain, Nausea and vomiting
Blurred vision , Halos around lights , Eye redness

4) Which heart condition is tolerable during pregnancy:


a. Eisenmenger syndrome
b. Aortic stenosis
c. Severe mitral regurgitation
d. Dilated cardiomyopathy with EF 20%
e. Mitral stenosis and the mitral area is 1 cm (or mm).
-the correct answer is c.

5) Diffuse abdominal pain “in wave like” and vomiting. The diagnosis is: a. Pancreatitis
b. Appendicitis
c. Bowel obstruction
d. Cholelithiasis
The correct answer is C
Signs and symptoms of intestinal obstruction include:
Crampy abdominal pain that comes and goes
Loss of appetite, Constipation, Vomiting
Inability to have a bowel movement or pass gas
Swelling of the abdomen
………………………………………………………………………………………………………………
6) Which type of contraceptive is contraindicative in lactation:
a. OCPs
b. Mini pills (progesterone-only birth control pills)
c. IUD
d. Condom
e. Depo-Provera (progesterone-only birth control injection)
-The correct answer is A. Because combined OCPs suppress lactation. Progesterone –only
contraceptive are not believed to decrease the quantity of milk

7) A long scenario about patient with polydipsia ad polyuria. osmolality in urine and serum was
given, Low Na.
- Q IS NOT COMPLETE . (Diabetes Insipidus) : failure to concentration of urine as a result of
central or nephrogenic ADH. Present with polyurea, polydepsia and hyponatremia. Diagnosis
central or nephrogenic by administration of Desmopressin acetate (DDAVP) , synthetic analog
of ADH, can be used to distinguish central from nephrogenic DI. 1- Central DI : DDAVP
challenge will decrease urine output and increase urine osmolality 2- Nephrogenic DI : DDAVP
change will not significant decrease urine output
………………………………………………………………………………………………………………

8) A 5 year old child came with earache. on examination there is fluid in middle ear and adenoid
hypertrophy. Beside adenoidectomy on management, which also you should do:
a. Myringotomy b. Grommet tube insertion c. Mastidectomy
d. Tonsillectomy
The correct answer is A.
- Myringotomy (is used for bulging acute otitis media)
Grommet tube insertion (is used for recurrent acute otitis media)
…………………………………………………………………………………………………
9) How the randomized control study become strong or of good validity:
Advantages of RCT:
By doing blinding and allocation concealment. The best way to increase the power of the study
is to increase the sample size.
 Good randomization will “wash out” any population bias
 Easier to blind/mask than observational studies
 Results can be analyzed with well known statistical tools
 Populations of participating individuals are clearly identified
……………………………………………………………………………………………………………
10) Woman with postpartum depression, beside the medical TTT, which should be include in
therapy:
a. Family therapy or support
b. Nonpharmacological treatment
Answer is A
strategies are: cognitive behavioral therapy, psycho educational or support group, increase
partner support ,additional help with child care.

11) Classic Scenario of stroke on diabetic and hypertensive patient. What is the
pathophysiology of stroke:
a. Atherosclerosis b. Aneurysm c. ----- -
The correct answer is A.

12) Middle aged patient with an acyanotic congenital heart disease. the X-ray show ventrical
enlargement and pulmonary hypertension:
a. VSD b. ASD c. Truncus arteriosus d. Pulmonary stenosis
-The correct answer is B, because the pt. in middle age and acynotic.
ASD is the most common congenital anamoly which is detected in adults.
………………………………………………………………………………………………………
13) Best food in travelling is:
a. Boiling water b. Water c. Ice d. Partial cocked fish and meat
The correct answer is A.
……………………………………………………………………………………………………………
14) Mechanism of vitamin C in wound healing:
a. Epithelization b. Aerobic fibroblast synthesis c. Collagen synthesis
d. Enhance vascularization
The correct answer is c.
……………………………………………………………………………………………………………
15) Open globe injury. TTT is: a. Continuous antibiotic drops
b. Continuous water and NS drops
c. Continuous steroids drops
d. Sterile cover and the refer
The correct answer is D.

16) Adolescent female counseling on fast food. What you should give her:
a. Ca + folic acid
b. Vit C + folic acid
c. Zinc + folic acid
d. Zinc + Vit C
The correct answer is A.
17) What is the TRUE about backache with osteoporosis:
a. Normal x ray vertebra excludes the diagnosis
b. Steroid is beneficial TTT
c. Vit D deficiency is the cause
-The correct answer is C , because vit. D deficiency is cause osteomalacia. Osteomalacia is
considering secondary causes of osteoporosis..
……………………………………………………………………………………………………………
18) Adolescent female with eating disorder and osteoporosis
a. Weight gain
b. Vit D
c. Bisphosphonates
The correct answer is A.

19) Fourth degree hemorrhoids, Management:


a. Haemoroidectomy b. band ligation c. sclerotherapy d. fiber diet -
The correct answer is A.
- Classification of internal hemorrhoid : First degree: hemorrhoid do not prolapsed
Second degree: hemorrhoid prolapsed upon defecation but spontaneously reduce
Third degree: hemorrhoid prolapsed upon defecation, but must be manually reduce
Fourth degree: hemorrhoids are prolapsed and cannot be manually reduce
First & Second degree haemorrhoid: life style modification (fiber diet) Third: life style
modification with (band ligation, sclerotherapy or cryotherapy) , if Failed go to surgery.
Fourth: surgery (hemorrhiodectomy)

20) Nulligravida at 8 weak gestational age, follow up for genetic screening, she refused the
invasive procedure but she agree for once screening , what is the appropriate action now:
a. do ultrasound b. Do USG+ blood screening c. 2nd screening d. 3rd screening
e. Amniocentesis
The answer is correct B

21) 70 year old male with chronic Hepatitis B virus antigen carrier. The screening of choice is:
a. Alfa feto protien + liver ultrasound
b. Alfa feto protien + another tumor marker
c. Abdominal CT + abdominal ultrasound
Answer is A.
22) 35 year old smoker , on examination shown white patch on the tongue, management:
a. Antibiotics
b. No ttt
c. Close observation
d. biopsy and excision
Answer is D
This is a case of leukoplakia and the management includes: ask the pt to stop smoking, do a
biopsy for the lesion; if there is pre-cancerous changes or cancer in the biopsy ; surgical excision
should be done.
………………………………………………………………………………………………………………
23) Regarding GDM:
a. Screening for GDM at 24 to 28 weeks
b. Diet control is always successful TTT
c. Screening at 8 weeks The correct answer is A.
. …………………………………………………………………………………………………………………………………………………
24) On examination of newborn the skin show papules or (pastules) over erythema base:
a. transient neonatal pustular melanosis
b. erythema toxicum neonatorum
The correct answer is B.

25) The most common cause of precocious puberty:


a. Idiopathic
b. Functional ovary cysts
c. Ovary tumor
d. Brain tumor
e. Adenoma
-The correct answer is A.
………………………………………………………………………………………………………………
26) Mother worry about radiation from microwave if exposed to her child. What you tell her:
a. Not all radiation are dangerous and microwave one of them
b. Microwave is dangerous on children
c. Microwave is dangerous on adult
d. all microwave is dangerous on children
Answer is D. Its question debate. Discussion needed

27 Earlier sign of puberty in male is:


a. Appearance of pubic hair
b. Increase testicular size
c. Increase penis size
d. Increase prostate size
The correct answer is B.

28) Question about nutritional marasmus on definition. (protein energy malnutrition).


Marasmus is a form of severe protein-energy malnutrition. A child with marasmus looks
emaciated. Body weight may be reduced to less than 80% of the average weight that
corresponds to the height. Marasmus occurrence increases prior to age 1, whereas Kwashiorkor
occurrence increases after 18 months.
It has protein wasting with the presence of edema. The prognosis is better for Kwashiorkor.
……………………………………………………………………………………………………………
29) What is the most risk of antihypertensive drugs on elderly patient:
a. Hypotension
b. Hypokalemia
c. CNS side effect
The correct answer is A.
……………………………………………………………………………………………………………
30) Asymptomatic woman with trichomonas:
a. Treat if symptomatic b. Treat if she is pregnant c. Treat her anyway
-The correct answer is C.
Single dose of metronidazole for symptomatic and asymptomatic pt. and treatment partner
(because it is sexual transmitted disease).
………………………………………………………………………………………………………………
31 What is the risk of GDM on her life later:
a. DM type 1 b. DM type 2 c. Impaired fasting glucose
-The correct answer is B.

32) What is the major sign that can tell you that patient have polycythemia vera rather than
secondary polycythemia:
a. Hepatomegaly b. Splenomegaly c. Venous engorgement d. Hypertension
The correct answer is B.
In polycythemia vera there is clonal proliferation of a pluripotent marrow stem cell. Secondary
polycythemia is associated with excessive diuresis, severe gastroenteritis and burn.
……………………………………………………………………………………………………………
33) What is the deficient vitamin in infantile beri beri :
a. B1(thiamine)
b. vit C
c. vit E
d. Niacin The correct answer is A.
* VITAMINES DEFICIENCY : Vit. A : night blindness & dry skin
Vit, B1 (thiamine): Beriberi (polyneuritis, dilated cardiomayopathy, edema)
Vit. B2 (riboflavin): angular stomatitis, cheilosis. Corneal vascuarization. Vit. B3 (nacine) :
pellagra ( Diarrhea, Dermatitis, Dementia )
Vit. B5 (pantothenate): dermatitis, enteritis, alopecia, adrenal insufficiency
Vit. B6 (pyridoxine): convulsion, hyperirritability
Vit B12 (cobalamin): macrocytic megaloblastic anemia, neurologic symptoms
FOLIC ACID : macrocytic megaloblastic anemia without neurologic symptoms.
Vit C : scurvy ( swollen gums, brusing, anemia, poor wound healing) Vit D : rickets in children ,
osteomalasia in adult Vit. E : increase fragility of RBC
Vit. K : neonatal hemorrhage, increase PT & PTT, normal BT
………………………………………………………………………………………………………………
34) 20 year old male found to have hepatitis b surface antibodies :
a. Previous vaccination
b. Previous infection
c. Active infection
-The correct answer is A.
HBsAg : indicate carrier state. HBsAb : indicate provide immunity to HBV
HBcAg: associated with core of HBV
HBcAB: during widow period, HBcAb-IgM indicate recent disease
………………………………………………………………………………………………………………
35) Most common cause of secondary hypertension in female adolescent is:
a. Cushing syndrome
b. Hyperthyroidism
c. Renal disease
d. Essential HTN
e. Polycystic ovary disease
-The correct answer is C. Most common causes of secondary HTN in young female is renal
artery stenosis (caused by fibromuscular dysplasia )

36) clomiphene citrate: a. induce ovulation


The correct answer is A.
37) Child present with stiffing of neck, fever, headache. You suspect meningitis what is your
initial TTT should be:
a. Tobramycin
b. Levoflaxicine
c. Peneciline(ampicillin)
d. Doxycyclin
The correct answer is c.
AGE Causative organism Treatment
< 1 MONTH GBS, E coli--- Ampicillin + cefotaxime or gentamicin
1-3 MONTHS S. pneumoniae, H.influenza meningococci
Vancomycin + cefotaxime or ceftriaxone
3 MONTHS - ADULT----Pneumococci, meningococci
Vancomycin + cefotaxime or ceftriaxone
>60 YEAR\alcoholism -----Pneumococi, meningococci
Gram –ve bacilli
Treatment: Ampicillin + vancomycin + cefotaxime or ceftriaxone

38) A patient have tender, red nodule on lacrimal duct site. Before referred him to
ophthalmologist what you will do:
a. Topical steroid b. Topical antibiotics c. Oral antibiotics d. Nothing The correct answer is C.
(this is a case of dacrocystitis).
………………………………………………………………………………………………………………
39) About DM in KSA: a. about < 10 %
b. Most of the pt of insulin dependent type
c. female more affected with type 2 DM
d. most of NIDDM are obese
-The correct answer is D.
……………………………………………………………………………………………………………
40) Major aim of PHC in Saudi Arabia :
a. To provide comprehensive maternal & child health
b. ------------ -other aims are: community participation, immunization, referral, chronic disease
management, prescribing, health education, management of communicable disease and
environmental health.

41) 17 yr old , she missed her second dose of varicella vaccine, the first one about 1 yr ago what
you'll do:
a. give her double dose vaccine b. give her the second dose only
c. revaccinate from start d. see if she has antibody and act accordingly
-The correct answer is B.
………………………………………………………………………………………………………………
42) there is outbreak of diphtheria and tetanus in community , regarding to pregnant woman:
a. contraindication to give DT vaccine
b. if exposed ,terminate pregnancy immediately
c. if exposed , terminate after 72 hour d. give DT vaccine anyway
-The correct answer is D.
Live vaccines are contraindicated during pregnancy.

43)All of the following are live vaccine except:


a. MMR b. Oral polio c. Varicella d. Hepatitis B vaccine e. BCG
-The correct answer is D
MMR, Oral polio vaccine, BCG and live Typhoid vaccines are contraindicated during pregnancy
but not during lactation.
………………………………………………………………………………………………………………
44) Pt with scoliosis, you need to refer him to the ortho when the degree is:
a. 5 b. 10 c. 15 d. 20
-The correct answer is D. No idea about it. Check the references but could not find anything
…………………………………………………………………………………………………………………………………………………
45) 87 year old who brought by his daughter, she said he is forgettable, doing mess thing in
room , do not maintain attention , neurological examination and the investigation are normal
a. Alzheimer disease
b. Multi-Infarct Dementia c. ---
-The correct answer is B.
Multi-infarct dementia (MID) is a common cause of memory loss in the elderly
Symptoms include confusion or problems with short-term memory; wandering, or getting lost in familiar places;
walking with rapid, shuffling steps; losing bladder or bowel control; laughing or crying inappropriately; having
difficulty following instructions; and having problems counting money and making monetary transactions. MID,
which typically begins between the ages of 60 and 75, affects men more often than women

46) Scenario of trauma , on face examination there is shifted mouth angle, loss of sensation of
ant. Third of tongue, which CN is affected:
a. Facial nerve b. Trigeminal nerve c. ---- -
The correct answer is A

47) Patient with cancer. You want to break bad news, which of the following is the answer:
a. Inform his family
b. Find out how much the patient know about it
c. Let social service inform him d. Don’t tell him
-The correct answer is B.

48) HIV patient has hemorrhagic lesion in the mouth and papules in the face. Skin biopsy show
spindle cells and vascular structures: a. Kaposi sarcoma
Kaposi sarcoma : malignant multifocal neoplasm characterized by vascular tumors of skin and
viscera , caused by Human herpesvirus 8 (HHV8) , most commonly associated with AIDS patient.
Lesion usually papules. in biopsy show spindle shape of cell.
………………………………………………………………………………………………………………
49) Patient with retrosternal chest pain , barium swallow show corkscrew appearance
a. Achalasia
b. Esophagitis
c. GERD
d. Diffuse esophageal spasm
-The correct answer is D.
-Achalasia : show esophagus dilation with (bird beak) tapering of distal esophagus
Diffuse esophagus spasm: show corkscrew appearance.
………………………………………………………………………………………………………………
50) Most common cause of intra cerebral hemorrhage:
a. ruptured aneurysm
b. Hypertension
c. Trauma
-The correct answer is B.

51) 5 yr old child with h/o fever and swelling of the face ant to the both ears (parotid gland
enlargement) what is the most common complication:
a. Labrynthitis
b. meningitis
c. encephalitis
d. orchitis.
The correct answer is B.
Orchitis is the most common complication in post pubertal males adolescents and adults.
………………………………………………………………………………………………………………
52) classic symptoms of tension headache .
Tension headache : is the most common type of headache diagnosed in adult. Present with
tight, band-like pain . may be generalized or most intense in the frontal, occipital, and neck
region. Usually occurs at the end of day. Treatment avoidance of exacerbating factors, NSAID
and acetaminophen.
……………………………………………………………………………………………………………
53 prophylaxis of Asthma
a. oral steroid
b. inhaled steroids
c. inhaler bronchodilator B agonists
-The correct answer is B.
……………………………………………………………………………………………………………
54) 30 year woman with dysmenorrhea, menorrhagia, infertility, and on examination found
immobile mass on uterosacral ligaments :
a. uterine fibroid
b. endometriosis
The correct answer/r is B

55) Classic symptoms of primary dysmenorrhea


Primary dysmenorrhea: menstrual pain associated with ovulatory cycle in absence of
pathological finding.
Secondary dysmenorrhea: Menstrual pain which organic causes exist, most common caused by
endometriosis, adenomyosis, tumor and fibroid.
………………………………………………………………………………………………………………
56) Newly married woman complain of no pregnancy for 3 month with unprotected sexual
intercourse :
a. Try more (infertility is defined as no pregnancy for one year)
………………………………………………………………………………………………………………
57) Younger diabetic patient came with abdominal pain, vomiting and ketones smelled from his
mouth. What is frequent cause:
a. Insulin mismanagement
b. Diet mismanagement
The correct answer is A..
But if the infection is one of the answers , you should choose it because the most common
cause of DKA is infection.
……………………………………………………………………………………………………………
58) Cause of syncope in aortic stenosis
a. Systemic hypotension
59) On stroke management: all are correct except
a. IV fluid not include dextrose
b. Diazepam for convulsion
c. No indication of anticonvulsive drugs
-The correct answer is C.
………………………………………………………………………………………………………………
60) Patient use illegal drug abuse and the blood show RNA virus. Which hepatitis
a. A
b. B
c. C
d. D
-The correct answer is C.
Hepatitis B is DNA virus

61) Classic symptoms and signs of infectious mononucleosis (EBV) ?


Most common occur in young adult, usually caused by EBV , transmission through exchange of
body fluid include saliva. Present with triad (fever, sore throat, lymphadenopathy) , also, may
present tonsillar exudates, splenomegaly, maculopapular rash and bilateral upper eyelid
edema. Management is mostly supportive and corticosteroids are indicated for airway
compromise due to tonsillar enlargement.
………………………………………………………………………………………………………
62) Treatment of EBV ( in scenario there patent with tonsillar exudates, lymphadenopathy,
splenomegaly) :
a. Oral acyclovir
b. Oral antibiotic
c. IM or IV acyclovir
d. Supportive -
The correct answer is D.

63) 25 year old woman with weight loss, heat intolerance, irritable ….
a. Hyperthyroidism
………………………………………………………………………………………………………………
64) Patient with coloured pustules around his mouth, organism show herpes simplex type 1,
what is the treatment:
a. Oral antiviral
b. Iv antiviral
c. Supportive
-The correct answer is A.
66) Acute otitis media , initial TTT: a. Amoxicillin
………………………………………………………………………………………………………………
66) Most common symptoms of renal cell carcinoma is
a. Hematuria b. Abdominal mass c. Flank pain
The correct answer is A.
67) Link the ttt with organism:
a. Shigella –metronidazole
b. Salmonella --erythromycin
c. Comphylobacter--amoxicillin
d. Giardia lamblia-- metronidazole
Correct answer is D
68) Medical student had RTA systolic pressure is 70 mmhg, what you will do next in
management:
a. IV fluid therapy b. ECG c. Abdominal U/S d. X-ray
The correct answer is A.
Always ABC first. BLS protocols
………………………………………………………………………………………………………………
69) 20 year old male had been stabbed on midtriceps muscle area,
On microscopic examination of this greenish fluid show gram positive cocci in chains:
a. Streptococcal gangrene
b. Clostridium gangrene
c. Fournier’s gangrene d. meningococcemia
The correct answer is A.

70) Patient around his nose there are pustules, papules and telangiectasia lesions.
The diagnosis is: a. Rosacea

71) Child with radial head dislocation, what is the next in management:
a. Reduction b. x ray c. MRI
-The correct answer is B.
Always take x-ray first before reduction

72) In cervical LNs there are well differentiated thyroid cells, during operation you find no lesion
on thyroid what will you do next
a. Total thyroidectomy
b. Total thyroidectomy + radical cervical LNs dissection
c. Total thyroidectomy + specific LNs dissection
d. Thyroid lobectomy
-The correct answer is B.
73) Irritable bowel syndrome, rapid intestinal transit produce which symptom
a. Vomiting
b. Diarrhea
c. Constipation
d. Abdominal pain
Answer B

74) Free fluid accumulate in abdominal cavity cause:


a. Hypovolemic shock
b. Cardiogenic shock
c. Sepsis
d. Emesis
-The correct answer is C.

75) Woman with cyclic bilateral nodularity in her breast since 6 month, on examination there is
3 cm tender mobile mass in her breast : what you will do next
a. FNA with cytology
b. Mammogram
c. Biopsy
d. Follow up for next cycle
e. Observation
-The correct answer is D.

76) Most common symptoms of soft tissue sarcoma :


a. Paralysis
b. growing mass
c. pain
The correct answer is B. growing mass: painless and slow-growing.
77) the most common symptom in placenta abruption is
a. Vaginal bleeding
b. Uterine tenderness
c. Uterine contractions
d. Fetal distress
The correct answer is A
Placental abruption is mainly a clinical diagnosis with all the above findings. the most common
symptom is dark red vaginal bleeding with pain during the third trimester of pregnancy (80%)
and abdominal or uterine tenderness (70%).
Bleeding may occur at various times in pregnancy
Bleeding in the first trimester of pregnancy is quite common and may be due to the following:
miscarriage, ectopic pregnancy .
Bleeding in late pregnancy (after about 20 weeks) may be due to the following: placenta previa
or placental abruption.

78) Female presented with vaginal discharge, itching, and on microscope showed mycotic cells
and spores. This medical condition is most likely to be associated with:
a. TB
b. Diabetes
c. Rheumatoid Arthritis
The correct answer is B
Vaginal thrush or Candidal vulvovaginitis is a common infection caused by a yeast called
Candida albicans. Vulvovaginal candidiasis is usually secondary to overgrowth of normal flora
Candida species in the vagina. Conditions that interrupt the balance of normal vaginal flora
include: antibiotic use, oral contraceptives, contraceptive devices, high estrogen levels, and
immunocompromised states such as diabetes mellitus and HIV. Women are prone to vaginal
thrush between puberty and the menopause because, under the influence of the hormone
estrogen, the cells lining the vagina produce a sugar and yeasts which Candida albicans are
attracted to. That is why thrush is rare before puberty.
79) Prim gravida in her 8th week of gestation, presented to your clinic wanting to do genetic
screening, she declined invasive procedure. the best in this situation is
a. Amniocentesis
b. 1st trimester screening
c. 2nd trimester screening
d. Ultrasound
The correct answer is B
First trimester screening, also called the first trimester combined test, has two steps:

 A blood test to measure levels of two pregnancy-specific substances in the mother's


blood — pregnancy-associated plasma protein-A and human chorionic gonadotropin
(HCG)
 An ultrasound exam to measure the size of the clear space in the tissue at the back
of the baby's neck (nuchal translucency)
Typically, first trimester screening is done between weeks 11 and 14 of pregnancy
………………………………………………………………………………………………………………………………………….
80) Trichomoniasis is classically have: a. Clue cells b. Greenish frothy discharge
The correct answer is B
Trichomoniasis is caused by the flagellated protozoan Trichomonas vaginalis; it's the most
common curable sexually transmitted disease. It usually presents with frothy yellow-green
vaginal discharge, strong-unpleasant odor, pain during urination and sexual intercourse.
Strawberry cervix is typical sign of Trichomoniasis
81) Obsessive neurosis: (OCD)
a. Treatment is east
b. Clomipramine doesn’t not work
c. Mostly associated with severe depression
d. Can be cured spontaneously
the correct answer is C
82) Patient came to you complaining of hearing voices, later he started to complain of thought
get into his mind and can be taken out
a. SCZ
b. Mood
c. Mania
d. Agoraphobia
The correct answer A.
Schizophrenia

83) Female had history of severe depression, many episodes, she got her remission for three
months with Paroxitine ( SSRIs) .. now she is pregnant .. your advise
a. Stop SSRI's because it cause fetal malformation
b. Stop SSRIs because it cause premature labor
c. Continue SSRI and monitor her depression
The correct answer is C
Use of antidepressants such as selective serotonin reuptake inhibitors during pregnancy has no
known significant teratogenic effects, according to the Primary Care Companion to the Journal
of Clinical Psychiatry. In general, SSRI have the least side effects then other classes of
antidepressants. Some SE: Sleep disturbance, dizziness, sexual dysfunction, Nervousness, and
diarrhea

84) Hallucinations and Paranoia:


a. SCZ
b. Mood
c. Mania
d. Phobia
The correct answer is A
………………………………………………………………………………………………………………
85) Female presented with thirst and polyuria.. all medical history is negative and she is not
known to have medical issues.. .she gave history of being diagnosed as Bipolar and on Lithium
but her Cr and BUN is normal. What is the cause of her presentation
a. Adverse affect of lithium
b. Nephrogenic DI
c. Central DI
The correct answer is A.
Nephrogenic DI is the diagnosis here, not the cause.
Nephrogenic diabetes insipidus is characterized by a decrease in the ability to concentrate urine
due to a resistance to ADH action in the kidney. Nephrogenic diabetes insipidus can be
observed in chronic renal insufficiency, lithium toxicity, hypercalcemia, hypokalemia.

86) Most common medical problems encountered in Primary care is:


a. Coryza
b. UTI
c. HTN
The correct answer is A

87) Regarding diabetes mellitus in pregnancy :


a. Prevalence of diabetes mellitus in pregnancy is 10%
b. Glucose screening is best in 24-28 week
c. Diabetic and non diabetic have same perinatal mortality
d. Gestational diabetes can be diagnosed by abnormal FBS test
The correct answer is B.
OGTT is the diagnostic for GDM
88) Left red eye, watery discharge, photo phobia, peri-auricular non-tender lymph nodes. Dx
a. Bacterial conjunctivitis b. Viral conjunctivitis
The correct answer is B
Watery discharge differentiates viral from bacterial conjunctivitis.
In Bacterial usually purulent or mucopurulent discharge.
………………………………………………………………………………………………………………
89) One of the Anti-psychotics causes ECG changes, Leukopenia, drooling :
a. Respiredone b. Clozapine c. Amisulpride
The correct answer is B
Common clozapine side effects may include:
weight gain; tremor, dizziness, spinning sensation;
headache, drowsiness; fast heart rate;
nausea, constipation; dry mouth, or increased salivation;
blurred vision; or increased sweating

90) One of the following decrease chance of colon cancer :


a. Zinc b. Vit. E c. Vit C d. Folic acid
Answer D. Both vit D and folic acid
Both C, E, Beta carotene all are Antioxidants, they are correct but C is more likely. A big dose of
vitamin C fights the big "C." some others: Fiber, Vitamin C, Calcium, Vitamin E, Selenium.

91) Best sentence to describe specificity of screening test, is the population of people who :
a. Are negative of disease, and test is negative
b. Are positive of disease, and test is negative
c. Are positive comparing to total other people
d. Negative disease , positive test e. Positive disease , negative test
The correct answer is A
Specificity measures the proportion of negatives which are correctly identified (e.g. the
percentage of healthy people who are correctly identified as not having the condition).
Sensitivity (also recall rate) measures the proportion of actual positives which are correctly
identified (e.g. the percentage of sick people correctly identified as having the condition).
………………………………………………………………………………………………………………
92) Heavy smoker came to you asking about other cancer, not Lung cancer, that smoking
increase its risk:
a. Colon b. Bladder c. Liver
The correct answer is B
smoking-associated cancers are lung, head &neck (like esophagus and larynx), bladder and
kidney, pancreatic, cervical and stomach.

93) Mid clavicle fracture


a. Surgery is always indicated if fracture is displaced
b. Figure-8-dressing has better outcomes than simple sling
c. Figure-8-dressing is strongly indicated in patient with un-union risk
d. Both figure-8 and simple sling has similar outcomes
the correct answer is D - simple sling has been to give the same result as a figure-8 (more
comfort and fewer skin problem).
………………………………………………………………………………………………………………
94) patient complains of "can't breathe air in one nostril "; on examination showed edematous
mucosa structure, best to give initially
a. Corticosteroids b. Decongestants c. Alfa-adrenergic blockers
The correct answer is A
95) Pediatric pt came to you in ER with wheezing, dyspnea, muscle retractions ( most probably
asthma), best to give initially is :
a. theophylline
b. Albuterol nebulizers
c. oral steroids
the correct answer is B
96) Female presented with complain of neck pain and occipital headache, no other symptoms,
on X-ray has cervical spine osteophytes and narrow discs :
a. cervical spondylosis
Although plain films of the cervical spine are the least costly and most widely available imaging
modality, the imaging study of choice is MRI.
………………………………………………………………………………………………………………
97) Patient complaining of pain at night when he elevate his arm, tingling on lateral arm side
and lateral three fingers , Dx
a. brachial plexus neuropathy
b. shoulder impingement syndrome
c. brachial artery thrombophlebitis d. thoracic outlet syndrome
the correct answer is D
Branchial plexus neuropathy is characterized by acute onset of intense pain in the shoulder or
arm followed shortly by focal muscle weakness.
Impingement syndrome, swimmer's shoulder, and thrower's shoulder, is a clinical syndrome
which occurs when the tendons of the rotator cuff muscles become irritated and inflamed. This
can result in pain, weakness and loss of movement at the shoulder. The pain is often worsened
by shoulder overhead movement and may occur at night, especially if the patient is lying on the
affected shoulder.
Thoracic outlet syndrome: numbness and tingling in the fingers; pain in the neck, shoulder, and
arm; weakness of the arm and dropping things from the hand; worsening of the symptoms
when elevating the arm to do such things as comb or blow dry one's hair or drive a car; and
coldness and color changes in the hand. The symptoms are often worse at night or when using
the arm for work or other activities.

98) Young adult presented with pain on lateral elbow, tingling of lateral arm, he plays Squash:
a. carpel tunnel syndrome b. tennis elbow ( Lateral epicondylitis)
the correct answer is B
Lateral epicondylitis ( inflammation of common extensor tendon ) also known as tennis elbow,
shooter's elbow and archer's elbow is a condition where the outer part of the elbow becomes
sore and tender. It is commonly associated with playing tennis and racquet sports.
-Medial epicondylitis (inflammation of common flexor elbow) also know (golfer elbow)
99) male came to you complaining of sudden progressive loss of vision of left eye over last
two/three days, also pain on the same eye, on fundoscopy optic disk swelling was seen , Dx :
a. central retinal artery occlusion
b. central retinal vein occlusion
c. optic neuritis d. macular degeneration
CRAO CRVO and macular degeneration causes painless loss of vision
Correct answer is C
Central retinal Artery occlusion: presenting complaint is an acute sudden painless unilateral
loss of vision. Shows a classic cherry red spot Central retinal vein occlusion: usually sudden
painless variable visual loss; the fundus may show retinal hemorrhages, dilated tortuous retinal
veins, cotton-wool spots, macular edema, and optic disc edema.
Optic neuritis: Major symptoms are sudden loss of vision (partial or complete), or sudden
blurred or "foggy or washed out" vision, and pain on movement of the affected eye. Many
patients with optic neuritis may lose some of their color vision in the affected eye (especially
red). The optic disk becomes swollen
Macular degeneration is a leading cause of permanent painless irreversible vision loss in the
elderly.
Retinal Detachment; Symptoms are decreased peripheral or central vision, often described as a
curtain or dark cloud coming across the field of vision. Associated symptoms can include
painless vision disturbances, including flashing lights and excessive floaters.

100) unilateral headache, exaggerated by exercise and light , Dx :


a. migraine b. cluster headache c. stress headache
the correct answer is A
101) 70 years old with progressive dementia , no personality changes , neurological
examination was normal but there is vision deficit , on brain CT shower cortex atrophy and
ventricular dilatations :
a. multi micro infract dementia
b. Alzheimer dementia
c. parkinsonism dementia
the correct answer is B
Alzheimer dementia : most common cause of dementia.
Age and family history are risk factors for AD. Etiology unknown but toxic b-amyloid deposit in
brain. Present with amnesia for newly acquired information is usually the first presentation,
followed by language deficit, acaluia, depression, agitation and finally apraxia (inability to
perform skilled movement).
Diagnosis by exclusion that can be definitive diagnosis only on autopsy: MRI & CT may show
atrophy , ventricular enlargement and can rule out other causes. On brain microscopy amyloid
plaques and neurofibrially tangle. Death usually occurring secondary to aspiration pneumonia.
treatment by supportive therapy for Pt. and family , and cholinesterase inhibitor .
Multi micro infarct dementia ( vascular dementia ) dementia associated with history of stroke.
Criteria for vascular dementia include presence of dementia and 2 or more of the following: 1-
focal neurological signs 2- symptoms onset that was abrupt , stepwise, or related to stroke 3-
brain imaging showing evidence of fold infarction or extensive deep white matter changes
secondary to chronic ischemia.

………………………………………………………………………………………………………

102) 70 years old with progressive dementia , on brain microscopy amyloid plaques and
neurofibrillary tangles are clearly visible also Plaques are seen : Dx
a. lewy dementia
b. Parkinsonism
c. Alzheimer
The correct answer C
103) After bite, pediatric patient presented with abdominal pain and vomiting , stool occult
blood , rash over buttock and lower limbs , edema of hands and soles , urine function was
normal but microscopic hematuria was seen:
a. Lyme
b. Henoch-Schonlein Purpura
The correct answer is B Henoch-Schonlein purpura (HSP) is a small-vessel vasculitis
characterized by purpura, arthritis, abdominal pain, and hematuria.
………………………………………………………………………………………………………………
104) for the above disorder (Q 103 ), which one is considered pathological
a. gross hematuria b. microscopic hematuria c. rashes
The correct answer A
105) Young adult presented with painless penile ulcer rolled edges .. what next to do :
a. CBC b. Darkfeild microscopy c. Culture
The correct answer is B
Syphilis also known as "great imitator" is a sexually transmitted disease caused by the
spirochete bacteria Treponema pallidum. Classically presents as single painless non-itchy skin
ulceration with sharp borders. T pallidum is too small to be seen under the light microscope. So
use dark field microscopy when sores are present. Blood tests can confirm the presence of
antibodies. The antibodies remain in your body for years, so the test can be used to determine
a current or past infection.

106) Diabetic female her 24h-urine protein is 150mg ,,


a. start on ACEIs b. refer to nephrologist c. do nothing , this is normal range
the correct answer is A

107) which prevent or decrease incidence of getting post herpetic neuralgia


a. Amitriptyline b. Acyclovir c. Varicella vaccination
The correct answer is C
prevention of PHN involves the use of a varicella vaccine to prevent varicella and herpes zoster.
A live, attenuated varicella vaccine is effective in protecting against varicella and its
complications

………………………………………………………………………………………………………

108) Adult Polycystic kidney disease mode of inheritance:


a. Autosomal dominant

………………………………………………………………………………………………………

109) Female came with complain of diarrhea in the last 6 months, she lost some weight, she
reported that mostly was bloody , when you preformed sigmoidoscopy you found fragile
mucosa with bleeding ,Dx
a. colon cancer b. Crohn's c. Ulcerative colitis d. Gastroenteritis e. Hemorrhoids
The correct answer is C
110) Anal fissure commonest site
a. Posteriorly b. Anteriorly
Answer is A
In 90% of cases, the fissure manifests as a painful linear ulcer lying in the posterior midline of
the anal canal. The fissure may occur in other areas as well, such as the anterior midline (more
commonly in female patients). Patients describe the pain of anal fissures as "passing broken
glass" and a burning pain that can remain for several hours after defecation.

………………………………………………………………………………………………………

111) mother gave birth of baby with cleft lip and palate, she want to get pregnant again what is
the percentage of recurrence
a. 1%
b. 4%
c. 15%
Siblings (overall risk) 4% for cleft lip and palate
1.8% if cleft palate alone
The correct answer is B
112) 4 years old girl presented with her parents to ER with sore throat and serosanginous
vaginal discharge with no pain what is most probable cause,
a. Candida
b. Foreign body
c. Chlamydia
d. Gonococci
e. streptococcus
The correct answer is E
Pediatric vaginal discharge: 1- infectious vulvovaginitis: present with malodorous , yellow
green, most common caused by group A streptococcus. (may be present with sexual abuse
"STDs" ) 2-foreign body
3-candidal infection: may associated with diabetes … measure glucose.
………………………………………………………………………………………………………………
113) patient complaining of pain along median nerve distribution, And positive tinel's sign.
treatment include casting of both hand in what position
a. Dorsiflexion
b. plantar flexion
c. extension
d. Abduction
The correct answer is A
………………………………………………………………………………………………………………
114) dermatomyosistis what is true
a. distal muscle weakness
b. Underlying malignancy
c. Generalized Skin rash
The correct answer is c
Polymyositis : progressive , systemic tissue disease characterized by immune-mediate striated
muscle inflammation, present with symmetric progressive proximal muscle weakness and pain .
DERMATOMYOSITIS : present polymyositis plus cutaneous involvement, heliotrope rash
(violaceous periorbital rash) , shawl sign ( rash involving the shoulder, upper chest and back ) ,
Gottron"s papule ( popular rash with scale ).

115) pt with hypertrophic sub aortic stenosis referred from dentist before doing dental
procedure what is true
a. 50 % risk of endocarditis
b. 12 % risk of endocarditis
c. No need for prophylaxis
d. post procedure antibiotic is enough
the correct answer is C
………………………………………………………………………………………………………………
116) pt want to quit smoking you tell him that symptoms of nicotine withdrawal appear after
a. 1-2 days
b. 2-4 days
c. 5-7 days
d. 8- 10 days
The correct answer is A

………………………………………………………………………………………………………

117) pt taking bupropion to quit smoking what is SE


a. Arrhythmia b. Seizure c. xerostomia d. Headache
The correct answer is B

………………………………………………………………………………………………………

118) 14 years old girl complaining of painless vaginal bleeding for 2-4 days every 3 weeks to 2
months ranging from spotting to 2 packs per day; she had 2ry sexual ccc 1 year ago and had her
menstruation since 6 months on clinical examination she is normal sexual ccc, normal pelvic
exam appropriate action
a. OCP can be used
b. You should ask for FSH and prolactin level
c. Reassure. It may be normal menstruation
Answer: C

119) Pt want to do dental procedure, he was dx to have mitral valve prolapse clinically by
cardiologist, he had never done echo before what is appropriate action
a. Do echo
b. No need for prophylaxis
c. give ampicillin
d. Give amoxicillin clavulanic
The correct answer is B
The current infective endocarditis guidelines state that use of preventive antibiotics before
certain dental procedures is reasonable for patients with:
prosthetic cardiac valve or prosthetic material used for cardiac valve repair
a history of infective endocarditis
a cardiac transplant that develops cardiac valvulopathy
the following congenital heart disease
unrepaired cyanotic congenital heart disease, including palliative shunts and conduits
a completely repaired congenital heart defect with prosthetic material or device, whether
placed by surgery or by catheter intervention, during the first six months after the procedure
any repaired congenital heart defect with residual defect at the site or adjacent to the site of a
prosthetic patch or a prosthetic device (that inhibit endothelialization)
Except for the conditions listed above, antibiotic prophylaxis is no longer recommended for any
other form of congenital heart disease.

………………………………………………………………………………………………………

120) Rx of cyclical mastalgia


a. Stop OCP, analgesic, NSAID, Fat reduction, and magnesium
Mastalgia : painful breast tissue that can be cyclic and usually associated with hormonal
change, often bilateral .
Management : Reassurance, Stop current hormonal therapy , stop smoking, fat reduction, avoid
caffeine, exercise, NSAID.
Three main categories are: cyclical, non-cyclical and extra-mammary mastalgia. Cyclical pain,
the commonest, has a temporal association with the menstrual cycle. Pain characteristically
starts in the days before menstruation and gradually increases. It tends to subside once
menstruation has started and often disappears after a few days. Pain in these women usually
abates after the menopause. These factors suggest a hormonal etiology, namely estrogen.

121) 4years old child what can he do


a. Copy square and triangle
b. Speak in sentences
The correct answer is A
………………………………………………………………………………………………………

122) baby can sit without support, walk by holding furniture. Pincer grasp, pull to stand how old
is he
a. 8 months
b. 10 months
c. 12-month
d. 18 month
The correct answer is B

123) 73 year patient complain of progressive loses of memory with decrease in cognition
function . C.T reveal enlarge ventricle and cortical atrophy , diagnosis is
a- Alzheimer b-multi infarct dementia c- multiple sclerosis d-????????
The correct answer is A
Alzheimer (Dx by exclusion. Its associated with progressive memory loss, decreased cognition
function , & enlarged ventricles with cortical atrophy)
multi infarct dementia (NOT progressive & it has focal neurological abnormality) multiple
sclerosis (recurrent relapsed & complete remission. Its associated with demyelination of gray-
matter)

…………………………………………………………………………………………………………………………………………

124) 62 female with –ve pap smear you should advice to repeat pap smear every:
A- 6m
b- 12m
c- 18m
d- no repeat
The correct answer is d
Screening pap smear:
1- starting at age 21 years or no more than 3 years after becoming sexually active.
2- women > 30 years who have three consecutive normal test screening ( 1 / 3yeasr).
3- screening should be discontinued for women > 60-70 years who have had 3 or more normal
pap smear.
…………………………………………………………………………………………………………………………………………

125) 12 year boy with sensorineural hearing loss , C.T scan show mass) so diagnosis is
a- acoustic neuroma
b- meningioma
c- barotraumas
The correct answer is A
- acostic neuroma (benign tumor of cranial nerve VIII. Mostly occur between age of 30-60 years.
, Mostly unilateral except if it's associated with neurofibromatosis in which its bilateral) -
meningioma (mostly benign, with age, more with female. Its occur in the cerebellopontine
angle) - barotraumas (mostly in the divers. The damage occur due to pressure)
..……………………………………………………………………………………………….………………………………………………

126) 50 yr with uncontrolled diabetes, complain of black to brown nasal discharge. So


diagnoses is
a- mucormycosis
b- aspirglosis
c-foreign body
The correct answer is A
mycormycosis (fungal infection caused by Mycorales, affect nasal sinus & lungs, .
characterized by black nasal discharge, Dx by biopsy).
………………………………………………………….……………………………………………………………………………………
127) 55 y complain of dyspnea, PND with past history of mitral valve disease diagnosis is
a- LT side HF
b- RT side HF
c- pneumothorax
d- P.E
The correct answer is A
……………………………………………………………………………………………………………………………………
128) clonidine increases the effect of
a- benzotropin (anticholinergic for Parkinson. Not affected by Clonidine)
b- levo dopa
c- rubstin
the correct answer is B
Clonidine is α2 agonist used to TTT hypertension.
carbidopa-levodopa oral increases effects of clonidine oral by added drug effects. Consider
decreasing dosage of medication for reducing high blood pressure.

clonidine-chlorthalidone oral increases effects of levodopa oral by added drug effects

129) 2 y baby with gray to green patch in lower back, no redness or hotness, diagnosis is
a- child abuse
b-no ttt needed
c- bleeding tendency
The correct answer is B I think it is " Mongolian spot " , visible in 6 month and normally
disappear to 3-5 years. No need treatment.
130) 15y boy with unilateral gynecomastia your advice is
a- may resolve spontaneously
b- there is variation from person to person
c- decrease use of soda oil or fish oil
the correct answer is A
- uni- or bilateral gynecomastia occur normally in newborn & at puberty

…………………………………………………………………………………………………………………………………………

131) 6m baby with mild viral diarrhea , ttt by ORS as


a-100ml/kg for 4 hours then 50 ml/kg /day after
b-50 ml/kg for 4 hours then 50 ml/kg /day after
c-100 ml/kg for 4 hours then 100 ml/kg /day after
d-50 ml/kg for 4 hours then 100 ml/kg /day after
Mild Dehydration Total ORS: 50 ml/kg over 4 hours by syringe,
Give an additional 10 ml/kg per stool or 2 ml/kg per Emesis

Moderate Dehydration (accelerated replacement)


Total ORS: 100 ml/kg over 4 hours
Give an additional 10 ml/kg per stool or 2 ml/kg per Emesis
Correct answer: D

132) 36 yr female with breast mass mobile and change with menstrual cycle , no
skin dimple or fathering. Your advice is
a- repeat exam after 2 cycle
b- make biopsy
c- fine needle aspiration
d- oral contraception
The correct answer is A
133) 50y female with breast cancer and CA125 elevate. So elevation due to
a-breast cancer
b-associate with ovarian cancer
c-due to old age
d-normal variation
The correct answer is A
- CA125 is a tumor marker mostly used for ovarian Ca, but it's also used with endometrial,
fallopian, breast, & GIT Ca

…………………………………………………………………………………………………………………………………………

134) 25y female with bradycardia and palpitation. ECG normal except HR130. past history of full
ttt ovarian teratoma, so your advice is
a- Struma ovary should be consider b-vagal stimulation c- referred to cardiology
The correct answer is A
135) pt with alcohol intake complain of headache, dilated pupil, hyperactivity, agitation. he had
history of alcohol withdrawal last week so ttt is
a-diazepam
b-naxtrol
c-haloperidol
the correct answer is A

136) 50y man with chronic psychosis and not complains for ttt .your advice
a- depot haloperidol or floxtin
b- oral lorasepam
c- oral buspiron
d-???????
The correct answer is A
137) 15y boy appear with patch in rt lower leg. these patches are clear center , red in
peripheral, no fever no other complain so diagnosis
a-contact dermatitis
b-tinea corporis
c- lyme disease
The correct answer is B

…………………………………………………………………………………………………………………………………………

138) old pt with of IHD complain for 2 month of redness in lower leg and pulse diminished in
dorsalis pedis these redness increase in dependent position and limp is cold and no swelling
,diagnosis is:
a-arterial insufficiency
b-thrombophibitis
c-cellulites
the correct answer is A
-thrombophlebitis: present with pain, swelling and redness .
-cellulitis: present with redness, hotness and tender.
139) pt with heart disease complain of LL ischemia your advice
a-referred to cardiology
b-vascular surgery
c- start heparin
Correct answer is C.
Start LMWH heparin first followed by vascular surgery referral
……………………………………………………………………………………………………………………………………
140) female after sexual attack on exam hymen tear in
a-2 o'clock
b-4 O, clock
c-6 O clock
d-8 O clock position
the correct answer is C

…………………………………………………………………………………………………………………………………………

141) 7y boy complain of limping. CT show avascular necrosis in epiphysis of femur your advice
a- surgical ttt
b-splint for 6m
c- physiotherapy
Leg calve perths disease
Age is less than 8 yrs (small children) even e extensive AvN => tx is conservative e bed rest and bracing
Age is more than 8 yrs e extensive AVN => for Surgery

Physiotherapy initial treatment and observe for progression


This case is "Perthe's disease" : avascular necrosis of femoral head. 5- 10 years . usually
self limiting with symptoms lasting< 18 months. Present with painless limp , limited abduction
and internal rotation.
Treatment: 1- observation if there is limited femoral head involvement or full ROM .
2- if extensive or if decrease ROM, consider bracing, hip abduction with cast
142) pt with trachoma in eye . for prevention you should
a- water sanitation
b- """""""""""""""""+eradication of organism
c- mass ttt
the correct answer is A
-trachoma in the eye is a bacterial infection caused by Chlamydia trachomatis which is
transmitted by poor hygiene & contaminated water. TTT by antibiotic as erythromycin &
Doxycycline. Surgery to prevent scar

143) pt with severe headache and decrease in visual acuity, pupil is dilated, so ttt
a-pilocarpine drop and ophthalmology referral
b-ergotamine
c-NSAID
the correct answer is A
- this is closed angle glaucoma which characterized by sudden severe headache, red eye,
visual acuity, & dilated pupil.
Initial treatment is IV/Oral acetazolamide and timolol drops. Followed by Pilocarpine
…………………………………………………………………………………………………………………………………………………
144) main ttt of non inflammatory acne is
a-retinoic acid
b-clindmycin;
c-azalic acid
d-erythromycin
the correct answer is A
Topical clindamycin & erythromycin are treatment of inflammatory acne.
azalic acid : treatment of non-inflammatory & inflammatory acne.
retinoic acid : treatment of severe acne & non-inflammatory
…………………………………………………………………………………………………………………………………………………
145) pregnant with insulin dependent with good control, so to decrease risk of congenital
disease
a- good metabolic control before pregnancy
b- good metabolic control in 1st trimester
c- good metabolic control 2nd trimester
d- good metabolic control in 3rd trimester
the correct answer is A.
To decrease the risk of congenital anomalies glycemic control must start before conception
…………………………………………………………………………………………………………………………………………………

146) female not married with normal investigation except FBS=142. RBS196. so ttt
a-give insulin subcutaneous
b-advice not become married
c-barrier contraceptive is good
d- BMI control
The correct answer is D. This is fasting hyperglycemia. Initial treatment is Life style
modifications

147) Diabetic pt come unconscious RBS : 65. so main drug that cause hypoglycemia:
a-sulphonyl urea b-biguanide c-acarbose α-glucosidase inhibitor d-pheniform
the correct answer A
sulphonyl urea (increases insulin secretion, so, it can cause hypoglycemia)
-biguanide ( insulin sensitization, so, can not cause hypoglycemia)
-acabos α-glucosidase inhibitor ( used for postprandial hyperglycemia, so, can . .
not cause hypoglycemia)
-pheniform ( insulin sensitization, so, can not cause hypoglycemia)
…………………………………………………………………………………………………………………………………………………
148) 6m boy with fever you should give antipyretic to decrease risk of
a-febrile convulsion b- epilepsy c- disseminate bacteria d???????????????
The correct answer is A
…………………………………………………………………………………………………………………………………………………
149) picture in computer appear vesicle , bulla and erythema in chest skin so ttt
a- acyclovir cream b-betamethasone cream c-famcyclovir d- erythromycin
This case is "herpes zoster" treatment of herpes zoster is antiviral, analgesic.
Antiviral are ( systemic ) and include: acyclovir, famcyclovir.
correct answer. Famcyclovir: 500 mg T.I.D for 7 days

150) pt with scales on hair margin and nasal fold and behind ear with papule and irregular
erythema so ttt is
a-nizoral cream b- atovit c- acyclovir d-antibiotic
the correct answer is A ( seborrheic dermatitis )
…………………………………………………………………………………………………………………………………………………

151) 14y girl with arthralgia, photosensitivity, malar flush and proteinuria so diagnosis is
a-RA
b-lupus nephritis
c-UTI
The correct answer is B
…………………………………………………………………………………………………………………………………………………
152) question about diarrhea and Yersinia bacteria
Yersiniosis : it is infectious disease caused by Yersinia bacteria. There are 3 types of Yersinia
bacteria : 1- Y.entercolitis , 2- Y.pseudotuberculosis, 3-Y. pestis. Y.enetercolitis cause bloody
diarrhea, terminal ileitis and mesenteric adenitis. Diagnosis by serological : rise in antibody titer
. Treatment : usually self limiting and tetracycline for severe infection.
…………………………………………………………………………………………………………………………………………………
153) paraplegia pt with ulcer in lower back 2+2 cm and lose of dermis and epidermis, this ulcer
in stage
a- I
b-II
c-III
d-IV
the correct answer is B
stage I : non-blanch able redness that NOT subside after relief of the pressure
The skin may be painful, but it has no breaks or tears.
-stage II : skin breaks open. damage to epidermis & dermis but NOT deeper -
Stage III : subcutaneous tissue involvement. Fat may show in the sore, but not muscle, tendon,
or bone.
-stageIV : deeper than subcutaneous tissue as muscles & bones

154) PREGNANT LADY primi at labor pain , in stage I of labor so pain management is
a-morphine IM
b-epidural anesthesia
c-general anesthesia
d-local anesthesia
The correct answer is B
…………………………………………………………………………………………………………………………………………………
155) Psycatric pt on antipsychotic drug most drug that lead to impotence with antipsychotic is
a- propranolol
b-NSAIDs
c-ACEI d-?????????
the correct answer is B
…………………………………………………………………………………………………………………………………………………
156) man present with painless ulcer in his penis with indurate base and everted edge so
diagnosis is
a- syphilis
b- gonorrhea
c- choncroid
d- HSV
The correct answer is A
- syphilis : painless ulcer on the penis
- gonorrhea: STD by bacterial infection cause pain & discharge but NO ulcer -
choncroid: STD by bacterial infection cause pain & discharge but NO ulcer
- HSV : STD by virus that cause vesicles, painful ulcer with discharge.
…………………………………………………………………………………………………………………………………………………

157) man have long history of urethral stricture present with tender right testis and WBC in
urine so diagnosis is
a-epididymoorchitis b- testicular torsion c- varicocele d-??????
the correct answer is A
epididymoorchitis -: occur with UTI & urinary retention)
- testicular torsion : sudden acute severe testicular pain with elevated transversely lying testes

158) man use sildenafil (Viagra), to prevent hypotension you should not use
a-nitrate b-B blocker c-ACIE d-CCB
the correct answer is A
…………………………………………………………………………………………………………………………………………………
159) female complain of painless odorless and colorless vaginal discharge that appear after
intercourse so ttt
a-give antibiotic
b-douche after intercourse
c- cervical cancer
d-may be due to chronic salpingitis
the correct answer is B

…………………………………………………………………………………………………………………………………………………

160) UTI>14 day, most probably cause pyelonephritis


a-0.05% b-0.5% c-5% d-50%
No idea. No references found. I will go with C
…………………………………………………………………………………………………………………………………………………
161) patient complain of right iliac fossa mass so diagnosis
a-diverticulitis b-appendicitis c-pancreatitis d-crohn's disease
- the correct answer is D

162) pt with long history of UC on endoscopes see polyp and cancer lesion on left colon so ttt
a-ttt of anemia b-left hemicolectomy c-total colectomy d- remove polyp
Correct answer is C
…………………………………………………………………………………………………………………………………………………
163) female with hair on different site of body and refuse intake of food and BMI<18 and feel as
body is fat so diagnosis
a-anorexia nervosa b-bulimia nervosa c-body dysmorphic syndrome d- anxiety
the correct answer is A
-anorexia nervosa: decrease body wt. amenorrhea and lanugo (hair ).
-bulimia nervosa: normal or increase body wt. restrict eating following by overeating then guilt.
…………………………………………………………………………………………………………………………………………………
164) obese female, insulin resistance and hirsutism so diagnosis
a-poly cystic ovary b-hyperprolctinmia c-familial d-?"????????
the correct answer is A
…………………………………………………………………………………………………………………………………………………
165) boy 3 day after flu symptoms develop conjunctivitis with occipital and neck L.N enlarged
so diagnosis is
a-adenoviruses b-streptococcus c-HSV
The correct answer is A
…………………………………………………………………………………………………………………………………………………
166) child with asthma use betamethasone, most common side effect is
a-increase intraocular pressure b-epilepsy c-growth retardation d-?????????????
the correct answer is C
…………………………………………………………………………………………………………………………………………………
167) sickling pt after acute attack , discharge on
a- penicillin b-iron c-vitamin d???????????
the correct answer is A
prophylactic of pneumococcal infection by vaccination and oral penicillin
…………………………………………………………………………………………………………………………………………………
168) 6m with cough and wheezy chest. diagnosis is
a- asthma b- bronchiolitis c-pneumonia d-F.B aspiration
The correct answer is B
- asthma : after 2 years old)
- bronchiolitis : before 2 years old
-pneumonia : associated with high grade fever and crepitation in lungs
-F.B aspiration : sudden wheezing usually unilateral
…………………………………………………………………………………………………………………………………………………

169) 15y old with pilonidal sinus so ttt


a- incision surgery
b- local antibiotic
c- daily cleaning
d- ?????????
The correct answer is c
pilonidal sinus : sinus tract which commonly contain hairs.
Treatment , firstly conservative ttt in mild case remove all hair, washing cleaning .
Surgery for pilonidal abscess and chronic pilonidal disease
170) Female pt 8 wks postpartum, not smoker diagnosed to have asthma, her asthma was not
controlled she attended ER 3 times last month, on B2 agonist and oral steroid, she came c/o
wheezing and s.o.b mildly cyanosed using her intercostal muscles, wheezy chest,
BP:160/100 PR:120 PO2:72 PEF:36, there is edema in her foot up to the knee, the most likely
diagnosis:
a. COPD
b. pulmonary embolism
c. Acute asthma attack
d. Angioedema
The correct answer is B ( Not sure. Could be acute asthma also)
8 week post partum with swelling foot extending up to knee. Could be DVT with PE
……………………………………………………………………………………………………………………………………………………
171) Female pt developed sudden loss of vision (both eyes) while she was walking down the
street, also c/o numbness and tingling in her feet ,there is discrepancy b/w the complaint and
the finding, O/E reflexes and ankle jerks preserved, there is decrease in the sensation and
weakness in the lower muscles not going with the anatomy, what is your action:
a. Call ophthalmologist
b. Call neurologist
c. call psychiatrist
d. reassure her and ask her about the stressors
the correct answer is : D

Conversion disorder is a mental condition in which a person has blindness, paralysis, or other
nervous system (neurologic) symptoms that cannot be explained by medical evaluation.

Somatization disorder: also Briquet's syndrome was a mental disorder characterized by


recurring, multiple, and current, clinically significant complaints about somatic symptoms,
although it is no longer considered a clinical diagnosis .female before age 30 years . symptoms
include: two GIT four site of pain , one sexual dysfunction. .

- hypochondriasis: fear from life threatening disease. .


- body dysmorphic disorder: a psychological disorder in which a person becomes obsessed
with imaginary defects in their appearance..

- somatoform pain disorder: intensity pain is main symptoms

172) male pt developed corneal ulcer in his Rt eye after trauma what is the Mx:
a. Antibiotic and cycloplegic and refer to ophthalmology
b. topical steroid
the correct answer is A
..........................................................................................................................................................
173) female pt with Rt eye pain and redness with watery discharge, no h.o trauma, itching ,
O/E there is diffuse congestion in the conjunctiva and watery discharge what you'll do:
a. give Ab b. give antihistamine drops c. topical steroid d. refer her to the ophthalmologist
Answer is B
…………………………………………………………………………………………………………………………………………………….
174) Epidemic disease in poor sanitation areas affecting children and young adults:
a. hep A
b. Hep B
c. Hep C
d. Hep D
The correct answer is A

175) months baby with crying episodes + current jelly stool, looks slightly pale, signs of
obstruction wht is your Mx:
a. barium enema
b. immediate surgery
c. I.v fluid & wait for resolution
The correct answer is A
Barium enema is both for diagnosis and treatment
Intussusceptions: Condition in which one portion of bowel invaginates into an adjacent
segment "usually proximal to ileocecal valve" . most common in first two years of life " usually
between 3 months and 3 years of life" . Abrupt –onset , colicky abdominal pain , often
accompanied by flexed knee and vomiting , ( one-off pain ) child may appear well between
episodes. Classic triad : abdominal pain , vomiting , blood per rectum" only one third of pt." .
Late signs : bloody mucus in stool "currant jelly stool" , abdominal tenderness , palpable
"sausage- shape " . RUQ abdominal mas.
Investigation & treatment : correct any volume and electrolyte abnormality and check cbc)
Abdominal film may be normal in early stage , and see obstruction , perforation in late stage,
US see " target sign"
In setting of high clinical suspicion >>> air-contrast barium enema, should be performed
without delay. As diagnostic in 95% of cases and curative in 80% of cases perform surgical
reduction of gangrenous bowel.
……………………………………………………………………………………………………………………….……………………….....
176) 17 yr adolescent, athletic ,with h/o Rt foot pain planter surface, diagnosis is:
a. planter fasciitis
b. valux……
c. valux…..
Planter fasciitis( heel spur syndrome ) :
Repetitive strain injury causing micro tears and inflammation of planter fascia. Common in
athletes , also associated with D.M , obesity , seronegative and seropositive arthritis.
Clinical feature: morning pain and stiffness, intense pain when walking from rest and that
subsides as pt. continuous to walk. Swelling and tenderness over sole, greatest at medical
calcaneal tubercle and 1-2 cm distal to along planter fascia. pain with toe dorsiflexion

177) pregnant lady 16 wks presented with vaginal bleeding ,enlarged abdomen, vomiting ,her
uterus is smaller than expected for the gestational age, BhCG 80
U/S snowstorm appearance, diagnosis:
a. complete hydatiform mole b. partial hydatiform mole
The correct answer is A
Presence of fetal parts on ultra sound is diagnostic of partial hydatiform mole whereas
Snow storm appearance is seen in complete molar pregnancy
…………………………………………………………………………………………………………………………………………………....
178) 12 yr boy c/o abdominal pain after playing football, he denied any h/o trauma, the pain is
in the Lt Para umbilical region what inx you want to do:
a. CXR b. ultrasound C. no investigation and discharge
…………………………………………………………………………………………………………………………………………………....
179) A female patient on the 3rd week postpartum. She says to the physician that the
frequently visualizes snakes crawling to her baby’s bed. She knows that it is impossible but she
cannot remove the idea from her head. She says she wakes up around 50 times at night to
check her baby. This problem prevents her from getting good sleep and it started to affect her
marriage. What is this problem she is experiencing?
a. An obsession b. A hallucination c. A postpartum psychosis d. A Delusion
The correct answer is A
-obsession: persistent, unwanted, and intrusive ideas, thoughts, impulses or images -
hallucination: perception of objective event without existing external stimulus
-illusion: false perception of actual external stimulus. -
delusion: an idiosyncratic belief or impression maintained despite being contradicted by reality
or rational argument, typically as a symptom of mental disorder.
………………………………………………………………………………………………………………………….…………………………
180) female pt c/o sever migraine that affecting her work, she mentioned that she was
improved in her last pregnancy, to prevent that:
a. biofeedback b. propranolol
The correct answer is b

- migraine prophylactic :
1- b-blocker (propranolol) : first line treatment (note: contraindication in pregnancy)
2- TCA (amitriptyline)
3- anticonvulsant- topiramate and valproic acid
4- calcium channel blocker
…………………………………………………………………………………………………………………………………………………….
181) Flu vaccine not given to the baby who is allergic to: a. egg
contraindication of flu vaccine are egg allergic and hypersensitivity to thimerosal

182) Pt with asymptomatic Trichomoniasis:


a. treat her anyway regardless b. treat her if she is symptomatic only
The correct answer is A
treatment of Trichomoniasis :
Single dose of metronidazole for symptomatic and asymptomatic pt. and treatment partner.
…………………………………………………………………………………………………………………………..……………………
183) 7 y.o ,she missed her second dose of varicella vaccine, the first one about 1 y ago what
you'll do:
a. give her double dose vaccine
b. give her the second dose only
c. see if she has antibody and act accordingly
The correct answer is B
…………………………………………………………………………………………………………………………………………………….
184) pt with gonorrhea infection what else you want to check for a. Chlamydia trachomatis
…………………………………………………………………………………………………………………………………………………….
185) female pt with Aortic stenosis, she developed syncope while she was in the class and she
recovered immediately, what is the cause of syncope:
a. valvular rupture
b. systemic hypotension
The correct answer is B
186) male pt with acute urine retention what is your action:
a. insert folly's cath and ask him to come back to the clinic
…………………………………………………………………………………………………………………………………………………….
187) In battered women which is true:
a. mostly they come from poor socioeconomic area b. usually they marry a second violent
man c. mostly they come to the E/R c/o……….. d. mostly they think that the husband
respond like this because they still have strong feeling for them
Answer ( D )
Battered women is Women who are physically and mentally abused over an extended period,
usually by a husband or other dominant male figure. Characteristics of the battered woman
syndrome are helplessness, constant fear, and a perceived inability to escape. So I think choice
d is correct answer
…………………………………………………………………………………………………………………………………………………….
188) smoking withdrawal symptoms peak at:
a. 1-2 days b. 2-4 days c. 5.7 days d. 10-14 days
The correct answer is B
……………………………………………………………………………………………………………………….……………………………
189) Mother who is breast feeding and she want to take MMR vaccine what is your advice:
a. can be given safely during lactation
b. contain live bacteria that will be transmitted to the baby
c. stop breast feeding for 72 hrs after taking the vaccine
The correct answer is A
MMR : contraindication during pregnancy , and women should be avoid pregnant in 4 weeks
followed MMR vaccine . MMR is safe during lactation.

190) male pt c/o pain in his Rt elbow, he said that he is using the hummer a lot in his work
diagnosis: a. lateral epicondylitis
-Lateral epicondylitis (inflammation of common extensor tendon ) also known as (tennis elbow,
shooter's elbow and archer's elbow is a condition where the outer part of the elbow becomes
sore and tender. It is commonly associated with playing tennis and racquet sports. -Medial
epicondylitis (inflammation of common flexor elbow) also know (golfer elbow)

treatment of lateral epichondylitis:


1- first line : NSAID +rest + ice
2- second line : corticosteroid injection
3- third line : surgery .percutaneous release of common tendon
……………………………………………………………………………………………………………………………………………………
191) 50 y.o male with difficulty swallowing food with wt loss:
1. Oesophageal cancer
causes of dysphagia food than liquid are : 1- carcinoma … 2- stricture … 3- plummer vision
syndrome = web ( iron deficieny anemia + golssitis )
……………………………………………………………………………………………………………………………..…………………..
192) what drug that improve the survival in CHF
1. digoxin 2. Hydralazine
Improve mortality in CHF : B-blocker + ACE + ARB ( most important is ACE )

193) old man with bilateral knee pain and tenderness that increase with walking and relieved
by rest;
a. RA
b. OA
The correct answer is B
OA: pain with activity and weight bearing . and improve with rest .
RA: morning stiffness > 1 hour. painful and warm swelling of multiple symmetric joint.
……………………………………………………………………………………………………………………………………………………
194) Regarding peritonitis:
a. Complicated appendectomy the cause is anaerobe organism
b. rigidity and the cause is paralytic ileus
c. can be caused by chemical erosions
The correct answer is C
…………………………………………………………………………………………………………………………………………………….
195) regarding MI all of the following true except:
a. unstable angina, longer duration of pain and can occur even at rest.
b. stable angina, shorter duration and occur with excertion
c. there should be q wave in MI
d. even if there is very painful unstable angina the cardiac enzymes will be normal
Correct answer is C.
MI is not always have Q waves.
……………………………………………………………………………………………………………………………………………………
196) Pt with scoliosis, you need to refer him to the ortho when the degree is:
a. 5 b. 10 c. 15 d. 20
The correct answer is D
197) 6 months baby with undescended testis which is true:
a. tell the mother that he need surgery (immediate surgery)
b. in most of the cases spontaneous descent after 1 year
c. surgery indicated when he is 4 years
d. unlikely to become malignant
The correct answer is A
Surgery(orchiopexy) from 6 –18months
……………………………………………………………………………………………………………………………………………………
198) 24 y.o Pt with asymptomatic congenital inguinal hernia:
a. immediate surgery b. surgery indicated when he is >35 y
c. elective surgery if it is reducible
The correct answer is c
…………………………………………………………………………………………………………………………………………………..
199) the most effective thing regarding counseling:
a. family rapport b. well adjusted appointment before counseling
Answer A
…………………………………………………………………………………………………………………………………………………..
200) Pt. has DM and renal impairment when he had diabetic nephropathy??: there is curve
for albumin
a. 5y b. 10y c. 20y d. 25y
Answer B
Microalbuminuria generally precedes overt proteinuria by 5-10 years. Once proteinuria is
detected, renal function gradually deteriorates over 10-15 years
……………………………………………………………………………………………………………………………………
201) Pt has Alzheimer disease and hallucination and delusion ttt:
. Halopridole
Psychotic symptoms (e.g. hallucinations and delusions), agitation and aggressive behavior are
common in patients with Alzheimer’s Disease. A study suggests that haloperidol at a dose of 2-
3 mgs/day is effective and well tolerated by most patients.

202) Generalized anxiety disorder best ttt:


a. SSRIs b. tricyclic A D
The correct answer is A
……………………………………………………………………………………………………………………………………
203) Major depression management:
a. Initial therapy even severe
Management of major depression disorder:
1-pharmacotherapy: effective in 50 – 70% .allow for 2-6 weeks to take effect , treat more than
6 months ( SSRI, TCAs, MAOIs).
2-psycotherapy: psychotherapy combined with antidepressant is more effective than either
treatment alone.
3- Electroconvulsion ( ECCT ).
4- phototherapy: effective for pt. who has a seasonal pattern.
……………………………………………………………………………………………………………………………………
204) Psychiatric pt with un compliance of drugs ttt: a. depo halopredol
……………………………………………………………………………………………………………………………………
205) Strongest risk factor for stroke:
a. Hypertension
b. Atrial fibrillation
The correct answer is A

206) Pt. with salpingitis and there is swelling in pelvis in posterior fornix and it is fluctuant m:
a. Colpotomy
b. Laproscopic
The correct answer is A
In the procedure, the rectouterine pouch is often reached through the posterior fornix of the
vagina. The process of creating the hole is called "colpotomy" if a scalpel incision is made to
drain the fluid rather than using a needle.

……………………………………………………………………………………………………………………………………
207) Child swallowing battery in the esophagus, management?
a. bronchoscopy
b. Watch and serial x rays
Answer is A (it should be endoscope)
208) In the appendicitis the histology is:
a. leukocyte in muscle
b. layer of lymphoid
c. tumor
d. plasma cell
the correct answer is A
in appendicitis : neutrophil exudation throughout mucus, submucus, and muscularis
……………………………………………………………………………………………………………………………………
209) water in the body:
a. 40%
b. differs depend on age and sex
the correct answer is B
Guyton's Textbook of Medical Physiology states that "the total amount of water in a man of
average weight (70 kilograms) is approximately 40 liters, averaging 57% of his total body
weight. In a newborn infant, this may be as high as 75% of the body weight, but it progressively
decreases from birth to old age, most of the decrease occurring during the first 10 years of life.
Also, obesity decreases the percentage of water in the body, sometimes to as low as 45%".
……………………………………………………………………………………………………………………………………
210) regarding drainage of the abscess one of the following is true:
a. Carbuncle and furunculosis need drainage
b. Usually give ceftriaxone and penicillin post drainage
the correct answer is A
Frunculosis and carbuncles are pus-filled infected lumps on the skin. They usually occur as
a one-off in a healthy person. Treatment commonly involves draining the pus and taking a
course of antibiotics. If you have recurring boils you may be advised to have tests to check for
an underlying cause.
……………………………………………………………………………………………………………………………………
211) Salpingitis and PID on penicillin but not improve the most likely organism is :
a. Chlamydia trachomatis b. Neisseria gonorrheae
The correct answer is B
Empiric antibiotic treatment for PID: All regimens must be effective against C trachomatis and
N gonorrhoeae, as well as against gram-negative facultative organisms, anaerobes, and
streptococci.
Regimen A consists of the following:
Ceftriaxone 250 mg intramuscularly (IM) once as a single dose plus
Doxycycline 100 mg orally twice daily for 14 days
Metronidazole 500 mg orally twice daily for 14 days can be added if there is evidence or
suspicion of vaginitis or if the patient underwent gynecologic instrumentation in the preceding
2-3 weeks
Regimen B consists of the following:
Cefoxitin 2 g IM once as a single dose concurrently with probenecid 1 g orally in a single dose,
or another single-dose parenteral third-generation cephalosporin (eg, ceftizoxime or
cefotaxime) plus
Doxycycline 100 mg orally twice daily for 14 days
Metronidazole 500 mg orally twice daily for 14 days can be added if there is evidence or
suspicion of vaginitis or if the patient underwent gynecologic instrumentation in the preceding
2-3 weeks
……………………………………………………………………………………………………………………………………
212) Wound at end inflammatory phase when:
a. Epithelial tissue formation
b. Angiogenisis
c. when the wound clean
d. Scar formation
The correct answer is B
213) Juvenile RA ttt:
a. Aspirin b. Steroid c. Penicillamine d. hydroxycloroquin
The correct answer is A
The treatment of JRA focuses on suppressing inflammation, preserving and maximizing
function, preventing deformity, and preventing blindness. NSAIDs are the first choice in the
treatment of JRA.
214) Pt. come with history of tenia capitis ttt:
a. tar shampoo b. oral Fluconazole
The correct answer is B
Newer antifungal medications, such as ketoconazole, Iitraconazole, terbinafine, and
fluconazole, have been reported as effective alternative therapeutic agents for tinea capitis. Of
these agents, itraconazole and terbinafine are used most commonly.
……………………………………………………………………………………………………………………………………
215) Pt. with history of COPD the most action to prevent complication is:
a. Pneumococcal vaccine
b. Cessation of Smoking
c. Oral steroid
d. B2 agonist
The correct answer is B
……………………………………………………………………………………………………………………………………
216) Pt. with congenital hip dislocation :
a. abducting at flexed hip can causes click or tali

217) Colon cancer with stage 3 give the chemotherapy:


a. As soon as possible
b. 1 month
The correct answer is A
Treatment for Stage 3 Colon Cancer
Treatment for stage 3 colon cancer generally consists of a surgical resection followed by
chemotherapy. In a surgical resection, a surgeon removes the part of the colon affected by the
tumor and joins the remaining healthy sections together to form one long, healthy piece.
The standard chemotherapy regimen used to be six months of treatment with 5-FU and
leucovorin, but that "cocktail" was developed decades ago and is seldom used anymore. there
are many new chemotherapy regimens available for stage 3 colon cancer.
…………………………………………………………………………………………………………………………………
218) 56 y old present with vasomotor rhinitis
a. Local anti-histamine b. Local decongestant c. Local steroid d. Systemic antibiotic
Question is incomplete. Symptoms depends on major presenting symptoms.
……………………………………………………………………………………………………………………………………
219) Sickle Cell Anemia give prophlaxis:
a. Penicillin b. Iron
The correct answer is A
TREATMENT ■ Acute crisis: Analgesia and hydration. ■ Hydroxyurea to increase the amount of
fetal hemoglobin. ■ H. influenzae and pneumococcal vaccines;
prophylactic penicillin for Children 5 years of age. ■ Acute chest syndrome: Respiratory support
and exchange transfusion.

220) diagnosis of thalassemia minor:


a. Hb A2 and Hb f b. Microcytosis
the correct answer is A
……………………………………………………………………………………………………………………………………
221) Pt. with MCV decrease and reticulocyte decrease iron deficiency anemia investigation:
a. Ferritin level and TIBC and serum iron
Low serum iron and ferritin levels with an elevated TIBC are diagnostic of iron deficiency. While
a low serum ferritin is virtually diagnostic of iron deficiency, a normal serum ferritin can be seen
in patients who are deficient in iron and have coexistent diseases (eg, hepatitis or anemia of
chronic disorders). These test findings are useful in distinguishing iron deficiency anemia from
other microcytic anemias
……………………………………………………………………………………………………………………………………
222) Born BCG
a. 1month hepatitis b oral polio dpt b. 2month s c. 3month s d. 9 to 12 month mmr
????? I can't understand th Q ( may the writer Q missed some information )

visit Vaccine schedule in Saudi arabia


At birth BCG, Hep B
2 months IPV, HiB, Hep B, Rota virus, Pneumococcal conjugate
vaccine, DTaP
4 months IPV, HiB, Hep B, Rota virus, Pneumococcal conjugate
vaccine, DTaP
6 months OPV, IPV, HiB, Hep B, Pneumococcal conjugate vaccine,
DTaP
9 months Measles, Meningococcal conjugate quadrivalent
12 months OPV, MMR, Meningococcal conjugate quadrivalent
Pneumococcal conjugate vaccine,
18 months OPV, DTaP, HiB MMR, Varicella, Hep A
24 months Hep A
Primary school OPV, MMR, Varicella, DTaP (Td)
age
223) E.histolytica cyst is destroyed by:
a. Freezing b. Boiling
c. Iodine treatment
d. Chlorine
The correct answer is b
Amebiasis (or amoebiasis) is the name of the infection caused by E. histolytica. To help prevent
infection: • Avoid raw vegetables when in endemic areas as they may have been fertilized
using human feces. • Boil water or treat with iodine tablets
……………………………………………………………………………………………………………………………………………………
224) Patient after accident, the left ribcage move inward during inspiration and outward during
expiration:
a. Flial chest
Flail chest is a clinical anatomic diagnosis noted in blunt trauma patients with paradoxical
or reverse motion of a chest wall segment while spontaneously breathing. This clinical finding
disappears after intubation with positive pressure ventilation, which occasionally results in a
delayed diagnosis of the condition.
……………………………………………………………………………………………………………………………………………………
225) Greatest risk of stroke:
a. DM
b. Elevated blood pressure
c. Family history of stroke
d. Hyperlipidemia
e. Smoking
the correct answer is B
226) Child has pallor , eats little meat , by investigation :microcytic hypochromic anemia . what
will you do:
a. Trial of iron therary
b. Multivitamin with iron daily
the correct answer is A
……………………………………………………………………………………………………………………………………………………
227) Treatment of mania that does not cause hepatotoxicity
a. Lithium
b. carbamazepine
c. valporic acid
d. lamotrigine
The correct answer is A
Treatment of mood disorder : Lithium : hepatotoxicity
carbamazepine : agranulocytosis
valproic acid : neural tube defect
……………………………………………………………………………………………………………………………………………………
228) Sickle cell anemia patient, the macula is cherry red , and absence of afferent papillary light
reflex
a. Retinal artery occlusion. .
b. central retinal vein occlusion
the correct answer is A , cherry red spot is sign of retinal artery occlusion.

229) Inflammatory bowel disease is idiopathic but one of following is possible underlying cause
a. Immunological
Inflammatory Bowel Disease Causes
……………………………………………………………………………………………………………………………………………………
230) Patient present with high blood pressure (systolic 200) , tachycardia , mydriasis , sweating
. what is the toxicity:
a. Anticholinergic
b. Sympathomimetic
c. Tricyclic antidepressant
d. Organophosphorus compounds
the correct answer B
231) Treatment of Chlamydia with pregnancy:
a. Azithromycin
b. Erythromycin base
the correct answer B
For treatment of chlamydia during pregnancy, the CDC recommends:
-erythromycin base, 500 mg orally, four times a day for seven days; or
amoxicillin (Amoxil), 500 mg, three times a day for seven days.
Alternatives include: -
Erythromycin base 250 mg, four times a day for 14 days;
-erythromycin ethylsuccinate 800 mg orally, four times a day for seven days; or
-erythromycin ethylsuccinate 400 mg orally, four times a day for 14 days.
Azithromycin 1 gr orally one dose
……………………………………………………………………………………………………………………………………………………
232) the maximum body length will be reached after menarche by
a -6 months
b. 1 year
c. 2 years
correct answer is C
A child will have also reached her final adult height about two years after menarche.
……………………………………………………………………………………………………………………………………………………
233) Patient developed sudden loss of vision bilaterally while she was walking in the street ,
followed by numbness , the subjective symptoms are different from objective , and does not
match anatomical , what is your diagnosis:
a. Conversion syndrome
Conversion disorder: Symptoms or deficits of voluntary motor or sensory function (e.g.,
blindness, seizure) suggest a condition incompatible with medical processes. Close temporal
relationship to stress or intense emotion. More common in young females and in lower
socioeconomic and less educated groups.
……………………………………………………………………………………………………………………………………………………
234) After inflammatory phase of wound , there will be wound healing by:
a. If the wound is clean
b. Angiogenesis
c. Epithelial tissue
the correct answer is B

235) Old male with tender knee, pain and crepitus . the diagnosis:
a. Osteoarthritis
b. Ankylosing spondylitis
c. Rheumatoid
the correct answer A
Osteorhritis OARTHRITIS (OA) ■ A chronic, noninflammatory arthritis of movable joints (e.g.,
DIP joints). Characterized by deterioration of the articular cartilage and osteophyte formation
at joint surfaces. ■ Risk factors include a _ family history, obesity, and a history of joint trauma.
■ Hx/PE: Crepitus; decrease range of motion (ROM); pain that worsens with activity and weight
bearing but improves with rest.
……………………………………………………………………………………………………………………………………………………
236) Mother has baby with cleft palate and asks you what is the chance of having a second
baby with cleft palate or cleft lip ,
a. 25%
b. 50%
c. %1
d. 4%
the correct answer is D
……………………………………………………………………………………………………………………………………………………-
237) 1 liter fluid deficit equals :
. a.1 kg
. Liter of fluid deficit equal 1 kg of fluid as hydration protocol
…………………………….……………………………………………………………………………………………………...................
.
238) After accident patient with tachycardia, hypotension, what will be your initial step:
a. Rapid IVF crystalloid
b. CT
management ABC then rapid IVF crystalloid.
239) 6 years child was born to HBS positive mother is HBS positive, he was only vaccinated by
BCG after birth , what you will give him now :
a. HBV + oral polio + DTP + hib
b. HBV + oral polio + dt + MMR +hib
c. HBV + oral polio + Dt + MMR
Except Hep B vaccine all cn be given
Perinatal HBV transmission can be prevented by identifying HBV-infected (i.e., hepatitis B
surface antigen [HBsAg]-positive) pregnant women and providing hepatitis B immune globulin
and hepatitis B vaccine to their infants within 12 hours of birth.
……………………………………………………………………………………………………………………………………………………
240) Treatment of comedones:
a. Topical retinoids.
Suitable topical agents include:
Benzoyl peroxide
Azelaic acid
Salicylic acid +/- sulfur and resorcinol
Glycolic acid
Retinoids such as tretinoin, isotretinoin, adapalene
Comedones: The plural of comedo, the primary sign of acne, consisting of a dilated (widened)
hair follicle filled with keratin squamae (skin debris), bacteria, and sebum (oil).
Comedones may be closed or open.
……………………………………………………………………………………………………………………………………………………
241) Treatment of papules or pustules:
a.Topical benzoyl
b.Peroxide plus topical antibiotics, mainly clindamycin or erythromycin.
c. In severe cases, intralesional steroid injection or oral antibiotics, such as tetracycline or
erythromycin may be added.
The correct answer B
TREATMENT ■ Mild acne: Topical clindamycin or erythromycin; benzoyl peroxide; topical
retinoids.
■ Moderate acne: The above regimen plus oral antibiotics such as tetracycline.
■ Severe nodulocystic acne: Oral isotretinoin (Accutane).

242) Which is not true In emergency management of stroke


a. Give IVF to avoid D5 50%
b. Give diazepam in convulsions
C. anticonvulsants not needed in if seizures
d.-Must correct electrolytes
e. Treat elevated blood pressure
the correct answer is C
……………………………………………………………………………………………………………………………………………………
243) SCA complications in adults
a. Cerebral infarction
b. Cerebral hemorrhage
……………………………………………………………………………………………………………………………………………………
244) The most common risk for intracerebral stroke
A. Hypertension
245) The antidepressant used for secondary depression that cause sexual dysfunction
a.Sertatline (SSRIs) b. Imapiramine c. Levofluxine
the correct answer A
……………………………………………………………………………………………………………………………………………………
246) Previously healthy female patient presented to ER with dyspnea , anxiety , tremor , and
she breath heavily , the symptoms began 20 minutes before she came to ER , in the hospital she
developed numbness periorbital and in her fingers , what you will do
a. Ask her to breath into a bag
b. Take blood sample to look for alcohol toxicity
Answer may be A
Hyperventilation syndrome or anxiety syndrome
……………………………………………………………………………………………………………………………………………………
247) What is the most important in counseling
a. Exclude physical illness
b. Establishing rapport
c. Family
d. Schedule appointment
may be B correct
248) In breaking bad news
a. Find out how much the patient know
b. Find out how much the patient wants to know
the correct answer A
Robert Buckman's Six Step Protocol for Breaking Bad News
1. Getting started. 2. Finding out how much the patient knows. 3. Finding out how much the
patient wants to know. 4. Sharing the information. 5. Responding to the patient's feelings. 6.
Planning and follow-through.
…………………………………………………………………………………………………………………………………………………
249) Patient with chest pain that aggravated by coughing, there is added sound on left sternal
border .in ecg you will find
a. ST changes
b. PR prolongation
c. Hyper voltage
the correct answer A .
250) The most common site for visceral hemangioma is
a. Liver ( most common site for visceral hemangioma )
A hepatic hemangioma is the most common noncancerous tumor of the liver. It is believed to
be a birth defect. Hepatic hemangiomas can occur at any time, but are most common in people
in their 30s - 50s. Women are affected more often than men, and usually have bigger tumors
than men. Babies may develop a type of hepatic hemangioma called benign infantile
hemangioendothelioma (also called multinodular hepatic hemangiomatosis). This rare,
noncancerous tumor has been linked to high rates of heart failure and death in infants. Infants
are usually diagnosed by the time they are 6 months old.
251) Child with large periorbital hemangioma, if this hemangioma cause obstruction to vision ,
when will be permanent decrease in visual acuity
a. After obstruction by one day
b. By 1 week
c. By 3 months
d. By 6 months
Correct answer is B
…………………………………………………………………………………………………………………………………………………
252) The symptoms of soft tissue sarcoma
a. Progressive enlarging mass ( on growing )
In their early stages, soft tissue sarcomas usually do not cause symptoms. Because soft tissue
is relatively elastic, tumors can grow rather large, pushing aside normal tissue, before they are
felt or cause any problems. The first noticeable symptom is usually a painless lump or swelling.
As the tumor grows, it may cause other symptoms, such as pain or soreness, as it presses
against nearby nerves and muscles. If in the abdomen it can cause abdominal pains commonly
mistaken for menstrual cramps, indigestion, or cause constipation
…………………………………………………………………………………………………………………………………………………
253) 35 year female with bilateral breast pain , that decrease after menstruation , the breast is
nodular with prominent 3 cm mass subareolar , axillary lymph nodes are not enlarged , what
you will do
a. Mammography followed by US
b. See her next cycle
c. Fine needle biopsy followed by tissue studies
( the correct answer is B )
254) Pregnant with bleeding for 12 hours and tissue, the cervix is 1 cm
a. Complete abortion
. b. Incomplete abortion
. c. Molar pregnancy
the correct answer is B (please check Q275 in pearson 1000 file for explanation)

255) 2 month infant with vomiting after each meal , he is in 50 centile , He passed meconium
early and stool , diagnosis is
a. Midgut volvulus
b. Meconium ileus (not passing early meconium and stool)
c. Hischsprung disease (not passing early meconium and stool)
I think this case is pyloric stenosis. If not mention choose A
…………………………………………………………………………………………………………………………………………………
256) Patient with dysphagia to solid and liquid , and regurgitation , by barium there is non
peristalsis dilatation of esophagus and air-fluid level and tapering end. diagnosis is
a. Osophageal spasm
b. Achalasia
c. Osophageal cancer
the correct answer is B
…………………………………………………………………………………………………………………………………………………
257) The most common cause of cough in adults is
a. Asthma
b. GERD
c. Postnasal drip
The correct answer C
The most common causes of acute cough is respiratory viral infection. The most common cause
of chronic cough are postnasal drip, asthma, and acid reflux from the stomach. These three
causes are responsible for up to 90 percent of all cases of chronic cough.
258) Girl with amenorrhea for many months . BMI is 20 and is stable over last 5 years .
diagnosis
a.Eating disorder
b.Pituitary adenoma
answer : B. BMI stable and normal.
…………………………………………………………………………………………………………………………………………………
259) Old female with itching of vulva , by examination there is pale and thin vagina , no
discharge . what is management
a. Estrogen cream
b. Corticosteroid cream
c. Fluconazole
the correct answer is A
Some women will opt for prescription medications as they go through menopause. The
most common prescriptions include:
1 -Hormone replacement therapy or anti-depressants to minimize hot flashes.
2 -Fosamax or Actonel (non-hormonal medications) to reduce bone loss and reduce the risk of
fractures .
3 -Selective estrogen receptor modulators (SERMs), which mimic estrogen's beneficial effects
on bone density.
4-Vaginal estrogen, administered locally, to relieve vaginal dryness and discomfort during
intercourse
…………………………………………………………………………………………………………………………………………………
260) Patient with dysuria , frequency , urgency , but no flank pain , what is the treatment
a. Ciprofloxacin po od for 3-5 days
b. Norfocin po od for 7 – 14 days
the correct answer is b
261) Patient with flank pain , fever , vomiting , treatment is
a. Hospitalization and intravenous antibiotics and fluid
This is most likely a case of pyelonephritis which need urgent hospitalization
262) Patient present with mid face pain , erythematous lesions and vesicles on periorbital and
forehead , the pain is at nose , nose is erythematous . what is diagnosis
a. Roseola .
b. HSV .
c. Herpes zoster
the correct answer is C

…………………………………………………………………………………………………………………………………………………
263) Male with itching in groin erythematous lesions and some have clear centers, what is
diagnosis :
a. Psoriasis
b. Tinea cruris
c. Erythrasma
the correct answer is B
Patients with tinea cruris report pruritus and rash in the groin. A history of previous
episodes of a similar problem usually is elicited. Additional historical information in patients
with tinea cruris may include recently visiting a tropical climate, wearing tight-fitting clothes
(including bathing suits) for extended periods, sharing clothing with others, participating in
sports, or coexisting diabetes mellitus or obesity. Prison inmates, members of the armed forces,
members of athletic teams, and people who wear tight clothing may be subject to independent
or additional risk for dermatophytosis. Large patches of erythema with central clearing are
centered on the inguinal creases and extend distally down the medial aspects of the thighs and
proximally to the lower abdomen and pubic area.
264) Vasoconstrictive nasal drops complication
a. Rebound phenomenon
Rebound congestion (Rhinitis medicamentosa) is known side effect of decongestant use.
Commonly used decongestants are: Ephidrine and phylyl ephrine  both short acting
Xylometazoline (Long acting)
Oxymetazoline (Long acting)

265) The useful exercise for osteoarthritis in old age to maintain muscle and bone
a. Conditioning and low repetition weight training
b. Walking and weight exercise
the correct answer is B
Exercise is one of the best treatments for osteoarthritis. The best exercises for osteoarthritis
suffers depend on what joints are affected. Swimming, walking, and cycling are often the best
exercises for people with osteoarthritis. Try to get thirty minutes of exercise five times per
week. The key is to start slowly.
…………………………………………………………………………………………………………………………………………………
266) Unilateral worsening headache, nausea , exacerbated by movement and aggravated by
light in 17 old girl.
a. Migraine (Photophobia, vomiting)
b. Cluster
the correct answer is A
267) Diet supplement for osteoarthritis
a. Ginger
A large number of dietary supplements are promoted to patients with osteoarthritis and
as many as one third of those patients have used a supplement to treat their condition.
Glucosamine-containing supplements are among the most commonly used products for
osteoarthritis. Although the evidence is not entirely consistent, most research suggests that
glucosamine sulfate can improve symptoms of pain related to osteoarthritis, as well as slow
disease progression in patients with osteoarthritis of the knee. Chondroitin sulfate also appears
to reduce osteoarthritis symptoms and is often combined with glucosamine, but there is no
reliable evidence that the combination is more effective than either agent alone.
Sadenosylmethionine may reduce pain but high costs and product quality issues limit its use.
Several other supplements are promoted for treating osteoarthritis, such as
methylsulfonylmethane, Harpagophytum procumbence (devil's claw), Curcuma longa
(turmeric), and Zingiber officinale (ginger), but there is insufficient reliable evidence regarding
long-term safety or effectiveness.
………………………………………………………………………………………………………………………………………………

268) Old male with abdominal pain , nausea , WBC 7. What is true about appendicitis in
elderly?
a. CT is not useful for diagnosis.
b. WBC is often normal.
c. Rupture is common
d. If there is no fever the diagnosis of appendicitis is unlikely
e. Anemia is common
The correct answer is C
Appendicitis in elderly
Appendicitis is a less common cause of abdominal pain in elderly patients than in younger
patients, but the incidence among elderly patients appears to be rising. Only approximately
10% of cases of acute appendicitis occur in patients older than 60 years, whereas one half of all
deaths from appendicitis occur in this age group. The rate of perforation in elderly patients is
approximately 50%, 5 times higher than in younger adults. This is largely because 75% of elderly
patients wait more than 24 hours to seek medical attention. The diagnosis can be difficult to
make, since more than one half of patients in this age group do not present with fever or
leukocytosis. Further confusing the picture, approximately one third do not localize pain to the
right lower quadrant, and one fourth do not have appreciable right lower quadrant tenderness.
Only 20% of elderly patients present with anorexia, fever, right lower quadrant pain, and
leukocytosis. The initial diagnosis is incorrect in 40-50% of patients in this age range .
269) Old patient with bilateral enlarged knee , no history of trauma , no tenderness , normal
ESR and C-reactive proteins . the diagnosis is
a. Osteoarthritis
b. Gout
c. Infectious arthritis
The correct answer is A
………………………………………………………………………………………………………………………………………………
270) Patient has decreased visual acuity bilateral, but more in rt side , visual field is not
affected , in fundus there is irregular pigmentations and early cataract formation .
a. Refer to ophthalmologist for laser therapy
b. Refer to ophthalmologist for cataract surgery
the correct answer is A
………………………………………………………………………………………………………………………………………………
271) What is the most common treatment for juvenile rheumatoid arthritis
a. Intra articular injection of steroid
b. Oral steroid
c. Paracetamol
d. Penicillamine
e. Aspirin
The correct answer is E
……………………………………………………………………………………………………………………………………………
272) Which of the following decrease mortality after MI
a. Metoprolol
b. Nitroglycerine
c. Thiazide
d. Morphine
The correct answer is A
only b-blocker and ASA are mortality benefit for treatment of angina
273) The cardiac arrest in children is uncommon but if occur it will be due to
a. Primary
b. Respiratory arrest
c. hypovolemic shock
d. neurogenic shock
the correct answer is B
Progressive respiratory insufficiency accounts for 60% of all pediatric arrests.
………………………………………………………………………………………………………………………………………………
274) Old female with recurrent fracture , Vitamin D insufficiency and smoker . which
exogenous factor has the greatest exogenous side effect on osteoporosis.
a. Old age
b. Smoking
c. . Vit D insufficiency
e. Recurrent fracture
The correct answer is B
275) patient presented with sudden chest pain and dyspnea , tactile vocal fremitus and chest
movement is decreased , by x-ray there is decreased pulmonary marking in left side , diagnosis
a. atelectasis of left lung
b. spontaneous pneumothorax
c. pulmonary embolism
Answer is ( B )

276) boy after running for hours, has pain in knee and mass on upper surface of tibia
a. Osgood scatter disease
b. Iliotibial band
the correct answer is A
………………………………………………………………………………………………………………………………………………
278) pancreatitis
a. Amylase is slowly rising but remain for days
b. Amylase is more specific but less sensitive than lipase
c. Ranson criteria has severity (predictive) in acute pancreatitis
d. Pain is increased by sitting and relieved by lying down
e. Contraceptive pills is associated
the correct answer is c
The Ranson and Glasgow scoring systems are based on such parameters and have been
shown to have an 80% sensitivity for predicting a severe attack, although only after 48 hours
following presentation. Check Q298 in pearson file for more info
279) Patient has fever , night sweating , bloody sputum , weight loss , ppd test was positive . x-
ray show infiltrate in apex of lung , ppd test is now reactionary , diagnosis
a. Activation of primary TB
b. sarcoidosis
c. Case control is
d. Backward study
the correct answer is A
The tuberculosis skin test (also known as the tuberculin test or PPD test) is a test used to
determine if someone has developed an immune response to the bacterium that causes
tuberculosis (TB).
……………………………………………………………………………………………………………………………………………
280) patient with DM presented with limited or decreased range of movement passive and
active of all directions of shoulder
a. frozen shoulder b. impingement syndrome c. osteoarthritis
the correct answer is A
……………………………………………………………………………………………………………………………………………
281) 48 years old with irregular menses presented with fatigue and no menstruation for 3
months with increased pigmentation around the vaginal area with no other symptoms. ur next
step would be
a. reassure the patient
b. do a pregnancy test
c. do ultrasound
the correct answer B

282) 3 years old presented with shortness of breath and cough at night which resolved by itself
in 2 days. he has Hx of rash on his hands and allergic rhinitis. he most likely had
a. croup b. bronchial asthma c. epiglottitis
History is short. Could be Spasmodic croup.
Spasmodic croup (recurrent croup) typically presents at night with the sudden onset of
"croupy" cough and stridor. The child may have had mild upper respiratory complaints prior to
this, but more often has behaved and appeared completely well prior to the onset of
symptoms. Allergic factors may cause recurrent croup due to respiratory epithelial changes
from the viral infection.
……………………………………………………………………………………………………………………………………………
283) a man went on vacation. he noticed a white patch in his chest which became more clear
after getting a sun tan which was spread on his chest. ur Dx is
a . pytriasis versicolor b. vitiligo c. pytiriasis rosacea
the correct answer A
284) a 4 years old presented with 2 day history of shortness of breath a seal like cough with no
sputum and mild fever. on examination he did not look I'll or in distress
a. acute epiglottitis b. croup c. angioedema
The correct answer B
Seal like barking cough is typical of croup

285)65 years old came with knee pain. and limited movement. on examination had crepition on
knee. dx
a. rheumatoid arthritis b. osteoporosis c. osteoarthritis
The correct answer C
286) which vitamin is given to new born to stop bleeding
a. vit. A b. vit. D c. vit. K d. vit E e. vit C
the correct answer C
………………………………………………………………………………………………………………………………………………
287) a child presented with erythematous pharynx, with cervical lymph nodes and rapid
streptolysin test negative and low grade fever with positive EBV. it next step
a. give antibiotics and anti-pyretic
b. give anti pyretic and fluids
d. culture and sensitivity
the correct answer B (Supportive treatment)
288) the most common cause of intracerebral or intra parenchymal bleeding is.
a. Hypertensive angiopathy
b. aneurysm c. AV malformation
the correct answer is A
289) young adult Sickle cell patients are commonly affected with
a. dementia b. multiple cerebral infarcts
the correct answer B
………………………………………………………………………………………………………………………………………………
290) 80 years old living in nursing home for the last 3 months. his wife died 6 months ago and
he had a coronary artery disease in the last month. he is now forgetful especially of short term
memory and decrease eye contact with and loss of interest. dx
a. Alzheimer b. depression c. hypothyroidism
Memory loss specially recently acquired memory and features of depression seen in Alzheimers
the correct answer A
………………………………………………………………………………………………………………………………………………
291) thyroid cancer can be from
a. hypothyroidism b. graves disease c. toxic nodule
Correct answer is B
………………………………………………………………………………………………………………………………………………
292) a mother came with her son who is 7 years old very active never sitting in class and with
poor concentration. ur management would be.
a. olanzipine b. amitilyne c. aloxane
No choice here looks reasonable. Could not find this Aloxane
this case is "attention –deficit hyperactivity disorder" treatment: 1- Methylphenidate
2- Dextroamphetamine 3- Aderall 4- Atomoxetine 5- Pemoline 6- Antidepressant ( SSRI )

293) a patient presented with progressive weakness on swallowing with diplopia and
fatigability. the most likely underlying cause of her disease is.
a. antibody against acetylcholine receptors
A case of Myasthenia
a condition causing abnormal weakness of certain muscles.
a rare chronic autoimmune disease marked by muscular weakness without atrophy, and caused
by a defect in the action of acetylcholine at neuromuscular junctions
…………………………………………………………………………………………………………………………………………

294) an 69 year old non diabetic. with mild hypertension and no hx of Coronary heart
disease. the best drug in treatment is.
a. thiazides b. ACEI c. ARB d. CCB
the correct answer A
………………………………………………………………………………………………………………………………………………
295) which of the following anti hypertensive is contraindicated for an uncontrolled diabetic
patient
a. hydrochlorothiazide b. Losartan c. hydralazine d. spironolactone
the correct answer is A
-side effect of thiazide : hyperglycemia
………………………………………………………………………………………………………………………………………………
296) a wound stays in it's primary inflammation untill
a. Escher formation b. epithelization c. after 24 hours d. wound cleaning
the correct answer D
……………………………………………………………………………………………………………………………………………
297) 70 years old male patient with mild urinary dripping and hesitancy ur Dx is mild BPH. ur
next step in management is
a. transurethral retrograde prostatectomy
b. start on medication
c. open prostatectomy
the correct answer B
………………………………………………………………………………………………………………………………………………
298) one of the following food is known to reduce cancer a. fibers
………………………………………………………………………………………………………………………………………………
299) smoking is a definitive risk of
a. sq.c.ca of bladder
b. liver
c. breast the correct answer is A
……………………………………………………………………………………………………………………………………………
300) exercise recommended for patients with CAD. is
a. isometric b. isotonic c. yoga
B
………………………………………………………………………………………………………………………………………………
301)what is the definitive treatment of frostbites a, Rewarming

………………………………………………………………………………………………………………………………………

302) the most common cause of failure to thrive in pediatric is a. malnutrition

………………………………………………………………………………………………………………………………

303) which of the following causes the highest maternal mortality in pregnancy
a. toxoplasma b. hyperbilirubinemia c. ????
Missing options here
Haemorrhage, hypertensive disorders and sepsis major causes
" The major causes of maternal death are bacterial infection, variants of gestational
hypertension including pre-eclampsia and HELLP syndrome, obstetrical hemorrhage, ectopic
pregnancy, puerperal sepsis (childbed fever),

304) Ttt of frostbite


a. Immersion in water 40-45 C
b. Debridement
c. Leave it at room temperature
The correct answer is A

………………………………………………………………………………………………………………………………………

305) Case about a child both RBS, FBS are elevated so he has DM1…what's the type of HLA
a.DR3 b.DR4 c.DR5 d.DR6 e.DR7
the correct answer is A (or B)
Types of HLA : DR2 : Good's pasture syndrome & multiple myeloma
DR3 : D.M , SLE , & grave's disease
DR27 : ankylosing spondylaitis & reiter's syndrome
B51 : behceat's disease
D11 : hashimot's disease
…………………………………………………………………………………………………………………………………
306) Definition of PPV ?
Predictive positive value ( PPV ) : proportion of people with a positive test who have a disease :
a / (a+b)
Prevalence of Disease= Tdisease/ Total × 100
Sensitivity: A/(A+C) × 100 or TP/TP+FNx100
Specificity: D/(D+B) × 100 or FN/FN+FP
Positive Predictive Value: A/(A+B) × 100
Negative Predictive Value: D/(D+C) × 100

………………………………………………………………………………………………………………………………………

307) What's the most common case in PHC centers


a.UTI
b.HTN
c. Coryza
the correct answer is C

308) all of the following will decrease (increase not decrease) pt compliance except:
a. involve pt in the plan
b. make simplified regimen
c. give easy written instructions
d. make appointments flexible
e. warn the patient about the danger of missing a pill
the correct answer is E Not sure why other options will decrease the patient compliance

309) Case about old diabetic patient who still have hyperglycemia despite increase insulin
dose…the problem with insulin in obese patients is
a. Post receptor resistance

………………………………………………………………………………………………………………………………………

310) Case about a child with drooling, fever, barking cough in sitting position,
dx: a. Croup
b. Bronchiolitis
c. Pneumonia
The correct answer is a
Drooling saliva, sitting position and high fever all seen in epiglottitis whereas barking cough is
seen in Croup.
Cough is absent in epiglottitis

……………………………………………………………………………………………………………………………………

311) Mother came to you after her son had hematoma under the nail dt injury:
a. Send home with a pad on the head
b. Send home with acetaminophen
c. Do wedge resection
d. Evacuate the hematoma
The correct answer is D
312) What vaccine u'll give to a SCD ( sickle cell disease ) child
a.HBV
b. H. Influenza
c. pneumococcal
d. both A and B
e. all of the above
the correct answer is C

……………………………………………………………………………………………………………………………………

313) a mother with HBsAg positive came with her child 6 yrs old who has HBsAg +ve what will
you give him:
a. oral polio, DTP, MMR
the correct answer is A
All vaccine can be given except Hep B vaccine

………………………………………………………………………………………………………………………………………

314) Patient wil LLQ pain, vomiting, fever, high WBC (17.000), tenderness and rebound
tenderness
a. Diverticulitis b. Sigmoid volvulus c. Appendicitis d. Toxic enteritis
The correct answer is A
………………………………………………………………………………………………………………………………………………
315) What's the organism responsible for pseudomembranous colitis:
a. Pseudomonas
b. Clostridium diffcile
c. E.coli
d. Enterococcus fecalis
The correct answer is B

316) Mother came with her child who had botulism, what you will advice her:
a. Never eat canned food again
b. Store canned food at home
c. Boil canned food for 40-50 min
d. Check expiry date of canned food
The correct answer is D

………………………………………………………………………………………………………………………………………

317) Old pt presented with abdominal pain, back pain, pulsatile abdomen what's the step to
confirm dx: this is a case of aortic aneurysm
a. Abdominal US b. Abdominal CT c. Abdominal MRI The correct answer is a b -initial
investigation US – CONFIRM by CT
……………………………………………………………………………………………………………………………………………
318) 18 months old came with bite by her brother, what you will do:
a. Give Augmentin b. Give tetanus toxoid c. Suture
Correct answer is A
No Tetanus toxoid ad the patient is 18 months old and must be already on immunization
schedule.
Suturing only if laceration is big

………………………………………………………………………………………………………………………………………

319) 19 yrs old after bike accident, he can't bring the spoon in front of himself to eat, lesion is
in:
a. Temporal lobe b. Cerebellum c. Parietal lobe d. Occipital lobe
The correct answer is B

320) How to dx DVT:


a. Contrast venography b. Duplex US
The correct answer is B

………………………………………………………………………………………………………………………………………

321) Pt came after RTA, GCS 14, near complete amputation of the arm, 1st step:
a. Secure air way b. Tourniquet on the arm
The correct answer is A
ABC first 3
322) Pt with hx of prolonged heavy bleeding 2 hrs post-partum, you will give:
a. Ringers lactate
b. NS
c. NS+ packed erythrocytes
The correct answer is C

………………………………………………………………………………………………………………………………………

323) What is special about placenta abruption:


a. Abnormal uterine contractions
b. PV bleeding
c. Fetal distress
The correct answer is B

………………………………………………………………………………………………………………………………………

324) Pt 34 wks, hx of PV bleeding for many hrs, dark blood, abdominal tenderness, FHR 120,
uterine contractions every 3 min,
Dx: a. Abruption placenta

325) One of the steps in managing epistaxis:


a. Packing the nose
b. Press the fleshy parts of nostrils
c. Put patient of lateral lying position
The correct answer is B. Initial step is pressing the fleshy part of nose. If not controlled then go
for nasal packing

……………………………………………………………………………………………………………………………………

326) Pt with hx of diarrhoea, abdominal pain, agitation, headache, dizziness, weakness,


pulsatile thyroid, unsteady gate. Examination was normal. Dx:
a. Hypochondriasis
b. Somatization disorder
c. Thyroid Ca
d. Anxiety
The correct answer is B

Mental disorder characterized by recurring, multiple, and current, clinically


significant complaints about somatic symptoms, although it is no longer
considered a clinical diagnosis.

………………………………………………………………………………………………………………………………………

327) Child with headache that increase by changing head position, unilateral, photophobia
a. Sinus headache b. Migraine c. Cluster headache d. Tension headache
The correct answer is B
328) Young pt with pain in LL after running 3 KM, more at night, swelling, XR was NL
a. Stress fracture
As the name suggests, a stress fracture is a small crack in any of the weight-bearing bones of
the body. Runners can get a wide variety of stress fractures, but the most common stress
fractures in runners are (in order) tibia (bigger shin bone), metatarsal, femur, fibula and
navicular.

329) HTN pt, with decrease vision, fundal exam showed increase cupping of optic disc dx:
a. Open angle glaucoma
b. Closed angle glaucoma
c. Cataract
d. HTN changes
The correct answer is A
The optic disc can be flat or it can have a certain amount of normal cupping. But glaucoma,
which is in most cases associated with an increase in intraocular pressure, often produces
additional pathological cupping of the optic disc. The pink rim of disc contains nerve fibers.

……………………………………………………………………………………………………………………………………

330 ) Pt with unilateral red eye, tearing, clear tears no swelling or discharge:
a. Give topical antihistamine
b. Give topical AB
c. Give systemic AB
d. Reassurance
The correct answer is D

………………………………………………………………………………………………………………………………………

331) Business man went to Pakistan, came with bloody diarrhea, stool examination showed
trophozoite with RBC inclusion, Dx:
a. Amebic dysentery (Entamoeba histolytica )

………………………………………………………………………………………………………………………………………

332) 5 yrs old child with abdominal pain after 2 wks of URTI, HB 8, retics 12% WBC NL
peripheral blood smear showed target cells, RBC inclusions dx:
a. SCA (the only hemolytic anemia in the answers) ?????
333) Child had hb electrophoresis showed hb 2% HBA 40 HBS 35 HBF 5 HBA2
a. Sickle cell trait
b. Thalassemia major
Presence of Haemoglobin S is the clue here. Answer is A

334) young pt with hx of cough, chest pain, fever CXR showed RT lower lobe infiltrate:
a. Amoxicillin b. Cefuroxime c. Imipenam d. Ciprofloxacin
The correct answer is A
Initial treatment for community acquired pneumonia

………………………………………………………………………………………………………………………………………

335) Best thing to reduce mortality rate in COPD:


a. Home O2 therapy
b. Enalipril
c. Stop smoking
The correct answer is C
336) Drug that will delay need of surgery in AR:
a. digoxin
b. verapamil
c. Nifedipine
d. enalpril
the correct answer is C

…………………………………………………………………………………………………………………………………

337) Child with atopic dermatitis, what you will give other than cortisone
a. There were many drugs, steroids and AB and only one strange name maybe moisturizer.
That's the answer
Treatment of atopic dermatitis is centered around rehydrating the skin with emollients like
petroleum jelly and the cautious use of topical steroids to reduce inflammation and itching.
Oral antihistamines may be helpful in breaking the "itch-scratch" cycle
338) A man had increase shoe size and jaw, the responsible is:
a. ACTH
b. Somatostatin
c. TSH
d. Cortisone
The correct answer is B

339) Child with skin rash, pericarditis, arthritis dx: Kawasaki


…………………………………………………………………………………………………………………………………

340) Pic of skin with purple flat topped polygonal papules, dx: a. Lichen plannus

………………………………………………………………………………………………………………………………………

341) The best advice to patient travelling is:


a. Boiled water b. Ice c. Water d. Salad and under cooked sea shells
The correct answer is A

………………………………………………………………………………………………………………………………

342) Seldinfil is contraindicated with:


a. Nitrate b. Methyldopa c. Gabapentine
The correct answer is A

……………………………………………………………………………………………………………………………………

343) Pt with TB, had ocular toxicity symptoms, the drug responsible is:
a. INH b. Ethambutol c. Rifampicin d. Streptomycin
The correct answer is B
INH : peripheral neuritis and hepatitis . so add ( B6 )
Ethambutol : optic neuritis
Rifampicin : body fluid orange

344) Another pt treated for TB started to develop numbness, the vit deficient is:
a. Thiamin
b. Niacin
c. Pyridoxine
d. Vit C
THE correct answer is C
………………………………………………………………………………………………………………………………………………
345) Pt with bilateral breast disease, dx:
a. Paget b. Papilloma c. Medullary d. Lobular
correct answer is D.
Both papilloma and lobular can cause bilateral breast disease
346) Child fell on her elbow and had abrasion, now swelling is more, tenderness, redness,
swelling is demarcated (they gave dimensions) child has fever. Dx:
a. Gonococcal arthritis
b. Synovitis
c. Cellulitis of elbow
The correct answer is C

………………………………………………………………………………………………………………………………………

347) About burn


a. You will give 1\2 fluid in the 1st 8 hrs
b. 1\4 in the 1st 8 hrs
The correct answer is A
Parkland formula : fluid in first 24 hours = 4 x wt. in kg x % BSA
50% in first 8 hours and another in next 16 hours

348) You r supposed to keep a child NPO he's 25 kgs, how much you will give:
a. 1300
b. 1400
c. 1500
d. 1600
The correct answer is D
For the first 10 kg of weight, a child needs 100 mL per kg of weight. For the next 10 kg of
weight (11-20 kg), a child only needs 50 mL per kg of weight. for anything over 20 kg (21 kg of
weight and higher), the child only needs 20 mL per kg of weight. Take a 35 kg child, for
example. He needs 1000 mL for his first 10 kg of weight (10 kg x 100 mL), 500 mL for his second
10 kg of weight (10 kg x 50 mL), and 20 mL/kg for any weight above 20 kg (15 kg x 20 mL). A 35
kg child, therefore, needs approximately 1800mL of water or free liquids.
………………………………………………………………………………………………………………………………………………
349) Young patient with pharyngitis, inflammation of oral mucosa and lips that has whitish
cover and erythmatous base, febrile, splenomegaly. Dx: (this is infectious mono)
a. Scarlet fever b. EBV c. HZV
The correct answer is B

………………………………………………………………………………………………………………………………………

350) Female with greenish vaginal discharge, red cervix. Dx: Trichomoniasis

………………………………………………………………………………………………………………………………………

351) Another female with malodorous discharge and pain maybe. Dx: Bacterial vaginosis

………………………………………………………………………………………………………………………………………

352) The best way to reduce the weight in children is:


a. stop fat intake
b. Decrease calories intake
c. Drink a lot of water
The correct answer is B
353) You have to advice a teenager that mainly eats fast food to take:
a. Folic acid and Ca
b. Vit C and Ca
c. Ca alone
The correct answer is A (choose vit C & folic acid if present)

………………………………………………………………………………………………………………………………………

354) Patient has symptoms of infection, desquamation of hands and feet, BP 170\110 dx:
a. Syphilis
b. Toxic shock syndrome
c. Scarlet fever
The correct answer is C
Toxic shock syndrome has hypotension
355) Snellen chart, if patient can read up to line 3, how much is his vision loss :

Q not complete: but this pt. see letters at 20 feet , where normal person see it at 70 feet.

356) Diabetic mother asking about risk of diabetes related congenital defect. It will be in:
a. 1st trimester b. 2nd trimester c. 3rd The correct answer is A

………………………………………………………………………………………………………………………………………

357) If diabetic mother blood sugar is always high despite of insulin, neonate complication will
mostly be:
a. Maternal hyperglycemia
b. Maternal hypoglycemia
c. Neonatal hypoglycemia
d. Neonatal hyperglycemia
The correct answer is C

………………………………………………………………………………………………………………………………………
358) Obsessive neurosis patients will have:
a. Major depression
b. Lake of insight
c. Schizophrenia
The correct answer is A

………………………………………………………………………………………………………………………………………

359) Before giving bipolar patient lithium you will do all of the following except:
a. TFT b. LFT c. RFT d. Pregnancy test
The correct answer is B

360) Patient came with PV small bleeding (she fell down and felt decrease in fetal movement
but on examination baby was ok) all her 3 previous pregnancies were normal, you will do:
a. Immediate CS
b. Careful observation of the bleeding
c. Medication (strange name)
d. Mg sulphate
Answer could be B. AS baby was ok on examination we can wait and observe the bleeding

………………………………………………………………………………………………………………………………………

361) drinking of dirty water causes a. hepatitis A b. B c. C d. D


The correct answer is A
362) Pt came to you missing her period for 7 wks, she had minimal bleeding and abdominal
pain, +ve home pregnancy test, 1st thing to order is:
a. BHCG
b. US
c. Drugs maybe
The correct answer is A
………………………………………………………………………………………………………………………………………

363) Patient came to you and you suspect pre eclampsia, which of the following will make it
most likely:
a. Elevated blood pressure
b. Decrease fetal movement c. ??
The correct answer is A

………………………………………………………………………………………………………………………………………

364) Old patient male, hematuria, passing red clots and RT testicular pain:
a. Testicular Ca
b. RCC ( renal cell carcinoma )
c. Cystitis
The correct answer is B

365) to get more information from the pt


a. open end question
(avoid leading question to get more information )

………………………………………………………………………………………………………………………………………

366) gingivitis most likely cause


a.HSV
( The most common cause of gingivitis is poor oral hygiene that encourages plaque to form.)
Acute herpetic gingivostomatitis is a painful viral infection of the gums and other parts of the
mouth caused by the herpes virus

………………………………………………………………………………………………………………………………………

367) watery discharge from eye , conjunctivitis treated by


a. topical corticosteroid
History is unclear. Could be viral or allergic conjunctivitis.
For viral conjunctivitis supportive treatment. Antiviral drops only for Herpes virus.
For allergic conjunctivitis anti histamines and saline drops
368) blow out fracture eyelid swelling , redness other symptoms
a. present air fluid level
b. enopthalmos
The correct answer is B
Periorbital ecchymosis and edema accompanied by pain are obvious external signs and
symptoms, respectively. Enophthalmos is possible but initially can be obscured by surrounding
tissue swelling. This swelling can restrict ocular motility, giving the impression of soft tissue or
inferior rectus entrapment. Retrobulbar or peribulbar hemorrhage may be heralded by
proptosis. A bony step-off of the orbital rim and point tenderness are possible during palpation.

………………………………………………………………………………………………………………………………………

369) giemsa stained blood film a. malaria

370) regarding peritonitis


a. Complicated appendectomy the cause is anaerobe organism
b. rigidity and the cause is paralytic ileus
c. can be caused by chemical erosions.
The correct answer is C

………………………………………………………………………………………………………………………………………

371) fracture of humerus associated with


a. radial N injury
b. median nerve injury
the correct answer is A ( radial groove )
Radial nerve injury occurs in as many as 18% of humeral shaft fractures.[55] Although the
oblique distal third humeral fracture (Holstein-Lewis) is better known for an association with
radial nerve palsy than other humeral shaft fractures are, such palsy most commonly occurs
with middle third humeral fractures.[56] Most of these nerve injuries are neurapraxic or
axonotmetic types, 90% of which resolve to at least grade IV strength in 3-4 months.

………………………………………………………………………………………………………………………………………
372) Diagnosis of acute lymphocytic leukemia ALL
The total number of white blood cells may be decreased, normal, or increased, but the number
of red blood cells and platelets is almost always decreased. In addition, very immature white
blood cells (blasts) are present in blood samples examined under a microscope. A bone marrow
biopsy is almost always done to confirm the diagnosis and to distinguish ALL from other types
of leukemia

……………………………………………………………………………………………………………………………………

373) diagnosis of hemochromatosis a. serum ferritin?!


Hemochromatosis is suggested by a persistently elevated transferrin saturation in the absence
of other causes of iron overload. It is the initial test of choice .
Ferritin concentration can be high in other conditions, such as infections, inflammations, and
liver disease . Ferritin levels are less sensitive than transferrin saturation in screening tests for
hemochromatosis.

374) Kawasaki disease associated with a. strawberry tongue


Kawasaki disease Multisystem acute vasculitis that primary affected young children. Fever plus
four or more of the following criteria for diagnosis: 1-fever > 40 C for at least five days 2-
bilateral, nonexudative, painless conjunctivitis 3-polymorphous rash ( primarily truncal ) 4-
cervical lymphadenopathy ( often painful and unilateral ) 5- diffuse mucous membrane
erythema ( strawberry tongue ) , dry red 6- erythema of palm and sole 7- other maindfestation
: gallbladder hydrops, hepatitis, arthritis
Untreated Kawasaki diease can lead to coronary aneurysms and even MI Treatment : 1-high dos
ASA ( for fever and inflammation) & IVIG ( to prevent aneurysmal ) 2-referral to pediatric
cardiologist

………………………………………………………………………………………………………………………………………

375) most common tumor in children


a. ALL
b. rhabdomyosarcoma
c. wilm's tumor
the correct answer is A
-ALL : most common childhood cancer -rhabdomyosarcoma : most common soft tissue tumor
-wilm's tumor: most common intra- abdominal childhood tumor
-------------------------------------------------------------------------------------------------------------------------
376) Most common intra- abdominal tumor in children:
a. wilm's tumor
b. lymphoma
the correct answer is A
the most common intra-abdominal tumor in children are neruoblastoma and wilm's tumor.

377) pt c/o of hypo pigmented skin , nerve thinking diagnosis


a.leprosy
Leprosy : chronic granulomatous infection caused by Mycobacterium leprae, acid-fast bacillus
preferentially affecting cooler regions of body
( e.g skin , mucous membrane, peripheral nerve) .
diagnosis by skin lesion (hypopigmentation or hyperpigmentation macule ) and enlarge nerves
with sensory loss.
………………………………………………………………………………………………………………………………………………
378) regrading COPD to reduce complication we should give
a. theophylline
b. pneumococcal vaccine
c. smoking cessation.
The correct answer is C
…………………………………………………………………………………………………………………………………………
379) pseud-gout its
a. CACO3 b. CACL3
The correct answer is A
Gout : deposition of ( MSUM ) monosodium urate monohydrate –ve of birefringent , needle
shape,
Pseudo gout : deposition of ( cppd ) calcium pyrophosphates dehydrate crystal , +ve
birefringent , rhomboid shape, ( CACO3)
380) man with history of alcohol association with
a. high MCV
b. folic acid deficiency
c. B12 deficiency
d. hepatitis
The correct answer is B
………………………………………………………………………………………………………………………………………………
381) neonate 9 days on breast feed develop jaundice
a. breast feed jaundice
b. pathological jaundice
c. physiological jaundice
the correct answer is A
1-pathological jaundice : conjugated ( direct ) bilirubin, start in 1st 24 hours, bilirubin > 15 mg
/dl , persistent 1 week in term infant and 2 weeks in preterm infant
2-physiological jaundice: unconjugated ( indirect ) bilirubin , start in 2- 3 days, bilirubin < 15
mg/dl , persistent 1 week in term infant and 2 weeks in preterm infant.
3-breast feeding jaundice : unconjugated ( indirect ) bilirubin , start in 7 days of life , persistent
to one or more months.
……………………………………………………………………………………………………………………………………………
382) lady c/o headache band like pain
a. tension headache
………………………………………………………………………………………………………………………………………………
383) regarding breast screening
a. self breast examination early detection of tumor
b. mammogram not advise before 35 y I Think A correct here.
………………………………………………………………………………………………………………………………………………
384) 19 yrs old c/o abdominal pain within menstruation for last 6 years diagnosis
a. primary dysmenorrhea
b. secondary dysmenorrheal
The correct answer is A
Primary dysmenorrhea :
Onset within 6 months after menarche . Lower abdominal/pelvic pain begins with onset of
menses and lasts 8-72 hours

385) bilateral breast mass diagnosis


a. ductal carcinoma b. Paget's disease
A Intraductal papilloma and Lobular carcinoma
………………………………………………………………………………………………………………………………………………
386) beriberi cause of deficiency a
. VIT B1 b. VIT B2 c. VIT B3
The correct answer is A. Thiamine deficiency causes beri beri
………………………………………………………………………………………………………………………………………………
387) Asystole
a. adrenaline
b. atropine
The correct answer is A
asystole has only 2 drugs
Epinephrine and vasopressin
Atropine has no role during CPR and ACLS
………………………………………………………………………………………………………………………………………………
388) pt has diarrhea, dermatitis and dementia diagnosis
a. pellagra
Niacin deficiency causes pellagra
………………………………………………………………………………………………………………………………………………
389) regarding injectable progesterone a . can cause skin prob ?!!
Injected progesterone ( medroxy-progestrone ) : advantages : light or no periods, safe with
breastfeeding, IM injection every 3 months.
Disadvantages: irregular bleeding, wt. gain, decrease bone mineral density ( reversible ) , delay
fertility discontinue .
………………………………………………………………………………………………………………………………………………
390) secondary prevention is
a. coronary bypass grafting (this is tertiary prevention)
Secondary prevention generally consists of the identification and interdiction of diseases that
are present in the body, but that have not progressed to the point of causing signs, symptoms,
and dysfunction

391) 22 yrs old c/o insomnia/ sleep disturb treatment


a. SSRI
Short history , But the initial management of insomnia : good sleep hygiene .
……………………………………………………………………………………………………………………………………………
392) cellulitis in children most common causes
a. group A streptococcus
In children, the most common cause of cellulitis is S aureus or group A streptococcus.
………………………………………………………………………………………………………………………………………………
393) antidepressants associated with hypertensive crisis treatment
a. SSRI
b. MOAIs
c. TCAs
the correct answer is B
………………………………………………………………………………………………………………………………………………

394) pt his MBI = 24 kg he is


a. normal weight
b. over weight
c. morbid weight
d. mild weight
The correct answer is A
Normal BMI 18.5 to 24.9
Overweight: BMI is 25 to 29.9.
Obese: BMI is 30 or more

………………………………………………………………………………………………………………………………………………
395) baby c/o fever , chills , rigors and head rigidity +ve kerning's sign rash on his lower limb
diagnosis : - meningococcal meningitis ???????

396) 48 yrs pt with abdomina pain , nausea, vomiting tenderness in right hypochondrium -
acute cholecyctitis

397) 29 pt c/o dysuria his microscopic showed G -ve organism is


a. legionella b. E. coli
the correct answer is B
398) 30 yrs pt c/o feeling heaviness in the lower abdomen having bulge palpable at the top
scrotum that was reducible and increasing in Valsalva maneuver diagnosis
a. hydrocele
b. variocele
c. indirect inguinal hernia
d. direct inginal hernia
The correct answer is C

399) anticoagulation prescribed for


a. one month
b. 6 months
c. 6 weeks
d. one year
The correct answer is B
400) Cushing syndrome best single test to confirm
a. plasma cortisone
b. ATCH
c. Dexamethasone Suppression Test
The correct answer is C
………………………………………………………………………………………………………………………………………………
401) 23 yrs old history of URTI then he developed ecchymosis best treated
a. local AB
b. local antiviral
c. steroid
The correct answer is C

402) chronic psychotic disorder managed by a. haloperidol


……………………………………………………………………………………………………………………………………………
403) 29 yrs old lady B-HSG 160 c/o vomiting, abdominal pain which is more accurate to
diagnosis
a. BHCG serial
b. pelvic US
c. laparoscopy
Correct answer B.
It may be Molar pregnancy. USG more accurate to diagnose complete or incomplete hydatiform
mole
405) 70 yrs old man c/o fever , vesicular rash over forehead management
a. IV AB
b. IV antiviral
c. Acyclovir
the correct answer is C. possible case of herpes zoster
406) celiac disease involves :
a. proximal part of small intestine
b. distal part of small intestine
c. proximal part of large intestine
d. distal part of large intestine
The correct answer A
407) 6 yr old pt cyanosis past history of similar attack 6 month ago u will do for him
a. CxR
b. PFT
c. secure airway
d. CBC
choose B.

407) side effect of diazepam Sedation, dependence, respiratory suppression , antegrade


amnesia , confusion (especially pronounced in higher doses) and sedation
………………………………………………………………………………………………………………………………………………
408) endemic means:
Endemic :is the constant presence of a disease or infectious agent in a certain geographic area
or population group. ( usually rate of disease )
Epidemic : is the rapid spread of a disease in a specific area or among a certain population
group. ( excessive rate of disease )
Pandemic : is a worldwide epidemic; an epidemic occurring over a wide geographic area and
affecting a large number of people.
……………………………………………………………………………………………………………………………………………
409) Epidemic curve :
a graph in which the number of new cases of a disease is plotted against an interval of time to
describe a specific epidemic or outbreak.

410) Pt with Hodgkin's lymphoma , and red strunberg cell in pathology and there is eosinophil
lymphocyte in blood so pathological classification is:
a. Mixed-cellularity subtype
b. nodular sclerosis subtype of Hodgkin's lymphoma
the correct answer is B
Classical Hodgkin's lymphoma can be sub classified into 4 pathologic subtypes based upon
Reed-Sternberg cell morphology and the composition of the reactive cell infiltrate seen in the
lymph node biopsy specimen
.
1. Nodular sclerosing CHL Is the most common subtype and is composed of large tumor nodules
showing scattered lacunar classical RS cells set in a background of reactive lymphocytes,
eosinophils and plasma cells with varying degrees of collagen fibrosis/sclerosis
2.Mixed-cellularity subtype Is a common subtype and is composed of numerous classic RS cells
admixed with numerous inflammatory cells including lymphocytes, histiocytic, eosinophils, and
plasma cells. without sclerosis. This type is most often associated with EBV infection and may
be confused with the early, so-called 'cellular' phase of nodular sclerosing CHL
3. Lymphocyte-rich or Lymphocytic predominance Is a rare subtype, show many features which
may cause diagnostic confusion with nodular lymphocyte predominant B-cell Non-Hodgkin's
Lymphoma (B-NHL). This form also has the most favorable prognosis
4 Lymphocyte depleted Is a rare subtype, composed of large numbers of often pleomorphic RS
cells with only few reactive lymphocytes which may easily be confused with diffuse large cell
lymphoma. Many cases previously classified within this category would now be reclassified
under anaplastic large cell lymphoma

411) 62 y male with DVT and IVC obstruction due to thrombosis so most like dd is
a. nephrotic syndrome b. SLE C. Christmas disease
Possible answer here is A.
Nephrotic syndrome causes recurrent thrombosis
……………………………………………………………………………………………………………………………………………
412) Pt with abdominal pain heamaturia , HTN, and have abnormality in chromosome 16 ,
diagnosis is
a. POLY CYSTIC KIDNEY
There are three genetic mutations in the PKD-1, PKD-2, and PKD3 gene with similar
phenotypical presentations. Gene PKD1 is located on chromosome 16 and codes for a protein
involved in regulation of cell cycle and intracellular calcium transport in epithelial cells, and is
responsible for 85% of the cases of ADPKD
.…………………………………………………………………………………………………………………………………
413) 17 year pt with dyspnea Po2 , PCO2 ,X-ray normal PH increase so dd is:
a. acute attack of asthma
b. P E
c. pneumonia
d. pneumothorax
the correct answer is A
………………………………………………………………………………………………………………………………………………
414) A long scenario about patient with polydipsia ad polyuria. Serum osmolality high .
desmopressin induction no change urine osmolality and plasma osmolality so dd is
a. nephrogenic type b. central type
The correct answer is A
The water deprivation test (ie, the Miller-Moses test), a semi quantitative test to ensure
adequate dehydration and maximal stimulation of ADH for diagnosis, is typically performed in
patients with more chronic forms of DI
In central and nephrogenic DI, urinary osmolality will be less than 300 mOsm/kg after water
deprivation. After the administration of ADH, the osmolality will rise to more than 750
mOsm/kg in central DI but will not rise at all in nephrogenic DI
……………………………………………………………………………………………………………………………………………

415) 50 year old Man presented to ER with sudden headache, blurred of vision and eye pain.
The diagnosis is:
a. Acute glaucoma
b. Acute conjunctivitis
c. Corneal ulcer
The correct answer is A
416) RTA with hip dislocation and shock so causes of shock is
a. blood loss
b. urethral injury
c. neurogenic
The correct answer is A
High-energy blunt force trauma is the most common cause of hip dislocation
Posterior dislocations comprise approximately 80-90% of hip dislocations caused by RTAs. The
femoral head is situated posterior to the acetabulum.
………………………………………………………………………………………………………………………………………………

417) most common causes of hand infection


a. trauma b. immunocompromised c. ………
the correct answer is A
………………………………………………………………………………………………………………………………………………

418) ttt of cholesteatoma is


a. antibiotic b. steroid c. surgery d. Grommet tube
the correct answer is C
Cholesteatoma is a destructive and expanding growth consisting of keratinizing squamous
epithelium in the middle ear and/or mastoid . treatment : Surgery is performed to remove the
sac of squamous debris and a Mastidectomy is performed.

419) twins one male and other female . his father notice that female become puberty before
male so what you say to father
a. female enter puberty 1-2 year before male
b. female enter puberty 2-3 year before male
c. female enter puberty at the same age male
the correct answer is B
………………………………………………………………………………………………………………………………………………
420) in developing country to prevent dental caries , it add to water
a. fluoride
b. zinc
c. copper
d. iodide
the correct answer is A
………………………………………………………………………………………………………………………………………………
421) 12 y.o boy c/o abdominal pain after playing football, he denied any h/o trauma ,the pain is
in the Lt periumbilical region what inx you want to do:
a. CXR b. ultrasound kidney
The correct answer is B ,
………………………………………………………………………………………………………………………………………………
422) pt child with back pain that wake pt from sleep So diagnosis
a. lumber kyphosis
b. osteoarthritis c. RA
d. Scoliosis
the correct answer is D
-Musculoskeletal strain is most often responsible for back pain in children and adolescents.
-In adolescents, increased roundness of the back (when viewed from the side) — also called
Scheuermann's kyphosis — is a common cause of pain in the middle of the back. It is the second
most common cause of back pain in children and young adults
-Spondylolysis, or stress fracture, may cause lower back pain in adolescents
-A slipped vertebra, or spondylolisthesis
-In young children, infection in a disk space (diskitis) can lead to back pain
A child who is awakened at night by pain may have an infection, arthritis, or tumor of the spine.

423) child with papule vesical on oropharynx and rash in palm and hand so dd:
a. CMV
b. EBV
c. MEASLES
d. ROBELLA
???? Coxsackie A virus. Vesical in buccal mucosa is measles.
Hand-foot-and-mouth disease is a vesicular eruption in the oropharynx, palms, soles, and
interdigits of toddlers and school-aged children. The oral vesicles are not usually painful.
The most common causative agent is coxsackie virus group A, serotype 16, but strains of
enterovirus 71 also can cause
………………………………………………………………………………………………………………………………………………
424) child with dental caries and history of bottle feeding So dd
a. nurse milk caries
According to the American Dental Association, "As soon as a baby's first teeth appear—usually
by age six months or so—the child is susceptible to decay. This condition is often referred to as
Baby Bottle Tooth Decay or Early Childhood Caries
…………………………………………………………………………………………………………………………………………..
425) psychotherapy, medication, and electroconvulsive therapy Psychiatric pt with
un compliance of drugs ttt:
a. depo haloperidol injection b. oral clonazepam
the correct answer is A

426) mild diarrhea management :


a. oral antibiotis
b. IVF
c. ORS
the correct answer is C
…………………………………………………………………………………………………………………………………………..
427) salpingitis and PID on penicillin but not improve the most likely organism is :
a. chlamydia
b. Neisseria
c. SYPHLIS
d. HSV
the correct answer B
………………………………………………………………………………………………………………………………………………
428) Used for treatment of pseudomembranous colitis:
The decision to treat C difficile infection (CDI) and the type of therapy administered depend on
the severity of infection, as well as the local epidemiology and type of C difficile strains present.
No treatment is necessary for asymptomatic carriers.
For mild/moderate disease, oral metronidazole (500 mg 3 times daily for 10 days) is
recommended as initial treatment
In patients for whom oral treatment is inappropriate, fidaxomicin may be used; specific
indications include first-line treatment in patients with recurrence or at risk for recurrence
For patients with severe CDI, suitable antibiotic regimens include vancomycin (125 mg 4 times
daily for 10 days; may be increased to 500 mg 4 times daily) or fidaxomicin (200 mg twice daily
for 10 days)
………………………………………………………………………………………………………………………………………………
429) Lady with of right hypochondrial pain, fever and slight jaundice. What is your diagnosis?
a. Acute cholecyctitis.
Q not complete.
Most common presenting symptom of acute cholecyctitis is upper abdominal pain often
radiating to right scapula. Nausea and vomiting generally present. Patients may report fever.

430) Deep jaundice with palpable gallbladder ?


a. Cancer head of pancreas.
Q not complete
Pancreatic cancer presents with abdominal pain radiating toward the back, as well as with
jaundice, loss of appetite, nausea, vomiting, weight loss, weakness, fatigue, and indigestion.
Examination: may reveal a palpable, nontender gallbladder (Courvoisier’s sign) or migratory
thrombophlebitis (Trousseau’s sign). Diagnose by CT. treatment usually palliative (due to
metastasis at Dx).
……………………………………………………………………………………………………………………………………………
431) Patient with perforated gallbladder underwent cholecystectomy. Return to you with fever.
On abdominal X-ray, there is elevation of right hemidaiaphragm. What is the possible
diagnosis?
a. Sub phrenic abscess
………………………………………………………………………………………………………………………………………………
432) Osteoarthritis distal interphalangeal nodules
a. Heberden's nodes
Heberden's nodes are at the DIP while Bouchard's nodes are at PIP.
433) Elderly patient with RLQ fullness, weight loss, changed bowel habit, anemic and pale.
What is the investigation of choice?
a. Colonoscopy.
This is now the investigation of choice if colorectal cancer is suspected provided the
patient is fit enough to undergo the bowel preparation. It has the advantage of not only picking
up a primary cancer but also having the ability to detect synchronous polyps or even multiple
carcinomas, which occur in 5% of cases.
………………………………………………………………………………………………………………………………………………

434) Investigation of choice in Iron deficiency anemia?


a. serum iron b. ferritin c. TIBC
the correct answer is B
Serum ferritin is the most sensitive lab test for iron deficiency anemia.
………………………………………………………………………………………………………………………………………………
435) In patients with hypertension and diabetes, which antihypertensive agent you want to
add first?
a. β-blockers
b. ACE inhibitor
c. α-blocker
d. Calcium channel blocker
the correct answer is B Plz check Q464 in Pearson 1000 file

436) Patient presented with retinal artery occlusion, which of the following is wrong:
a. painful loss of vision
b. painless loss of vision
the correct answer is A
The most common presenting complaint of retinal artery occlusion is an acute persistent
painless loss of vision.

437) ECG finding of acute pericarditis?


a. ST segment elevation in all leads
Classic ECG findings in pericarditis: Low-voltage, diffuse ST-segment elevation.
438) Epidemiological study want to see the effect of smoking in the bronchogenic carcinoma,
they saw that is 90% of smokers has bronchogenic carcinoma. 30% of non–smokers has the
disease, the specificity of the disease as a risk factor is:
a. 70 %
b. 30 %
c. 90 % I did not get answer for this

439) Baby present with pain in the ear, by examination there is piece of a glass deep in his ear
canal, the mother mention a history of a broken glass in the kitchen but she clean that
completely. We treat that by:
a. By applying a stream of solution to syringing the ear.
b. Remove it by forceps.
c. Refer her to otolaryngology
I will choose C as a GP.
* Foreign bodies in the ear canal: Techniques appropriate for the removal of ear foreign
bodies include mechanical extraction, irrigation, and suction.
Irrigation is contraindicated for organic matter that may swell and enlarge within the auditory
canal (e.g seeds). Insects, organic matter, and objects with the potential to become friable and
break into smaller evasive pieces are often better extracted with suction than with forceps. Live
insects in the ear canal should be immobilized before removal is attempted.
………………………………………………………………………………………………………………………………………………
440) Treatment of open tibial fracture:
a. cephazolin
b. cephazolin + gentamycin
c. gentamicin
d. cephazolin + gentamicin + metronidazole
treatment of open fracture D
1- First-generation cephalosporin (Gram-positive coverage) such as cephalexin (1-2 g q6-8h)
suffice for Gustilo type I open fractures.
2- An aminoglycoside (Gram-negative coverage) such as gentamycin (120 mg q12h; 240 mg/d)
is added for types II and III injuries.
3- Additionally, metronidazole (500 mg q12h) or penicillin (1.2 g q6h) can be added for coverage
against anaerobes.
4- Tetanus prophylaxis should be instituted.
5- Antibiotics generally are continued for 72 hours following wound closure.

441) Which of the following found to reduce the risk of post herpetic neuralgia:
a) corticosteroids only
b) corticosteroids + valacyclovir
c) valcyclovir only
The correct answer is C
Patients with herpes zoster should receive treatment to control acute symptoms and
prevent complications. Patients over the age of 50, irrespective of other risk factors, are at
much greater risk of developing postherpetic neuralgia and should be offered treatment. By
inhibiting replication of varicella zoster virus, the antiviral agents acyclovir, famcyclovir, and
valacyclovir attenuate the severity of zoster— specifically, the duration of viral shedding is
decreased, rash healing is hastened, and the severity and duration of acute pain are reduced.
Attenuation of the severity of the acute infection and the neural damage it causes should
reduce the likelihood of post herpetic neuralgia.
……………………………………………………………………………………………………………………………………………

442) 8 months old infant with on & off recurrent crying episodes & hx of current jelly stools:
a) intussception b) intestinal obstruction c) mickel's diverticulitis d) strangulated hernia
The correct answer is A
443) a man with oblong swelling on top of scrotum increase in size with Valsalva maneuver
most likely Dx:
a) direct inguinal hernia
b) indirect inguinal hernia
c) varicocele
d) femoral hernia
the correct answer is B
* Indirect hernia: Herniation of abdominal contents through the internal and then external
inguinal rings and eventually into the scrotum (in males). - The most common hernia in both
genders. - Due to a congenital patent processus vaginalis. - Indirect inguinal hernia increases in
size it becomes apparent when the patient coughs, and persists until reduced.
* Direct hernia: Herniation of abdominal contents through the floor of Hesselbach’s Triangle.
(Hesselbach’s triangle is an area bounded by the inguinal ligament, inferior epigastricartery, and
rectus abdominis) - Hernial sac contents do not traverse the internal inguinal ring; they herniate
directly through the abdominal wall and are contained within the aponeurosis of the external
oblique muscle. - Most often due to an acquired defect in the transversalis fascia from
mechanical breakdown that ↑ with age.
444) 3 weeks old male newborn with swelling of scrotum transparent to light & irreducible:
a) epididymitis b) hydrocele
The correct answer is B
………………………………………………………………………………………………………………………………………………
445) Which drug causes SLE like syndrome:
a) hydralazine b) propranolol c) amoxicillin
The correct answer is A
Drug-induced SLE causes: Chlorpromazine, Hydralazine, Isoniazid, Methyldopa, Penicillamine,
Procainamide, Quinidine, Sulfasalazine.

446) When to give aspirin and clopidogrel?


a) pt with a hx of previous MI
b) Acute MI
c) hx of previous ischemic stroke
d) hx of peripheral artery disease
e) after cardiac cath The correct answer is B
447) a young girl experienced Crampy abdominal pain & proximal muscular weakness but
normal reflexes after receiving septra (trimethoprim sulfamethoxazole) :
a) functional myositis
b) polymyositis
c) guillian barre syndrome
d) neuritis
e) Porphyria
the correct answer is E
……………………………………………………………………………………………………………………………………………
448) 17 years with history of right iliac fossa pain rebound tenderness +ve guarding what is the
investigation that you will do?
a. Laparoscopy
b. US
c. CT scan
the correct answer is C
In appendicitis, CT scan with contrast has 95–98% sensitivity.
449) Child with history of URTI before 3 weeks now has both knees tender, red and inflamed.
What is your diagnosis?
a. Rheumatoid arthritis
Q is not complete

The diagnosis of JRA is established by the presence of arthritis, the duration of the disease for
at least 6 weeks, and the exclusion of other possible diagnoses. Although a presumptive
diagnosis of systemic-onset JRA can be established for a child during the systemic phase, a
definitive diagnosis is not possible until arthritis develops. Children must be younger than 16
years old at time of onset of disease; the diagnosis of JRA does not change when the child
becomes an adult. Because there are so many other causes of arthritis, these disorders need to
be excluded before providing a definitive diagnosis of JRA (Table 89-2). The acute arthritides
can affect the same joints as JRA, but have a shorter time course. In particular, JRA can be
confused with the spondyloarthropathies, which are associated with spinal involvement, and
enthesitis, which is inflammation of tendinous insertions. All of the pediatric
spondyloarthropathies can present with peripheral arthritis before other manifestations and
initially may be diagnosed as JRA (Table 89-3).
* Juvenile rheumatoid arthritis: mono- and polyarthropathy with bony destruction that
occurs in patients ≤ 16 years of age and lasts > 6 weeks. Approximately 95% of cases resolve by
puberty. Subtypes: Pauciarticular: ≤ 5 joints involved, associated with an risk of iridocyclitis
that lead to blindness if left untreated. Polyarticular: ≥ 5 small joints. Systemic features are less
prominent; carries a risk of iridocyclitis. Acute febrile: The least common subtype;
manifests as arthritis withdaily high, spiking fevers and an evanescent, salmon-colored rash.
Hepatosplenomegaly and serositis may also be seen. No iridocyclitis is present; remission
occurs within one year.

450) Old female came complain from pain in her joint increase with walking, what is your
diagnosis?
a. Osteoarthritis
Physical signs of OA: Crepitus, range of motion, pain that worsens with activity and weight
bearing but improves with rest.

451) In ulcerative colitis, what you will start treatment with?


a. Corticosteroids ??
- Sulfasalazine or 5-ASA (mesalamine) initial treatment
- Corticosteroids and immunosuppressant indicated if no refractory disease
452) Patient came with history of neck discomfort, palpitation, cold sweat, TSH low, T4
high, tender neck. What is your diagnosis?
a. Subacute thyroiditis.
- Thyroiditis: Inflammation of the thyroid gland. Common types are subacute granulomatous,
radiation, lymphocytic, postpartum, and drug-induced (e.g., amiodarone) thyroiditis. - Hx/PE: In
subacute and radiation, presents with tender thyroid, malaise, and URI symptoms. - Dx: Thyroid
dysfunction (typically hyperthyroidism followed by hypothyroidism), all with ↓ uptake on RAIU.
- Rx: - β-blockers for hyperthyroidism; levothyroxine for hypothyroidism. - Subacute thyroiditis:
Anti-inflammatory medication.
…………………………………………………………………………………………………………………………………………
453) Treatment of prostatitis?
a. Ciprofloxacin
treatment is for 4–6 weeks with drugs that penetrate into the prostate, such as
trimethoprim or ciprofloxacin.
……………………………………………………………………………………………………………………………………………
454) In IV cannula and fluid:
a. Site of entry of cannula is a common site of infection.

455) Patient with neck rigidity, rigor, fever, petechial rash over extremities.
The causative bacteria is:
a. Meningococcal meningitis
A reddish or purple skin rash is a very important sign to watch for. If it does not turn
white when you press a glass against it, the rash may be a sign of sepsis. This is a medical
emergency.

456) Female patient works in office came with vulva itching, yellow vaginal discharge:
a. Trichomoniasis

457) old pt c/o bilateral knee pain with mild joint enlargement ESR and CRP normal dx:
a. Osteoarthritis
b. Rheumatoid arthritis
c. Gout?
The correct answer is A
458) Sciatica increase the incidence of :
a. Lumbar lordosis b. Paresthesia
Q not complete but with these MCQs , correct answer is B

456) old male c/o knee pain on walking with crepitus xray show narrow joint space and
subchondral sclerosis:
a. Rheumatoid arthritis
b. Osteoarthritis
c. Gout
The correct answer is B
………………………………………………………………………………………………………………………………………………
457) 14y f with BMI 32.6 :
a. Overweight
b. Obese
c. Normal weight
The correct answer is B
458) vaginal discharge odorless watery microscopy show clue cells :
a. Bacterial vaginosis b. Candidiasis
The correct answer is A
……………………………………………………………………………………………………………………………………………
459) Pregnant women on 3rd trimester infected with measles can I give her MMR?????
No MMR vaccine during pregancy
………………………………………………………………………………………………………………………………………………
460) lactating women infected with rubella ……..management is :
a. MMR
b. Stop lactation
The correct answer is a MMR is safe during lactating
……………………………………………………………………………………………………………………………………………
461 ) which of following not live vaccine:
a. BCG
b. Hepatitis
B c. Oral polio
d. MMR
The correct answer is B
………………………………………………………………………………………………………………………………………………
462) the most common cause of community acquired pneumonia:
a. Hemophilus influenza
b. Strepto. pneumonia
c. Mycoplasma
d. Kliebsella
The correct answer is B
463) female after vaginal hysterectomy she complain of urine coming from vagina………dx:
a. Vesicovaginal fistula
b. Urethrovaginal fistula
c. Ureterovaginal fistula
The correct answer is A
……………………………………………………………………………………………………………………………………………
464) child rt ear pain and tenderness on pulling ear , no fever , O/E inflamed edematous rt ear
canal with yellow discharge >>>>>>>>>>
dx: a. Otitis media
b. Otitis externa
c. Cholesteatoma The correct answer is B
Tender pinna is point towards otitis externa
………………………………………………………………………………………………………………………………………………
465) male with perianal pain ,tenderness ,fluctuant (perianal abscess)
a. Incision and drainage
b. Warm bath
The correct answer is A
466) Pregnant 34 wk with vaginal bleeding………. what would you ask about :
a. Cigarette smoking
b. Recent sexual intercourse
Q not complete , but with theses MCQs may be correct answer is A
………………………………………………………………………………………………………………………………………………
467) In duodenal obstruction of newborn what is the sign that appear in xray:
a. Double bubble
468) the following more common with type2 DM than type1 DM:
a. Weight loss
b. Gradual onset
c. Hereditary factors
d. HLA DR3+-DR4
The correct answer is C
469) pt with open angle glaucoma and k/c of COPD and DM ttt:
a. Timolol
b. betaxolol
c. Acetazolamide
The correct answer is C
………………………………………………………………………………………………………………………………
470) 17 year boy admit to involve in recurrent illegal drug injection , what the screening test to
do:
a. HIV b. Hepatitis B c. Hepatitis C
-All of the above choices are correct
471) Rt upper quadrant pain and tenderness , fever, high WBC, jaundice, normal hepatic marker
a. Acute cholecyctitis
b. Pancreatitis
c. Acute hepatitis
-The correct answer is A.
………………………………………………………………………………………………………………
472) Man with sudden onset of scrotal pain , also had Hx of vomiting, on examination tender
scrotum and there is tender 4 cm mass over right groin, what you will do:
a. Consult surgeon
b. Consult urologist
c. Do sonogram
d. Elective surgery
-The correct answer is B , if suspected torsion of tests , don't waste time on radiological study
473) pt child with back pain that wake pt from sleep So diagnosis
a. lumber kyphosis
b. osteoarthritis
c. RA
d. Scoliosis
The correct answer is D
A child who is awakened at night by pain may have an infection, arthritis, or tumor of the spine
…………………………………………………………………………………………………………………………………………
474) 59 y/o presented with new onset supraventricular tachycardia with palpitation, no Hx of
SOB Or chest pain, chest examination normal , oxygen sat in room air = 98% no peripheral
edema Others normal, the best initial investigation:
a. ECG stress test
b. Pulmonary arteriography
c. CT scan
d. Thyroid stimulating hormone the correct answer is D
475 Lactating women presented with breast engorgement and tenderness Your
managements:
a. Warm compressor and continue breast feeding
b. Dicloxacillin and continue breast feeding
c. Dicloxacillin and milk expression
d. Discontinue breast feeding and cold compressor
the correct answer is A .

476) The Fastest route of antipsychotic is:


a. IM
b. IV
c. Oral
d. Sublingual
the correct answer is B
477) While you are in the clinic you find that many patients presents with red follicular
conjunctivitis (Chlamydia )
your management is:
a. Improve water supply and sanitation
b. Improve sanitation and destroying of the vector
c. Eradication of the reservoir and destroying the vector
d. Destroy the vector and improve the sanitation
The correct answer is A
478) The most important exogenous risk factor for osteoporosis is:
a. Alcohol intake
b. Age
c. Smoking
d. Lack of exercise the correct answer is C
479) Patient with family history of coronary artery disease his BMI= 28 came to you asking for
the advice:
a. Start 800 calorie intake daily
b. Decrease carbohydrate daytime
c. Increase fat and decrease protein
d. Start with decrease ……. K calorie per kg per week
the correct answer is D
In general, if you cut 500 calories from your typical diet each day, you'd lose about 0.45 kg a
week
(500 calories x 7 days = 3,500 calories).
Diets that have a 600 kcal/day deficit (that is, they contain 600 kcal less than the person needs
to stay the same weight) or that reduce calories by lowering the fat content (low-fat diets), in
combination with expert support and intensive follow-up, are recommended for sustainable
weight loss. [2006]
Consider low-calorie diets (800–1600 kcal/day), but be aware these are less likely to be
nutritionally complete. [2006, amended 2014]
Do not routinely use very-low-calorie diets (800 kcal/day or less) to manage obesity (defined as
BMI over 30). [new 2014]

480) Lactating mother newly diagnosed with epilepsy , taking for it phenobarbital you advice is:
a. Discontinue breastfeeding immediately
b. Breastfeed baby after 8 hours of the medication
c . Continue breastfeeding as tolerated
the correct answer is C
If mothers receiving ethosuximide, phenobarbital or primidone choose to breastfeed, they
should exercise caution and closely monitor the infant for sedation, lethargy and any significant
clinical findings.
Phenytoin, carbamazepine and valproate are probably safe during laction. These AEDs are all
moderately to highly protein-bound, and are not transferred in high concentrations in breast
milk.
481) Child with moderate persistent BA On bronchodilator inhaler. Presented with acute
exacerbation what will you add in ttt:
a. Corticosteroid inhaler
b. Ipratropium bromide inhaler
the correct answer is A

482) Pregnant women has fibroid.. which of the following is True:


a. Presented with severe anemia
b. Likely to regress after Pregnancy
c. Surgery immediately
d. Presented with Antepartum He
the correct answer is B
Fibroids may also be the result of hormones. Reproductive hormones like estrogen and
progesterone can stimulate cell growth, causing fibroids to form. During pregnancy, your influx
of hormones may cause your fibroids to grow in size. After pregnancy and during menopause
most fibroids begin to shrink, due to a lack of hormones.

483) A known case of chronic atrial fibrillation on the warfarin 5 mg came for follow up you find
INR 7 but no signs of bleeding you advice is:
a. Decrease dose to 2.5 mg
b. Stop the dose and repeat INR next day
c. Stop warfarin
d. Continue same and repeat INR
the correct answer is B
INR ACTION
>10 Stop warfarin. Contact patient for examination. MONITOR INR
7-10 Stop warfarin for 2 days; decrease weekly dosage by 25% or by 1 mg/d for next week (7
mg total); monitor INR
4.5-7 Decrease weekly dosage by 15% or by 1 mg/d for 5 days of next week (5 mg total); repeat
monitor INR
3-4.5 Decrease weekly dosage by 10% or by 1 mg/d for 3 days of next week (3 mg total); repeat
monitor INR.
2-3 No change.
1.5-2 Increase weekly dosage by 10% or by 1 mg/d for 3 days of next week (3 mg total);
<1.5 Increase weekly dose by 15% or by 1 mg/d for 5 days of next week (5 mg total);

484) Patient is a known case of CAD the best exercise:


a. Isotonic exercise
b. Isometric exercise
c. Anerobic exe
d. Yogha
the correct answer is A
Isotonic and aerobic exercises are good for patients with CAD

485) The mechanism of action of Aspirin:


a. Inhibit cyclooxygenase b. Inhibit phospholipase A2 c. Inhibit phospholipid D
the correct answer is A
486) The absolute contraindication of breastfeeding is :
a. Asymptomatic HIV patient
b. Active hepatitis C
c. Pulmonary TB on treatment 3 months
The correct answer is A

Absolute contraindication of breastfeeding :


1-Infants with galactosemia.
2-Mothers who use illegal drugs.
3-Mothers infected with HIV, human T-cell lymphotropic virus type I or type II, or who have an
active herpes lesion on the breast.
4-Mothers taking any of the following medications: radioactive isotopes, cancer chemotherapy
agents, such as antimetabolites

487) A boy fell down on his elbow , the lateral x-ray shows:
a. Anterior Pad sign
b. Posterior pad sign
c. Anterior line of humerus intersecting the cubilium
d. Radial line forming 90 degree with cubilium
The correct answer is B

488) A known case of treated Hodgkin lymphoma(mediastinal mass) with radiotherapy. Not on
regular follow up presented with gradual painless difficulty in swallowing and SOB , There is
facial swelling and redness : DX
a. SVC obstruction
b. IVC obstruction
c. Thoracic aortic aneurysm
d. Abdominal aortic aneurysm
The correct answer is A
________________________________________________________________________
489) Patient is presented with hand cellulitis and red streaks in the hand and tender axillary
lymphadenopathy. This condition is more likely to be associated with:
a. Malignancy b. Pyoderma
c. Neuropathy d. Lymphangitis
The correct answer is B
490) Young aged male presented to ER after blunt trauma to Abdomen, CT scan shows
intramural hematoma: your management is
a. Laparotomy with evacuation of the hematoma
b. Dissection of duodenum
c. Observation
the correct answer is C

491) Patient presented with sore throat, anorexia, loss of appetite, on throat exam showed
enlarged tonsils with petechiae on palate and uvula , mild tenderness of spleen and liver :DX
a. Group A strep
b. EBV
The correct answer is B

492) Patient with GERD has Barret's esophagus , this metaplasia increase risk of :
a. Adenocarcinoma
b. Squamous cell carcinoma
the correct answer is A

493) Complication of Sleep apnea is :


a.CHF
b. ……
the correct answer is a
sleep apnea : Hypoxic pulmonary vasoconstriction, PAH Cor Pulmonale CHF
complication of sleep apnea : sleep apnea increases health risks such as cardiovascular

494) Which of the following medication can be used as prophylaxis in appendectomy:


a. Cephalexin
b. Ceftriaxone
c. Metronidazole
d. Vancomycin
e. Ampicillin
The correct answer is B
1st line of antibiotic : 1- cefoxitin 2- cefotetan
2nd line of treatment : 1- metronidazole 2- ampicikkin-sulbactam
494) Which of the following prognostic factor for SLE:
a. ANA levels
b. Sex
c. Age
d. Renal involvement
The correct answer is D

495) The most common site for osteomyelitis is:


a. Epiphysis
b. Diaphysis
c. Metaphysis
d. Blood flow
The correct answer is C
496) In “holding breath holding” which of the following True:
a. Mostly occurs between age of 5 and 10
b. Increase Risk of epilepsy
c. A known precipitant cause of generalized convulsion
d. Diazepam may decrease the attack
The correct answer is C
Breath holding spells are the occurrence of episodic apnea in children, possibly associated with
loss of consciousness, and changes in postural tone. They are most common in children
between 6 and 18 months and usually not present after 5 years of age. They are unusual before
6 months of age. A positive family history can be elicited in 25% of cases. It may be confused
with a seizure disorder.
There are four types of breath holding spells.
1-The most common is termed simple breath holding spell, in which the manifestation is the
holding of breath in end expiration. There is no major alteration of circulation or oxygenation
and the recovery is spontaneous.
2-The second type are the Cyanotic breath-holding spells. They are usually precipitated by
anger or frustration although they may occur after a painful experience. The child cries and has
forced expiration sometimes leading to cyanosis (blue in color), loss of muscle tone, and loss of
consciousness. The majority of children will regain consciousness. The child usually recovers
within a minute or two, but some fall asleep for an hour or so. Physiologically, there is often
hypocapnea (low levels of carbon dioxide) and usually hypoxia (low levels of oxygen. There is no
"post ictal" phase (as is seen with seizures), no incontinence, and the child is fine in between
spells. EEGs are normal in these children. There is no relationship to the subsequent
development of seizures or cerebral injury as a consequence of breath holding spells.
3-In the third type, known as Pallid breath-holding spells, the most common stimulus is a
painful event. The child turns pale (as opposed to blue) and loses consciousness with little if any
crying. The EEG is also normal, and again there is no post ictal phase, nor incontinence. The
child is usually alert within a minute or so. There may be some relationship with adulthood
syncope in children with this type of spell.
4- A fourth type, known as Complicated breath-holding spells, may simply be a more severe
form of the two most common types. This type generally begins as either a cyanotic or pallid
spell that then is associated with seizure like activity. An EEG taken while the child is not having
a spell is still generally normal.
DX: clinical , good history include sequence of event , lack of incontinence and no post ictal
phase. Treatment : reassurance and iron.

497) Infant brought by the mother that noticed that the baby has decreasing feeding , activity
and lethargic On examination febrile(39), tachycardia ,his bp 75/30, with skin rash . DX:
a. Septic shock
Q not complete
Fever, hypothermia (very low body temperature), or seizures.
Eating, drinking, sucking poorly, or vomiting.
Fast or slow heart rate.
Fast breathing or trouble breathing.
Urinating very little or not at all.
Weakness, irritability, drowsiness, and harder to wake than normal.

498) Old patient newly diagnosed with hyperthyroidism presented with (hyperthyroidism
symptoms)
The best initial symptomatic treatment is:
a. BB ( beta-blocker )
b. PTU
The correct answer is B
Many of the neurologic and cardiovascular symptoms of thyrotoxicosis are relieved by beta-
blocker therapy. Before such therapy is initiated, the patient should be examined for signs and
symptoms of dehydration that often occur with hyperthyroidism. After oral rehydration, beta-
blocker therapy can be started. Beta-blocker therapy should not be administered to patients
with a significant history of asthma.
499) Infant presented with hemangioma on the back . your management is:
a. Intralesional injection of corticosteroids
b. Topical corticosteroids
c. Excise of the lesion (The correct option is Observation)
Choose A here
Capillary hemangiomas aren’t usually treated. As the child gets older, the growth tends to
shrink and will usually disappear on its own.
If a cavernous hemangioma is in an area that could interfere with sight or breathing (around the
eyes, nose, or throat), treatment may be necessary.
If the hemangioma is growing quickly, corticosteroids such as prednisone can slow or stop the
growth. They may be given orally, applied topically, or injected into the hemangioma.
Observation is appropriate for asymptomatic or mildly symptomatic hemangiomas
500) Pregnant lady , 34 wk GA , presented with vaginal bleeding more than her menstruation.
On examination , cervix is dilated 3 cm with bulging of the membrane, fetal heart rate = 170
bpm . The fetus lies transverse with back facing down . us done and shows that placenta is
attached to posterior fundus and sono translucence behind placenta. Your management is :
a. C/S
b. Oxytocin
c. Tocolytics
d. Amniotomy
the correct answer is A

501) Infant with congenital hip dislocation:


a. +ve click in flexion, abduction
b. The only treatment is surgery
c. Not reduced with flexion and abduction of the hip
the correct answer is A . This is ortolani test

502) In irritable bowel S. the following mechanism contraction and slow wave myoelectricity
seen in:
a. Constipation
b. Diarrhea
the correct answer is A

503) Which is not found in coarctation of the aorta:


a. Upper limb hypertension
b. Diastolic murmur heard all over precordium
c. Skeletal deformity on chest x-ray
the correct answer is B

504) Female patient presented with tender red swelling in the axilla with history of repeated
black head and large pore skin in same area: ttt is
a. Immediate surgery
b. Topical antibiotic
c. Cold compressor
d. Oral antibiotic
The correct answer here is A.
This is a case of Hidradenitis suppurativa. Recurrent disease and multiple lesions needs
surgery.

505) In indirect hernia the relation of the sac to the cord structure is:
a. Anteromedial
b. Anterolateral
c. Posteromedial
d. Posterolateral
the correct answer is B

506) The most common cause of croup is:


a. Parainfluenza
b. Influenza
the correct answer is A

507) kwashikor disease usually associated with :


a. decrease protein intake, decrease carbohydrate
b. increase protein , increase carbo
c. decrease protein , increase carbo

The correct answer is C.


Marasmus is protein energy malnutrition in which there is deficiency of both proteins and
energy. Child with marasmus will be emaciated. Whereas Kwashiorkor is protein deficiency
with adequate carbohydrate intake. Child with kwashiorkor will be edematous.
508) in cachectic patient, the body utilize the proteins of the muscles :
a. to provide Amino acid and protein synthesis
b. to maintain blood flow to vital organ
c. to increase body fat
the correct answer is A

509) patient is complaining of memory loss. Alzheimer disease is diagnosed what is the cause of
this:
a. death of brain cell
Alzheimer's Disease (AD) is a specific neurodegenerative disease and is the most common cause
of dementia in old people.
The first defined histopathologic features of AD were extracellular amyloid plaques and
intracellular neurofibrillary tangles

510) Parents brought their baby to you who is on bottle feeding. On exam whitish lesion on
either side of teeth seen with blackish lesion on maxillary incisors and second molar teeth.
There is history of leaving the baby with bottle in his mouth during sleeping. The Dx:
a. Nursery dental caries
b. Gingivostomatis
The correct answer is A

511) Which of the following medication if taken need to take the patient immediately to the
hospital:
a. Penicillin
b. diphenhydramine
c. OCPs
d. Quinine or Quinidine
the correct answer is D
512) 43 y/o female presented with severe DUB other examination normal . your management is
a. D &C
b. Ocps
c. Hysterectomy
d. Blood transfusion
the correct answer is A

513) Baby with vesicles on the face and honey comb crust which of the following organism
cause it:
Staph aureus
Impetigo is the most common bacterial infection in children. This acute, highly contagious
infection of the superficial layers of the epidermis is primarily caused by Streptococcus
pyogenes or Staphylococcus aureus.
Impetigo is caused by bacterial infection. Both S aureus and GABHS cause nonbullous impetigo;
S aureus accounts for approximately 80% of cases, GABHS accounts for 10% of cases, and both
organisms are recovered in 10% of cases. Bullous impetigo is caused almost exclusively by S
aureus.

514) Female patient presented with migraine headache which is pulsatile, unilateral , increase
with activity . Doesn't want to take medication. Which of the following is appropriate:?
a. Bio feedback
b. TCA
c. BB
the correct answer is A
Biofeedback has been shown to help some people with migraines. The two most common
types of biofeedback for migraines are thermal biofeedback and electromyographic
biofeedback.

515) Infant born with hemangioma on the rt eyelid what is appropriate time to operate to
prevent amblyopia:
a. 1 day
b. 1 week
c. 3 months
d. 9 months
The correct answer is B

516) Young patient on anti TB medication presented with vertigo which of the following drug
cause this:
a. Streptomycin
b. Ethambutol
c. Rifampcin
the correct answer is A , streptomycin cause 8th nerve damage.

517) The CPR for child is


a. 30 chest compression 2 ventilation … (Lone rescuer)
b. 15 chest c 2 ventilation … (2 or more rescures)
c. m15 chest compression 1 ventilation
the correct answer is A and B both r correct here.
518) Picture show large ulcer over medial side of the leg . what is your management
a. Shave biopsy
b. Elevate the legs and stocking
c. Topical steroids
the correct answer is D
…………………………………………………………………………………………………………………………………………………….
.
519) 2months infant with white plaque on tongue and greasy ,past h/o chlamydia conjunctivitis
after birth treated by clindamycin what is ttt:
a. Oral nystatin
b. Topical steroids
c. Topical acyclovir
d. Oral tetracycline the correct answer is A.
…………………………………………………………………………………………………………………………………………………….
.
520) child rt ear pain and tenderness on pulling ear , no fever , O/E inflamed edematous rt ear
canal with yellow discharge >>>>>>>>>>dx:
a. Otitis media
b. Otitis externa
c. Cholesteatoma
the correct answer is B
…………………………………………………………………………………………………………………………………………………….
.
521) 34y female with HIV pap smear negative, about cervical cancer screening :
a. After 3m if negative repeat after 6m
b. After 6 months and if negative annually
c. After 1y………………………………….annually
The correct answer is B

522) female about 30y with breast cancer (given cbc –chem. And reavel low hb and
hematocrite….) what is the next step in mangment:
a. Staging
b. Lumpectomy
c. Mastectomy
d. chemotherapy the correct answer is A
…………………………………………………………………………………………………………………………………………………….
.
523) about head and neck injury :
a. Hoarseness of voice and stridor can occur with mid facial injury
b. Tracheostomies contraindicated
c. Facial injury may cause upper air way injuries
Correct answer is C
…………………………………………………………………………………………………………………………………………………….
.
524) lactating women 10 days after delivery developed fever ,malaise, chills tender Lt breast
with hotness and small nodule in upper outer quadrant with axillary LN .Leucocytic count was
14 *10/L dx:
a. Inflammatory breast cancer
b. Breast abscess
c. Fibrocystic disease
The correct answer is B

525) 70y male with osteoporosis the T score of bone densometry would be :
a. -3.5 b. -2.5 c. 1 d. 2 e. 3.5
the correct answer is B
…………………………………………………………………………………………………………………………………………………….
526) child with epistaxis…….. management:
a. Compression on nose and leaning forward
the correct answer is A
…………………………………………………………………………………………………………………………………………………….
.
527) scenario about female underwent abdominal operation she went to physician For
check……….. U/S reveal metal thing inside abdomen (a.e missed during operation) What will you
do :
a. Call the surgeon and ask him what to do
b. Call attorney and ask about legal action
c. Tell her what you found
d. Tell her that is one of possible complications of operation
e. Don't tell her what you found
The correct answer is C
…………………………………………………………………………………………………………………………………………………….
.
528) male pt with scaly fine papular rash on front of scalp, nose and retro auricular……..(i think
tinea capitis) ttt is:
a. Ketoconazole cream…
b. Oral Augmentin
the correct answer is A
529) All can cause gastric ulcer except:
a- Tricyclic antidepressant.
b- Delay gastric emptying.
c- Sepsis.
d- Salicylates.
e- Gastric outlet incompetent.
The correct answer is A Tricyclic antidepressant use in treatment of peptic ulcer.
…………………………………………………………………………………………………………………………………………………….
.
530) 48year old female lost her menstruation for 2 cycles, the method of contraception is
condom, examination was normal except for dusky discoloration of the cervix. What u will do
next:
a.Progesterone challenge.
b.Beta HCG.
c.Pelvic u/s
The correct answer is B

531) 15 y/o female complaining of pain during menstruation, not sexually active, medical hx
unremarkable, physical examination normal, how to treat:
a- NSAID.
b- Danazole.
The correct answer is A
…………………………………………………………………………………………………………………………………………………….
.
532) What is true about alpha blocker:
a. Causes hypertension.
b. Worsen benign prostatic hyperplasia.
c. Cause tachycardia. d-
the correct answer is C alpha blocker :cause orthostatic hypotension and tachycardia.

533) cyclic menstruation that increase in frequency:


a. Polymenorrhea.
b. Hypermenorrhea.
c. Menorrhagia.
d. Dysmenorrhea. e. The correct answer is A
Polymenorrhea : frequent menestration ( < 21 day cycle )
Hypermenorrhea or menorrhagia : abnormally heavy and prolonged menstrual period at
regular intervals . more than 80 ml of blood loss per cycle or prolonged bleeding , more than 8
days.
Oligomenorrhea : increase length of time between menses ( 35-90 days b\w cycle)
Metrorrhagia : bleeding between period.
Menometrorrhagia : excessive and irregular bleeding.

…………………………………………………………………………………………………………………………………………………….
.
534) A pregnant lady came to you to screen her fetus for down syndrome, what is the best
method:
a. Amniocentesis. + Karyotyping
b. Choriocentesis. c. Triple screening
the correct answer is A
…………………………………………………………………………………………………………………………………………………….
.
535) What is the most common chromosomal abnormality?
a. Trisomy 13
b. Trisomy 21
the correct answer is B
-down syndrome (trisomy 21 ) is the most common chromosomal abnormality.
…………………………………………………………………………………………………………………………………………………….
.
536) Best 2 test to screen for hepatocellular carcinoma:
a. Liver biopsy and alpha-fetoprotein.
b. Liver ultrasound and alpha-fetoprotein.
c. Abdomen CT and d.
The correct answer is B
Ultrasonography as a screening method is reported to have 60% sensitivity and 97% specificity
in the cirrhotic population
When elevated, the AFP is 75-91% specific, and values greater than 400 ng/mL are generally
considered diagnostic of HCC in the proper clinical context, including appropriate radiologic
findings.
537) A child came to ER with fever, stridor, … , x-ray showed swollen epiglottis, in addition to
oxygen, what u will do?
a. Throat examination.
b. An emergency tracheostomy.
c. Endotracheal intubation.
d. Nasopharyngeal intubation.
The correct answer is C
Orotracheal intubation may be required with little warning. Equipment for intubation,
cricothyroidotomy, or needle-jet ventilation should be made available at the bedside.
538) 24 y/o female newly diagnosed type 2 DM, she is wearing glasses for 10 years, how
frequent she should follow with ophthalmologist:
a. Every 5 years.
b. Annually
The correct answer is B
-for type 1 diabetic : retina screening annually beginning 5 years after onset of diabetes,
general not before onset of puberty.
- for type 2 diabetic : screening at the time of diagnosis then annual
…………………………………………………………………………………………………………………………………………………….
539) What is the initial management for a patient newly diagnosed knee osteoarthritis.
a. Intra-articular corticosteroid.
b. Reduce weight.
c. Exercise.
d. Strengthening of quadriceps muscle.
The correct answer is B
…………………………………………………………………………………………………………………………………………………….
.
540) A lady came to your clinic said that she doesn’t want to do mammogram and preferred to
do breast self- examination, what is your response?
a- Mammogram will detect deep tumor.
b- Self-examination and mammogram are complementary.
c- Self-examination is best to detect early tumor
answer is B
…………………………………………………………………………………………………………………………………………………….
541) What is the best frequency for breast self-examination?
a. Daily.
b. Weakly.
c. Monthly.
d. Annually. The correct answer is C
542) Patient with left bundle branch block will go for dental procedure , regarding endocarditis
prophylaxis:
a. No need
b. Before procedure.
c. After the procedure. d. The correct answer is A
…………………………………………………………………………………………………………………………………………………….
.
543) classical characteristic for genital herpes. Painful ulcers & vesicles
…………………………………………………………………………………………………………………………………………………….
.
544) most common vaginal bleeding :
a. cervical polyps
b. menstruation c. ????
The correct answer is B
…………………………………………………………………………………………………………………………………………………….
.
545) best stimulant for breast milk secretion:
a. breast feeding
b. oxytocin c. ???
the correct answer is A
…………………………………………………………………………………………………………………………………………………….
.
546) child with aspirin intake overdose ...what kind of acid base balance:
a. metabolic alkalosis wt respiratory
b. metabolic acidosis wt respiratory alkalosis
c. respiratory alkalosis with metabolic acidosis
d. respiratory acidosis with metabolic alkalosis
The correct answer is C
Aspirin toxicity : in early stages, salicylate will stimulate respiratory center results in respiratory
alkalosis that will be compensated by metabolic acidosis.
TTT: hydration, correct K+, gastric lavage or activated charcoal, urine alkalization, hemodialysis)

547) therapeutic range of INR :


a. 2.5-3.5
b. 2.0-3.0 c. ???
The correct answer is B
…………………………………………………………………………………………………………………………………………………….
.
548) q about antidepressant:
a. start single type even patient have severe depression
b. start any one of them they all have the same efficacy
c. stop the medication after 2 weeks if no improvement
The correct answer is A
549) patient with depression started on amitriptyline, he had headache or dizziness ,
vomiting??
im not sure what exactly was the symptoms
a. change to SSRI b. ??
Q not complete , but amitriptyline is TCA .
…………………………………………………………………………………………………………………………………………………….
.
550) patient had hoarseness of voice for 3 weeks... next to do:
a. throat swab
b. laryngoscopy c. ???
The correct answer is B
Options may be missing for this question. The correct option could be CT scan of neck
…………………………………………………………………………………………………………………………………………………….
.
551) single diagnostic for stroke:
a. high cholesterol
b. high systolic blood pressure
the correct answer is B

552) unfavorable prognosis for schizophrenia:


a. family Hx
b. failed marrige
c. adolscent age?? not sure
d. presence of psychosis
the correct answer is A
…………………………………………………………………………………………………………………………………………………….
.
553) stage 3 colon ca
Give Chemo ASAP

Stage III colon cancers have spread to nearby lymph nodes, but they have not yet spread to
other parts of the body.
Surgery to remove the section of the colon with the cancer along with nearby lymph nodes
(partial colectomy) followed by adjuvant chemo is the standard treatment for this stage.
For chemo, either the FOLFOX (5-FU, leucovorin, and oxaliplatin) or CapeOx (capecitabine and
oxaliplatin) regimens are used most often
…………………………………………………………………………………………………………………………………………………….
554) patient with upper abdominal pain, nausea vomiting, with back pain, he is smoker for long
time daily, fecal fat was +ve
a. acute pancreatitis
b. chronic pancreatitis
c. pancreatic CA
The correct answer is B
555) patient had headache describe it as a band around his head, increase with stress and , i
dont remember the ques DX is:
a. tension headache
b. migraine
c. cluster headache
The correct answer is A
…………………………………………………………………………………………………………………………………………………….
.
556) Patient was presented by bullous in his foot , biopsy showed sub dermal lysis , fluorescent
stain showed IgG , what is the most likely diagnosis :
A. Bolus epidermolysis .
B. Pemphigoid vulgaris .
C. Herpetic multiform .
D. Bullous pemphigoid .
The correct answer is D
…………………………………………………………………………………………………………………………………………………….
.
557) Patient was presented by difficulties of breathing from one side of his nose , on
examination there was erythramatus swelling , what is the best initial treatment :
A. Decongestant.
B. Steroid.
C. Sympathomimetic.
The correct answer is A and C both r same
Explain: Unilateral erythematous nasal swelling could be nasopharyngeal angiofibroma which is
highly vascular swelling & prone to epistaxis. Or it could be simple hypertrophied turbinate.
Both needs decongestant as initial treatment. Pseudoephedrine and phenylephrine are 2 local
decongestant drops available. These r sympathomimetic drops.
558) Patient with cystic nodule (acne) and scars , what is the best treatment :
A. Retinoic .
B. Erythromycin .
C. Doxycycline .
The correct answer is A
Nodulocystic acne is characterized by comedones, inflammatory lesions, and large nodules
greater than 5 mm in diameter. Scarring is often evident
Current consensus recommends a combination of topical retinoid and antimicrobial therapy as
first-line therapy for almost all patients with acne.. Retinoid reduce abnormal desquamation,
are comedolytic, and have some anti-inflammatory effects.
As above pt is with severe disease it needs isotretinion, a systemic retinoid that is highly
effective in the treatment of severe, recalcitrant acne vulgaris
…………………………………………………………………………………………………………………………………………………….
.
559) Patient was presented by constipation, vomiting , abdominal distension , with old scar in
the lower abdomen , x ray showed dilated loops with air in the rectum , what is the best initial
management :
A. NGT decompression , and IV line .
B. Rectal decompression and antibiotics .
C. Suppositories .
The correct answer is A
…………………………………………………………………………………………………………………………………………………….
.
560) Which of the following is not live vaccine :
A. Hepatitis B .
B. MMR .
C. Oral polio
D. BCG
The correct answer is A
Hepatitis B is the only vaccine prepared by genetic engineering.
561) Six years old child was born to a mother with hepatitis B , he does not received any
vaccines before , what you will give now :
A. DTP , MMR , Hib
B. DTP , MMR .
C. DT , MMR , Hib
D. DT, MMR .
The correct answer is d
Except Hep B vaccine this patient can receive all the vaccines. Here in options some vaccines
are missing.
…………………………………………………………………………………………………………………………………………………….
.
562) Which of the following describes the end of the early inflammatory phase .
A. Formation of eschar .
B. Formation of ground base of collagen.
C. The end of angiogenesis.
The correct answer is C
…………………………………………………………………………………………………………………………………………………….
.
563) Female patient , known case of Vit D deficiency , smoking , and recurrent fall, which of the
following is the greatest exogenous risk for osteoporosis :
A. Advanced age .
B. Recurrent fall .
C. Vit D . D. Smoking .
The correct answer is D
Advanced age & female gender are the most important factors for osteoporosis but they are
endogenous.
Smoking is an independent exogenous risk factor for osteoporosis
564) Blood sugar in DM type 1 is best controlled by :
A. Short acting insulin .
B. Long acting .
C. Intermediate .
D. Hypoglycemic agents .
E. Basal and bolus insulin .
The correct answer is E
Very vague question. We can exclude hypoglycemic agents. Short acting insulin is best in
emergencies like DKA as it can be given IV. We can use either long acting alone daily or a
mixture of short & intermediate acting insulin daily. Basal & bolus , ( short acting + intermediate
or long ), bolus’ of short-acting or very-short-acting insulin before meals to deal with the
associated rise in blood-sugar levels at these times. In addition, they take an evening injection
of long- or intermediate-acting insulin that helps normalise their basal (fasting) glucose levels.
This offers greater flexibility and is the most commonly adopted method when intensified
insulin therapy is used to provide optimal glycaemic control.
…………………………………………………………………………………………………………………………………………………….
.
565) Which of the following is true regarding Perthe's disease :
A. Commonly seen between 11-16 years of age .
B. Always unilateral .
C. May present by painless limp.
D. Characteristically affect the external rotation of hip .
E. More in female .
The correct answer is C
Perthes disease (Leg-Calve perthes disease) is a condition affecting the hip joint where
there is degenerative avascular necrosis of the femoral head. It affects children aged 3-12 years
& is more common in males. It is most commonly but not always unilateral ( 85% is unilateral ).
It presents mainly by severe hip pain & limping that increases by movement but it can present
by painless limp. It characteristically affects the internal rotation & abduction of the hip & limits
these movements.
566 Which of the following is true regarding gastric lavage :
A. Patient should be in the right lateral position .
B. It is not effective after 8 hours of aspirin ingestion .
The correct answer is B. lavage is effective only 1 hour after ingestion of any poison.
…………………………………………………………………………………………………………………………………………………….
567) SSRI was prescribed to a patient with depression, the effect is suspected to be within :
A. One day .
B. Two weeks .
C. Three to four weeks . The correct answer is C
…………………………………………………………………………………………………………………………………………………….
568) Partner lost his wife by AMI 6 months ago , presented by loss of appetite , low mood ,
sense of guilt , what is the diagnosis :
A. Beverament.
B. Major depression episode .
The correct answer is A
- Major depression is a psychiatric condition that occurs regardless of events that happen in
life, while normally most people would have beverament after death of a close person.

569) An adult was presented by sore throat , congestion , fatigue , petechiae in soft palate ,
tender spleen , and liver , what is the most likely diagnosis : A. EBV .
This could be Rubella infection as petechia in soft palate (Forschheimer spots) occur in
rubella also. But liver & spleen affection are more common in EBV. Also since it is an adult then
we might suspect leukemia as the cause.
…………………………………………………………………………………………………………………………………………………….
570) Patient was presented by blepharitis , acne rosacea , but no keratitis , what is the best
treatment :
A. Topical chloramphenicol .
B. Topical gentamicin .
C. Oral doxycycline .
The correct answer is C
…………………………………………………………………………………………………………………………………………………….
571) Diabetic patient was presented by spastic tongue , dysarthria , spontaneous crying what is
the most likely diagnosis :
A. Parkinson .
B. Bulbar palsy .
C. Pseudobulbar .
D. Myasthenia gravis .
The correct answer is C
Spastic tongue, dysarthria and emotional liability are seen in pseudobulbar palsy.
Bulbar relates to the medulla. Bulbar palsy is the result of diseases affecting the lower cranial
nerves (VII-XII).
A speech deficit occurs due to paralysis or weakness of the muscles of articulation which are
supplied by these cranial nerves
Tongue - weak and wasted and sits in the mouth with fasciculations.
Drooling - as saliva collects in the mouth and the patient is unable to swallow (dysphagia).
Absent palatal movements.
Dysphonia - a rasping tone due to vocal cord paralysis; a nasal tone if bilateral palatal paralysis.
Articulation - difficulty pronouncing r; unable to pronounce consonants as dysarthria
progresses.
Pseudobulbar palsy results from disease of the corticobulbar tracts. Bilateral tract damage must
occur for clinically evident disease as the muscles are bilaterally innervated.

Aetiology of pseudobalbar pulsy

 Cerebrovascular events - eg, bilateral internal capsule infarcts.


 Demyelinating disorders - eg, multiple sclerosis.
 Motor neurone disease.
 High brainstem tumours.
 Head injury.
 Neurosyphilis
572) Which of the following is describe the normal developmental stage for 6 months old child :
A. Sits without support .
B. Rolls front to back . (7-8 months)
C. No head lag . (3 months)
D. Stand alone . (1 year)
The correct answer is A
…………………………………………………………………………………………………………………………………………………….
.
573) Which of the following personality is characterized by inflexibility, perfectionism ?
A. OCD .
B. Not otherwise specified .
C. Narcissistic .
The correct answer is A
Obsessive compulsive personality disorder is different from obsessive compulsive disorder
and is characterized mainly by perfectionism while OCD is characterized by repetitive actions
due to compulsion. if OCPD is present in the choices it would be the answer.
……………………………………………………………………………………………………………………………………………………
574) What is the commonest cause of otorrhea :
A. Acute otitis media .
B. Basal skull fracture .
C. External otitis .
The correct answer is A
…………………………………………………………………………………………………………………………………………………….
575) Patient was presented by ear pain , red tympanic membrane , apparent vessels , with
limited mobility of the tympanic membrane , what the most likely diagnosis :
A. Acute otitis media .
B. Tympanic cellulitis .
C. Mastoditis .
The correct answer is A
576) What is the management of acute congestive glaucoma : A. IV acetazolamide and topical
pilocarpine . (true)
…………………………………………………………………………………………………………………………………………………….
577) Eight years old child with late systolic murmur best heard over the sterna border , high
pitch , crescendo , decrescendo ,>> diagnosis is :
A. Physiological murmur .
B. Innocent murmur .
C. Ejection systolic murmur .
D. Systolic regurgitation murmur .
The correct answer is C
…………………………………………………………………………………………………………………………………………………….
578) Child was presented by Coryza , skin rash , conjunctivitis , and multiple spots in the mouth:
A. Measles .
B. Rubella .
The correct answer is A. Measles : due to conjunctivitis which is not present in rubella

…………………………………………………………………………………………………………………………………………………….
579) Which of the following could be seen in patient with bulimia :
A. Hypokalemia .
B. Metabolic acidosis.
The correct answer is A
bulimia is aka bing eating which means the patient eats a lot then does forced vomiting so
there is loss of acids & electrolytes which leads to hypokalemia & metabolic alkalosis.

580) Maximum spinal height is reached after menarche by how many years ?
A. Months .
B. Two years .
C. Three years .
The correct answer is C menarche is at 12-14 years and maximum height is about 17 years
…………………………………………………………………………………………………………………………………………………….
581) Pregnant lady 34 weeks of gestation presented by vaginal bleeding , which of the following
is relevant to ask about :
A. Smoking .
B. Desire of future pregnancy .
C. The result of last pap smear .
D. Hx of vaginal irritation .
The correct answer is A , because smoking is an important risk factor for antepartum Hg
…………………………………………………………………………………………………………………………………………………….
.
582) Female patient with marginal placenta Previa , with 6 cm dilated cervix , what is your
management :
A. Fetal monitoring .
B. Delivery if fully dilated .
C. Tocolytics >
The correct answer is A
I couldn’t find the answer correctly so based upon the given data, fetal monitoring is a
must in any case of high risk pregnancy. Also as the PP is marginal, there is possibility of
delivery without severe PPH and the cervix is already dilated.

583) Which of the following drugs increase the survival in a patient with heart failure :
A. Beta blocker .
B. ACE inhibitors .
C. Digoxin .
D. Nitrites .
The correct answer is B. New updated information. As ACE inhibitors inhibit aldosterone which
if present in high concentrations causes modification of the cardiac myocytes in the long term.
…………………………………………………………………………………………………………………………………………………….
.
584) Elderly patient presented by SOB , rales in auscultation , high JVP , +2 lower limb edema ,
what is the main pathophysiology :
A. Left ventricular dilatation .
B. Right ventricular dilatation .
C. Aortic regurgitation.
D. Tricuspid regurgitation .
The correct answer is A
Here we have both symptoms of Lft ventricular failure (SOB, Rales) & Rt ventricular failure
(High JVP & LL edema). So, more commonly left ventricular failure leads to rt ventricular failure
due to overload and not vice versa . so the most correct is Left ventricular dilatation.
…………………………………………………………………………………………………………………………………………………….
.
585) Patient with high output fistula , for which TPN was ordered , after 2 hours of the central
venous catheterization , the patient become comatose and unresponsive , what is the most
likely cause :
A. Septic shock .
B. Electrolytes imbalance .
C. Delayed response of blood mismatch .
D. Hypoglycemia .
E. Hypernatremia .
The correct answer is B
……………………………………………………………………………………………………………………………………………………
586) Child was presented by erythema , and swelling in his hand after 18 hours of bee sting ,
what is your management in this case :
A. Epinephrine .
B. Antihistaminic drugs .
C. Hospitalization .
The correct answer is B
18 hours is a long duration so its unlikely to be anaphylactic shock & more likely a regular
reaction to bee stings so antihistaminic is the best solution
…………………………………………………………………………………………………………………………………………………….
587) Which of the following is true regarding metformin :
A. Main complication is hypoglycemia .
B. Can lead to weight gain .
C. It suppress the hepatic gluconeogenesis .
The correct answer is C
…………………………………………………………………………………………………………………………………………………….
588) Child was presented by congested throat , coryza , high grade fever , which of the
following is true regarding this condition :
A. Viral > bacterial .
B. Bacterial > viral .
C. Antibiotics should be given any way .
D. It is most likely due to EBV .
The correct answer is B

589) Benign tumors of stomach represent almost :


A. 7 %
B. 21 %
C. 50 %
D. 90 %
The correct answer is A
…………………………………………………………………………………………………………………………………………………….
590) 60 years old patient presented by recurrent venous thrombosis including superior venous
thrombosis , this patient most likely has :
A. SLE .
B. Nephrotic syndrome.
C. Blood group O.
D. Antiphospholipid syndrome.
The correct answer is D
Antiphospholipid syndrome (APS) is an acquired autoimmune disorder that manifests clinically
as recurrent venous or arterial thrombosis and/or fetal loss
Thrombosis may involve the cerebral vascular system, coronary arteries, pulmonary system
(emboli or thromboses), arterial or venous system in the extremities, hepatic veins, renal veins,
ocular arteries or veins, or adrenal glands. Investigation is warranted if a history of deep venous
thrombosis (DVT, pulmonary embolism (PE), acute ischemia, myocardial infarction (MI), or
stroke (especially when recurrent) is present in a younger individual (males < 55 y; females < 65
y) or in the absence of other risk factors
…………………………………………………………………………………………………………………………………………………….
591) Elderly male patient was presented by left lower abdomen tenderness , fever , elevated
WBC count , what is the most likely diagnosis :
A. Inflammatory bowel .
B. Diverticulitis .
The correct answer is B
…………………………………………………………………………………………………………………………………………………….
592) 22 years old male patient was presented by recurrent attacks of diarrhea , constipation ,
and abdominal pain relieved after defecation , but no blood in the stool , no weight loss : what
is the diagnosis :
A. Irritable bowel Syndrome

593) Patient was presented by back pain relieved by ambulation , what is the best initial
treatment :
A. Steroid injection in the back .
B. Back bracing .
C. Physical therapy .
The correct answer is C
…………………………………………………………………………………………………………………………………………………….
594) Patient with long history of constipation presented by painful defecation , followed by
bleeding , pain persist for hours after defecation what is the diagnosis :
A. Anal fissure .
B. Thrombosed external piles .
C. Internal hemorrhoid .
The correct answer is A
…………………………………………………………………………………………………………………………………………………….
.
595) Elderly male patient underwent colectomy for colon cancer in which micrometastais was
detected in the lymph nodes , what is the best explanation :
A. Good prognosis .
B. Liver metastasis .
C. It is sensitive to chemotherapy . (Dukes class C cancer best for chemotherapy)
D. It is locally advanced .
The correct answer is C
…………………………………………………………………………………………………………………………………………………….
.
596) IV drug abuser was presented by fever , arthralgia , conjunctival hemorrhage , what is the
diagnosis :
A. Bacterial endocarditis .
…………………………………………………………………………………………………………………………………………………….
.
597) Female patient was presented by dysuria , epithelial cells were seen urine analysis , what is
the explanation in this case :
A. Contamination .
B. Infection .
The correct answer is B
598) Patient was presented by tremor, fever , palpitation , diagnosed as case of
hyperthyroidism , what is your initial treatment :
A. Surgery .
B. Radio iodine .
C. Beta blockers
D. Propylthioracil .
The correct answer is C, firstly B-blocker then Propylthioracil because we are afraid of
arrhythmias
…………………………………………………………………………………………………………………………………………………….
.
599) Young female patient presented by pain during the first 2 days of menses , menarche was
2 years back , what is your initial treatment :
A. OCP .
B. NSAID .
The correct answer is B , NSAID it is the best initial treatment for dysmenorrheal
…………………………………………………………………………………………………………………………………………………….
.
600) Female is planning to get pregnant , she want to get MMR vaccine also , what is your
action :
A. Delay the pregnancy 2-3 months after vaccination .
B. It is safe during pregnancy .
The correct answer is A
…………………………………………………………………………………………………………………………………………………….
.
601) Which of the following is true regarding varicella vaccine during breast feeding :
A. It is safe .
B. No breast feeding except after 3 days of the immunization .
The correct answer is A (Varicella vaccines are safe during lactation.)
Except for small pox vaccine and yellow fever vaccine, all others are relatively safe.
602) Patient underwent abdominal surgery due to intestinal perforation many years back ,
presented by abdominal pain , distension , constipation , what is the best investigation in this
case :
A. Barium enema .
B. Ultrasound .
C. Small bowel barium study .
The correct answer is C
…………………………………………………………………………………………………………………………………………………….
. 603) Patient with AMI and multiple PVC , is your treatment for this arrhythmia :
A. Amiodarone .
B. No treatment .
Correct answer is B
…………………………………………………………………………………………………………………………………………………….
604) Adenosine dose should be reduced in which of the following cases :
A. Chronic renal failure .
B. Patients on theophylline . (theophylline decrease the effect of adenosine)
The correct answer is B
 Second- or third-degree heart block (without a pacemaker)
 Sick sinus syndrome (without a pacemaker)
 Long QT syndrome.
 Severe hypotension.
 Decompensated heart failure.
 Asthma, traditionally considered an absolute CI. ...
 Poison/drug-induced tachycardia.
Patients with asthma is a contraindication. Here option B is correct as this patient is may be
taking theophylline for asthma control.
Theophylline also known to cause drug induced tachycardia.
…………………………………………………………………………………………………………………………………………………….
605) Which of the following is most likely seen in case of active glomerulonephritis : A. RBC
casts.
A urinalysis may show red blood cells and red cell casts in your urine, an indicator of possible
damage to the glomeruli.
A kidney biopsy is almost always necessary to confirm a diagnosis of glomerulonephritis.
606) Which the following is the commonest complication of patient with chronic atrial
fibrillation :
A. Sudden death . B. Cerebra vascular accidents .
The correct answer is B , due to multiple atrial thrombi

607) Target HbA1c is less than :


A. 6.5
B. 8
C. 9
The correct answer is A

608) Female patient with candida most likely has :


A. DM . B. SLE .
The correct answer is A
…………………………………………………………………………………………………………………………………………………….
609) Pregnant woman with suspected DVT , what is the best initial investigations :
A. Duplex US .
B. D dimer .
C. Platysomgraphy .
D. Venogram .
The correct answer is A

610) Patient with early rheumatoid arthritis , what is your management to decrease the
limitation of movement :
- Do not use analgesics or steroids, use DMARDs like methotrexate or antiTNF,
hydroxychloroquine
611 Which of the following indicates good prognosis in schizophrenia :
A. Family history of schizophrenia .
B. Gradual onset .
C. Flat mood .
D. Prominent affective symptoms .
E. No precipitating factors .
The correct answer is A
612) Which of the following is true regarding crohns disease :
A. Partial thickness involvement .
B. Fistula formation .
C. Continuous area of inflammation .
D. Mainly involve the recto sigmoid area .
The correct answer is B

613) Patient serology showed antibodies to the surface antigen for hepatitis B , what is your
diagnosis :
A. Previous infection or immunization .
B. Chronic carrier .
C. Highly infective .
The correcr answer is A
…………………………………………………………………………………………………………………………………………………….
.
614) Young male patient presented by acute scrotal pain , US showed reduced blood flow ,
what is the diagnosis :
A. Testicular torsion .
B. Trauma .
C. Infection .
D. Hernia .
The correct answer is A
…………………………………………………………………………………………………………………………………………………….
. 615) Female patient is sure that she is pregnant for 2 months , on examination , the uterus is
larger than suspected , B-hcg is very high , the doctor diagnosed her as having tumor which is
chemo sensitive , what is the diagnosis :
A. Ovarian cancer B. Endometrial cancer . C. Gestational trophoblastic .
The correct answer is c
…………………………………………………………………………………………………………………………………………………….
.
617) Which of the following is true regarding infertility :
A. It is Failure to conceive within 6 months . (1 year)
B. Male factor > female factors . (the reverse)
C. It could be due to high prolactin levels .
D. Rare to be due anovulotion . (common)
E. Only diagnosed by HSG . (need full lab & imaging investigations)
The correct answer is C

618) 32 years old female patient presented by irregular menses , menses occurs every two
months , on examination every thing is normal , which of the following is the LEAST important
test to ask about first :
A. CBC .
B. Pelvic US .
C. Coagulation profile .
D. DHES .
The correct answer is C
……………………………………………………………………………………………………………………………………….
620) Patient is known case of cervical spondylolysis, presented by paresthesia of the little finger
, with atrophy of the hypothenar muscles , EMG showed cubital tunnel compression of the
ulnar nerve , what is your action now :
A. Ulnar nerve decompression .
B. Steroid injection .
C. CT scan of the spine .
The correct answer is A

621) Patient with truncal obesity , easy bruising , hypertension , buffalo hump , what is the
diagnosis :
A. Cushing
622) Well known case of SCD presented by plueritic chest pain , fever , tachypnea , respiratory
rate was 30 , oxygen saturation is 90 % what is the diagnosis :
A. Acute chest syndrome .
B. Pericarditis :
C. VOC .
The correct answer is A , or pneumonia would be more correct if it was the answer
…………………………………………………………………………………………………………………………………………………….

623) Child with hemangioma around the eye , operation should be done within ……… :
A. 1 Week
B. 3 months .
C. 6 months .
The correct answer is A
We only fear of reduction in the function of the eye due to hemangioma so it should be
removed.
…………………………………………………………………………………………………………………………………………………….
624) Which of the following is the recommended diet to prevent IHD :
A. Decrease the intake of meat and dairy .
B. Decrease the meat and bread .
C. Increase the intake of fruit and vegetables .
????
The correct answer is C
…………………………………………………………………………………………………………………………………………………….
.625) Which of the following can lead to polyhydramnios :
A. Duodenal atresia .
B. Renal agenesis .
C. Post term pregnancy .
D. Diabetes inspidious . The correct answer is A
626) Adult polycystic kidney disease is inherited as:
A. Autosomal dominant .
B. Autosomal recessive .
C. X linked .
The correct answer is A
…………………………………………………………………………………………………………………………………………………….
627) The best treatment for bacteroid : A. Clindamycin .
In general, B. fragilis is susceptible to metronidazole, carbapenems, tigecycline, beta-
lactam/beta-lactamase inhibitor combinations (e.g., Unasyn, Zosyn), and certain antimicrobials
of the cephalosporin class, including cefoxitin.
…………………………………………………………………………………………………………………………………………………….
628) You have an appointment with your patient at 10 am who is newly diagnosed DM , you
came late at 11 am because you have another complicated patient , what are you going to say
to control his anger :
A. Do not say any thing .
B. Told him that there is another patient who really need your help .
Given the choices B is correct.
But other options are missing
…………………………………………………………………………………………………………………………………………………….
629) Well known case of DM was presented to the ER with drowsiness , in the investigations :
Blood sugar = 400 mg/dl , pH = 7.05 , what is your management ?
A. 10 units insulin + 400 cc of dextrose .
B. 0.1 unit/kg of insulin , subcutaneous .
C. NaHCO .
D. One liter of normal saline .
The correct answer is D
Treatment of dehydration is the first and important step in the management of DKA
…………………………………………………………………………………………………………………………………………………….
.
630) What is the best method for history taking :
A. Yes or no questions .
B. Open ended .
C. Silent listening .
The correct answer is B
663) What you will give to prevent hemorrhagic disease of newborns : A. Vitamin K .
…………………………………………………………………………………………………………………………………………………….
631) Arterial injury is characterized by :
A. Dark in color and steady .
B. Dark in color and spurting .
C. Bright red and steady .
D. Bright red and spurting .
The correct answer is D
…………………………………………………………………………………………………………………………………………………….
632) 14 years old girl complaining of painless vaginal bleeding for 2-4 days every 3Weeks to 2
months ranging from spotting to 2 packs per day; she had 2ry sexual ccc 1 year ago and had her
menstruation since 6 months on clinical examination she is normal sexual ccc, normal pelvic
exam appropriate action
a. OCP can be used
b. You should ask for FSH and prolactin level
c. Don’t do anything & explain this is normal
the correct answer is C
…………………………………………………………………………………………………………………………………………………….
. 633) 4 years old child what can he do
a. Copy square and triangle
b. Speak in sentences
the correct answer is A , if copy square 4 years , but if copy circle 3 years
…………………………………………………………………………………………………………………………………………………….
.
634) baby can sit without support, walk by holding furniture, Pincer grasp, pull to stand how old
is he
a. 8 months
b. 10 months
c. . 12 month
d. 18 month
The correct answer is B

635) Patient came after deep laceration at the anterior part of the wrist:
a. Wrist drop
b. Sensory loss only
c. Claw hand
d. Unable to do thumb opposition
The correct answer is D.
Radial nerve injury : wrist drop , common with humers injury (humers groove)
Ulnar nerve injury : claw hand , common with elbow injury
Median nerve inury :unable to do thumb opposition , common with wrist injury
…………………………………………………………………………………………………………………………………………………….
636) Best way to diagnose post streptococcus glomerulonephritis (spot diagnosis):
a) Low C3
b) ?or RBC casts c) ?
The correct answer is A
Most sensitive in diagnosis of post streptococcus glomerulonephritis is increase ASO titer :
lumpy-bumby immunofluorescence. or (streptozyme test, which tests antibodies to ASO) ( if
ASO titer written with MCQs , it is correct answer .
…………………………………………………………………………………………………………………………………………………….
637) Patient with lateral and vertical diplopia, he can’t abduct both eyes, the affected nerve is:
a) II b) III c) VI d) V
Correct answer is C. Lateral rectus muscle is supplied by abducens nerve
638) COPD patient with emphysema has low oxygen prolonged chronic high CO2, the
respiratory drive is maintained in this patient by:
a. Hypoxemia
b. Hypercapnemia
c. Patient effort voluntary
The correct answer is A
…………………………………………………………………………………………………………………………………………………….
639) Child with ear pain with positive pump test for tympanic membrane, treatment is:
a. Maryngiotomy
b. Amoxicillin/Potassium ……. ?
The correct answer is A

640) Patient has fatigue while walking last night. He is on atrovastatin for 8 months,
Ciprofluxacin, Dialtizem and alphaco….. the cause of this fatigue is:
a. Dialtizem and Atrovastatin
b. Atrovastatin and Ciprofluxacin
c. Atrovastatin and Alphaco d. ?
the correct answer is B
……………………………………………………………………………………………………………………………………………………
641) Patient with ischemic stroke present after 6 hours, the best treatment is:
a) ASA b) TPA c) Clopidogril d) IV heparin e) Other anticoagulant
The correct answer is A
1- TPA : administered within 3h of symptoms onset ( if no contraindication)
2- ASA: use with 48h of ischemic stroke to reduce risk of death.
3- Clopidogrel : can be use in acute ischemic
4- Heparin & other anticoagulant : in patient has high risk of DVT or AF
…………………………………………………………………………………………………………………………………………………….
642) Infant with hemangioma on forehead obscuring the vision, best thing to do is to remove
the lesion to avoid amblyopia, after:
a) One day b) One week c) One month d) 6 months e) One year
The correct answer is B
…………………………………………………………………………………………………………………………………………………….

643) Photophobia, blurred vision, keratic ?? behind cornea and cells in anterior chamber, the
best treatment is :
a) Topical antifungal
b) Topical Acyclovir
c) Antibiotic
A case of HSV keratouveitis
The correct answer is B
…………………………………………………………………………………………………………………………………………………….
644) Most common medical problems faced in primary health care is:
a) Coryza b) UTI c) Hypertension d) Diabetes
The correct answer is A
…………………………………………………………………………………………………………………………………………………….
645) Old lady afraid of Osteoporosis, to avoid the risk, you should advise her to do:
a) Weight bearing exercise and progressive resistance

646) Obese lady with essential hypertension, lab work showed: high Na, High K, The reason for
Hypertension is:
a) Obesity b) High Na intake c) High K intake d) ?
The correct answer is A , more than 85% of essential hypertension with BMI > 25 .
…………………………………………………………………………………………………………………………………………………….
647) Single thyroid nodule showed high iodine uptake, best treatment is:
a) Radio Iodine 131 b) Send home c) Antithyriod medication d) Excision if present
The correct answer is A
648) Athletic with tinea pedis best treatment is:
a) Topical antifungal
b) Systemic antifungal
c) Drug starts with terbinafine
The correct answer is A

649) Young patient with congested nose, sinus pressure, tenderness and green nasal discharge,
has been treated three times with broad spectrum antibiotics previously, what is your action?
a) Give antibiotic b) Nasal corticosteroid c) Give anti histamine d) Decongestant
The correct answer is B
Case of Chronic sinusitis
…………………………………………………………………………………………………………………………………………………….
650) Thyrotoxicosis include all of the following, Except:
a) Neuropathy
b) Hyperglycemia
c) Peripheral Proximal myopathy d)
Answer here is B
…………………………………………………………………………………………………………………………………………………….
651) Male patient has hair loss started as fronto-temporal and moving toward the vertex (top
of the head) the diagnosis is:
a) Androgenic alopecia b) Tinea Capitis c) ? d) ?
The correct answer is A
…………………………………………………………………………………………………………………………………………………….
652) Young healthy male has abdominal pain after basketball. Examination fine except for Left
periumbilical tenderness, what to do:
a) Abdominal US
b) Flat plate graph
c) Send home & reassess within 48 hours
The correct answer is A
653) Child with atopic dermatitis at night has stridor plus barking cough on and off from time to
time, diagnosis is:
a) BA
b) Croup
c) Spasmodic Croup
The correct answer is C Spasmodic croup : recurrent sudden upper airway obstruction
which present as stridor and cough . approximately 50% of children have atopic disease.
…………………………………………………………………………………………………………………………………………………….
654) Sickle cell anemia patient presented with asymptomatic unilateral hip pain, most likely
diagnosis is:
a) Septic arthritis b) Avascular Necrosis c) ?
The correct answer is B
…………………………………………………………………………………………………………………………………………………….
655) Best drug to treat depression in children and adolescent is: a) Fluxetine (Prozac) b) ?
…………………………………………………………………………………………………………………………………………………….
656) Old male with neck stiffness, numbness and paresthesia in the little finger and ring finger
and positive raised hand test, diagnosis is:
a) Thoracic outlet syndrome
b) Impingement syndrome
c) Ulnar artery thrombosis
d) Do CT scan for Cervical spine
The correct answer is A
657) Patient diagnosed with obstructive jaundice best to diagnose common bile duct
obstruction:
a) ERCP b) US c) ?
The correct answer is A
…………………………………………………………………………………………………………………………………………………….
658) Endometriosis best diagnosed by a) US b) Laparoscopy c) Laparotomy d) ?
The correct answer is B
659) Child was sick 5 days ago culture taken showed positive for meningococcal. Patient now at
home and asymptomatic your action will be:
a) Rifampicin
b) IM Ceftriaxone
Correct answer here is B
When oral rifampin (4 doses in 2 d) was compared with a single IM dose of ceftriaxone for
prophylaxis, follow-up cultures indicated that ceftriaxone was significantly more effective.
Ceftriaxone may provide an effective alternative to rifampin for prophylaxis in people in close
contact with patients with meningococcal meningitis.
…………………………………………………………………………………………………………………………………………………….
660) Infant with bright blood, black stool and foul smelling stool. Best way to know the
diagnosis:
a) US
b) Radio Isotope scan
c) Angiogram
EGD and colonoscopy are currently considered the first-line diagnostic and therapeutic
procedures of choice for both upper and lower gastrointestinal bleeding
Noninvasive imaging with technetium-99m–labeled red blood cell (RBC) or Tc-99m sulfur colloid
scintigraphy can be used to detect and localize gastrointestinal bleeding. Tc-99m RBC
scintigraphy is 93% sensitive and 95% specific for detecting a bleeding site with active arterial
or venous bleeding rates as low as 0.04 mL/min (16) anywhere within the gastrointestinal tract.

661) Primary hyperaldosteronism associated with:


a) Hypernatremia
b) Hypomagnesemia
c) Hypokalemia
d) Hyperkalemia
The correct answer is C
…………………………………………………………………………………………………………………………………………………….
662) Cardiac syncope: a) Gradual onset b) Fast recovery c) Neurological sequence after
The correct answer is B
663) A vaccination for pregnant lady with DT
a) Give vaccine and delivery within 24 hrs.
b) Contraindicated in pregnancy
c) Not contraindicated in pregnancy
The correct answer is C
…………………………………………………………………………………………………………………………………………………….
664) A study done to assess the risk of long taking Ca in two groups the diseased group with
long Ca plus control according to geographical location, site, and population. It adds (??) this
type of study:
a) Cohort
b) Case Control (retrospective)
c) Correlation study d) ?
The correct answer is A
Cross-Sectional Study: status of individual with respect to presence and absence of both
exposure and disease assessed at one point in time
case control study ( retrospective ) : samples a group of people who already have a particular
outcome (cases) and compares them to a similar sample group without that outcome (controls)
cohort study : ( prospective, incidence , longitudinal ) subjects are sampled and as a group are
classified on the basis of presence or absence of exposure to a particular risk factor
…………………………………………………………………………………………………………………………………………………….
665) Define Epidemiology
a) the branch of medicine which deals with the incidence, distribution, and possible control of
diseases and other factors relating to health
…………………………………………………………………………………………………………………………………………………….
666) Best way to decrease pain in elderly with bilateral knee pain and crepitation is:
a) NSAID
b) Decrease weight
c) Exercise
The correct answer is B
667) Using gastric lavage :
a) Useless after 8 hours of ASA ingestion
b) No benefit after 6 hours of TCA ingestion
The correct answer is A
Consider early gastric decontamination using charcoal if the patient presents within 2 hours of
ingestion TCA
Gastric lavage may increase drug absorption by pushing tablets into the small intestine.
Furthermore, gastric lavage in patients with TCA poisoning may induce hypoxia and a
tachycardia potentially increasing the risk of severe complications such as arrhythmias and
convulsions.
…………………………………………………………………………………………………………………………………………………….
668) 10 years old child with rheumatic fever treated early, no cardiac complication. Best to
advice the family to continue prophylaxis for:
a) 1 month b) 3 y s c) 4 ys d) 15 ys
The correct answer is D
American heart associated has recommended all patient with history rheumatic fever be
placed on long term penicillin prophylaxis. Duration of prophylaxis depends on presence or
absent of carditis, but for children without carditis, duration minimum 5 years or at age 21,
whichever is longer.

669) Picture of large neck mass only no other manifestations or organomegaly or


lymphadenopathy, diagnosis is:
a) Mononucleosis
b) I would say Goiter c) Lymphoma
The correct answer is B
…………………………………………………………………………………………………………………………………………………….
670) Patient with nausea, vomiting, and diarrhea developed postural hypotension.
Fluid deficit is: a) Intracellular b) Extracellular c) Interstitial
The correct answer is B
…………………………………………………………………………………………………………………………………………………….
671) Young female with whitish grey vaginal discharge KOH test ?? smell fish like diagnosis is:
a) Gonorrhea
b) Bacterial Vaginosis
c) Trichomonas Vaginalis d) ?
The correct answer is B
…………………………………………………………………………………………………………………………………………………….
672) Old lady with osteoporosis asked for treatment for prevention:
a) VIT. D
b) VIT. E
c) Retonic Acid d) ?
The correct answer is A
…………………………………………………………………………………………………………………………………………………….
673) Young male with morning stiffness at back relieved with activity and uveitis:
a) Ankylosing Spondylitis b) ?
…………………………………………………………………………………………………………………………………………………….
674) Best way to prevent infection in medical practice in pediatric
a) Wear gloves b) Wash hand c) Wear mask d) Wear gown
The correct answer is B
…………………………………………………………………………………………………………………………………………………….
675) High risk for developing colon cancer in young male is:
a) Smoking, high alcohol intake, low fat diet
b) Smoking, low alcohol intake, high fat diet
c) Red meat diet, garden’s disease (Gardner syndrome)
d) Inactivity, smoking
The correct answer is C
676) Alternative therapy for severe depression and resistance to anti-depressant medications
are:
a) SSRI b) TCA c) ECT
The correct answer is C
678) Patient had history of pancreatic cancer on chemotherapy then improved completely,
came to doctor concerning about recurrence of cancer and a history of many hospital visits.
This patient has:
a) Malingering b) Hypochondriasis c) Factitious d) Conversion
The correct answer is B
……………………………………………………………………………………………………………………………………………………
679) Patient came with neck swelling, ,moves when patient protrude his tongue. Diagnosis is:
a) Goiter b) Thyroglossal Cyst c) Cystic Hygroma d) ?
The correct answer is B
……………………………………………………………………………………………………………………………………………………
680) Pregnant patient came with neck swelling and multiple nodular non-tender goiter the next
evaluation is:
a) Thyroid biopsy
b) Give anti-thyroid medication
c) Radiation Iodine
d) TSH & Free T4, or just follow up
The correct answer is D
681) Young patient with HTN came complaining of high blood pressure and red, tender, swollen
big left toe, tender swollen foot and tender whole left leg. Diagnosis is:
a) Cellulitis
b) Vasculitis
c) Gout Arthritis d) ??
The correct answer is A , because tender and swollen whole left leg.
……………………………………………………………………………………………………………………………………………………
682) What is the injection that is routinely given to newborn to inhibit hemorrhage:
a) Vit. K
b) Vit.C
c) Vit. D d) Vit. E
The correct answer is A
683) Patient with strong genetic factor for colon cancer, what is the medication that could
decrease the risk of colon cancer:
a) Folic Acid.
b) Vit. C
c) Vit. K or A
d) Vit. E
The correct answer is A ( folaic acid and vit. C both are prevent colon cancer , but folate reduce
risk in people who genic predisposing )
…………………………………………………………………………………………………………………………………………………….
684) Patient with asthma, well controlled by albuterol, came complaining of asthma symptoms
not respond to albuterol, what medication could be added:
a) Corticosteroid inhaler
b) Long acting B-agonist
c) Oral corticosteroid
d) Theophylline
The correct answer is A

685) Henoch-Schonlein purpura affect:


a) Capillary
b) Capillary and venule
c) Arteriole, capillary and venule
d) Artery to vein
The correct answer is C
…………………………………………………………………………………………………………………………………………………….
686) Contraceptive pill that contain estrogen increase risk of:
a) Breast Ca b) Ovary Ca c) Cervical Ca d) ?
The correct answer is A
687) Patient came with upper respiratory tract infection with red conjunctiva, the cause is:
a) Viral infection
b) Bacterial infection
c) Fungal infection d) ?
The correct answer is A
…………………………………………………………………………………………………………………………………………………….
688) Healthy patient with family history of DM type 2, the most factor that increase chance of
DM are:
a) HTN and Obesity
b) Smoking and Obesity
c) Pregnancy and HTN
d) Pregnancy and Smoking
The correct answer is A

689) Patient complaining of back pain and hypersensitive skin of the back, on examination,
patient had rashes in the back, tender, red base distributed as blunt shape on the back,
diagnosis is:
a) Herpes Zoster
b) CMV
The correct answer is A
…………………………………………………………………………………………………………………………………………………….
690) Old patient complaining of hematuria, on investigation, patient has bladder calculi, most
common causative organism is:
a) Schistosoma b) CMV c) ? virus d) ? virus
The correct answer is A
…………………………………………………………………………………………………………………………………………………….
691) Blood culture show gram negative rod shape that grow only on charcoal free fungal
organism is:
a) Staph. Aureus b) Chlamydia c) Klebsiella d) Mycoplasma
Legionella : gram negative rod growth on charcoal agar (choose it if present)
692) Dermatomyosistis came with the following symptoms:
a) Proximal muscle weakness b) Proximal muscle tenderness c) ? d) ?
The correct answer is A
…………………………………………………………………………………………………………………………………………………….
693) Bursitis of the elbow joint caused by:
a) Elbow trauma
b) Autoimmune disease
c) Staph. Aureus
d) ? rupture of bursa
The correct answer is A
…………………………………………………………………………………………………………………………………………………….
694) Patient came with symptoms of anxiety including palpitation, agitation, and worry. The
first best line for treatment is:
a) SSRI
b) TCA
c) B-blocker
d) MAOI
The correct answer is A
……………………………………………………………………………………………………………………………………………………
695) Pregnant diagnosed with UTI. The safest antibiotic is:
a) Ciprofloxacin b) Ampicillin c) Tetracycline d) ?
The correct answer is B , but if present nitrofurantoin is more accurate answer .
UTI in pregnancy treated by : nitrofurantoin or cephalosporin ( 3– 7 days ) in symptomatic or
asymptomatic UTI .
696) Patient is complaining of right side pharynx tenderness on examination patient had
inflamed right tonsil and redness around tonsil with normal left tonsil. The diagnosis is:
a) Parenchymal tonsillitis b) Quinsy c) peripharyngeal abscess c) ? d) ?
The correct answer is B
………………………………………………………………………………………………………………………………………………
697) Patient came complaining of fever, night sweating, and hemoptysis with positive PPD test.
Examination was normal, CXR shows infiltrate of left apical lung but in lateral X-ray showed
nothing the repeated PPD test showed normal result diagnosis is: a) Sarcoidosis
b) Reactivated TB c) Mycoplasma infection d) Viral infection
The correct answer is b
…………………………………………………………………………………………………………………………………………………….
698) Female patient came with lower abdominal pain, fever on exam patient has lower
abdominal tenderness and tender cervical fornix, the most appropriate way to diagnose the
problem is:
a) Laparoscopy b) Heterosalpingography c) Abdominal CT d) Radionuclear Study
The correct answer is A
…………………………………………………………………………………………………………………………………………………….
699) The best non-medical therapy is proven to be of benefit for osteoarthritis is:
a) Muscle strength exercise b) Give NSAID c) Back slap d) ?
The correct answer is A

700) Hematological disease occurs in children, treated with heparin and fresh frozen plasma
what is the disease:
a) Hemophilia A
b) Hemophilia B
c) Von-wille brand disease
d) DIC thrombosis
The correct answer is D
……………………………………………………………………………………………………………………………………………………
701) Child with URTI is complaining of bleeding from nose, gum and bruising the diagnosis is:
a) Hemophilia A
b) ITT (ITP)
The correct answer is b
…………………………………………………………………………………………………………………………………………………….
702) Patient came complaining of vague abdominal pain for 6 hours then shifted to right lower
quadrant diagnosis is :
a) Acute appendicitis
b) Diverticulitis c) ? d) ?
The correct answer is A
…………………………………………………………………………………………………………………………………………………….
703) Female patient is complaining of abdominal distension, fever and nausea abdominal x-ray
showed (Ladder sign) management is:
a) Colostomy
b) Ileus treatment
c) Rectal de-obstruction d) Exploratory laparoscopy
The correct answer is b ( Some option is missing here)
Initial emergency department (ED) treatment consists of aggressive fluid resuscitation, bowel
decompression, administration of analgesia and antiemetic as indicated clinically, early surgical
consultation, and administration of antibiotics.
Initial decompression can be performed by placement of a nasogastric (NG) tube for suctioning
GI contents and preventing aspiration

704) The best stimulus for breast milk secretion is :


a) Estrogen b) Breast feeding c) ?
The correct answer is B
……………………………………………………………………………………………………………………………………………………
705) Female patient did urine analysis shows epithelial cells in urine, it comes from:
a) Vulva
b) Cervix
c) Urethra
d) Ureter
The correct answer is C
706) All of the following are risk factors for heart disease except:
a) High HDL
b) Male
c) Obesity
The correct answer is A
…………………………………………………………………………………………………………………………………………………….
707) The most signs and symptoms of abruption of placenta is:
a) Vaginal bleed
b) Fetal distress
c) Uterus pain and back pain
d) Abnormal uterine contraction
The correct answer is A

708) Sign of severe hypokalemia is:


a) P-wave absence
b) Peak T-wave
c) Wide QRS complex
d) Seizure
The correct answer is D.
Severe hypokalemia is not linked with any symptoms, but may cause:
1- muscle
2- myalgia or muscle pain
3- disturbed heart rhythm including ectopy (disturbance of the electrical conduction system of
the heart where beats arise from the wrong part of the heart muscle)
4- serious arrhythmias (electrical faster or slower than normal)
5- greater risk of hyponatremia with confusion and seizures
ECG changes in hypokalemia :
1-T-wave flattening
2-U-wave : ( additional wave after the T wave )
3-ST – segment depression
ECG changes in hyperkalemia :
1- peak T wave
2- wide QRS ( in severe case )
3- PR prolong ( in severe case )
4- loss of P wave
…………………………………………………………………………………………………………………………………………………….
709) Child came with his father and high BMI and look older than other children with same age,
on exam child has >95th percentile of weight and tall, management is:
a) Observe and appoint
b) Life style changes
c) Give program to decrease the weight
The correct answer is B
Both B and c are correct here.
Life style modification includes diet changes and exercises

710) Pregnant on 36th week came with 7 cm cervical width at 0 station. During birth, CTG
shows late deceleration, management is:
a) Give Oxytocin
b) O2 and change mother position
c) Give Mg sulfate
The correct answer is B

711) The way to determine the accuracy of occult blood test for 11,000 old patients is by
measuring:
a) Sensitivity
b) Specificity
c) Positive predictive value
d) Negative predictive value
The correct answer is A
712) Sickle cell patient, asymptomatic with history of recurrent gall-stones and recurrent crisis
the management is:
a) Cholecystectomy b) Hydroxyurea
The correct answer is A
…………………………………………………………………………………………………………………………………………………….
713) Patient came with HTN, KUB shows small left kidney, arteriography shows renal artery
stenosis, what is the next investigation:
a) Renal biopsy
b) Renal CT scan
c) Renal barium
d) Retrograde pyelography
The correct answer is B
…………………………………………………………………………………………………………………………………………………….
714) The way to differentiate between low iron level from iron deficiency anemia and anemia
of chronic disease is:
a) Ferritin
b) TIBC
c) Serum Iron
d) Serum Transferrin
The correct answer is A ( B is also correct but A is more specific)

715) Patient came with hallucination and illusion the medication that should be given is:
a) Carbamazepine b) ? Haloperidol c) ?
the correct answer is B
…………………………………………………………………………………………………………………………………………………….
716) As doctor if you see patient and you face difficulty to get accurate information from him
the best tactic to do it is:
a) Ask direct question b) Ask open question c) Control way of discussion d) ?
The correct answer is A
717) (long question) patient came with MDD so during communication with patient you will
find : (Major depressive disorder)
a) Hypomania b) Late morning awake c) Loss of eye contact d) ?
The correct answer is B
MAJOR DEPRESSIVE DISORDER
At least 2 weeks of a major depressive episode which causes significant distress or disability
No history of mania or hypomania
Not due to a medical or substance use disorder
…………………………………………………………………………………………………………………………………………………….
718) Child patient after swimming in pool came complaining of right ear tenderness on
examination patient has external auditory canal redness, tender, and discharge the
management is:
a) Antibiotics drops b) Systemic antibiotics c) Steroid drops d) ?
The correct answer is A

719) Child came with inflammation and infection of the ear the most complication is:
a) Labrynthitis b) Meningitis c) Encephalitis d) Mastoiditis
The correct answer is D
Most common extra cranial complication is post auricular abscess and most common intra
cranial complication is meningitis
…………………………………………………………………………………………………………………………………………………….
720) Elderly patient complaining of urination during night and describe when he feel the
bladder is full and need to wake up to urinate, he suddenly urinate on the bed this is:
a) Urgency incontinence
b) Urge incontinence
c) Stress incontinence
d) Flow incontinence
The correct answer is D Check Q 756 in Pearson file for explanation
722) Newborn came with red-lump on left shoulder, it is:
a) Hemangioma b) ? c) ?
Q is not complete
…………………………………………………………………………………………………………………………………………………….
723) Child came to ophthalmology clinic did cover test, during eye cover , his left eye move
spontaneously to left, the most complication is:
a) Strabismus b) Glaucoma c) Myobloma d) ?
The correct answer is A
…………………………………………………………………………………………………………………………………………………….
724) Newborn came with congenital hepatomegaly, high LFT, jaundice the most organism cause
this symptoms is:
a) Congenital TB b) Rubella c) HIV d) CMV
The correct answer is D
…………………………………………………………………………………………………………………………………………………….
725) Gross motor assessment at age of 6 months to be asked is:
a) Sitting without support
b) Standing
c) Role from prone to supine position
d) Role from supine to prone position
The correct answer is A

726) Female child came with precocious puberty the most cause is:
a) Idiopathic
b) Adrenal tumor
c) Brain tumor
d) ? ovarian tumor
The correct answer is A
…………………………………………………………………………………………………………………………………………………….
727) The immediate urgent referral of child that take
a) 10 pills contraceptive
b) 10 pills antibiotics?
c) 75 mg ?
Answer here could be B

728) Hemorrhoid usually occurs in:


a) Pregnancy and portal HTN
Q is not complete Most common causes of hemorrhoid : increase straining ( constipation ) ,
portal HTN , increase abdominal pressure ( chroming cough , pelvic tumor ) , obesity ,
pregnancy , smoking
…………………………………………………………………………………………………………………………………………………….
729) Most unwanted side effect of anti-cholinergic drugs is :
a) Constipation b) ? c) ?
more than 50% of patients taking anticholinergic have side effects : dry mouth, blurry vision,
constipation and urinary retention.
730) Patient with DM II with good vision, to prevent eye disease (Retinopathy) to develop is to
avoid:
a) HTN, Smoking
b) Obesity, Smoking
c) HTN, Obesity d) ?
The correct answer is C
The risk factors that increase diabetic retinopathy background are: 1- HTN 2- Poor glucose
control or long case D.M 3- Raised level of fat ( cholesterol) 4- Renal disease 5- Pregnancy ( but
not in diabetes caused by pregnancy )
…………………………………………………………………………………………………………………………………………………….
731) Best medication to be given for GDM (gestational) is:
a) Insulin b) Metformin c) ?
The correct answer is A
732) A child is complaining of severe headache which is unilateral, throbbing and aggravated
by light, diagnosis is:
a) Migraine b) Cluster Headache c) Stress Headache d) ?
The correct answer is A
…………………………………………………………………………………………………………………………………………………….
733) The most important factor for smoker to quit is :
a) Patient desire
b) Give nicotine pills
c) Give programmed plan
d) Change life style
The correct answer is A
…………………………………………………………………………………………………………………………………………………….
734) Patient is complaining of irritation, tachycardia, night sweating, labs done showed TSH:
Normal, T4: High, diagnosis is:
a) Grave’s disease
b) Secondary Hypothyroidism
c) Hashimoto’s thyroiditis d) ?
With given info answer here is A.
.……………………………………………………………………………………………………………………………………………………
735) The most active form is:
a) T4 b) T3 c) TSH d) TRH
The correct answer is B

736) Middle age man found to have heaviness in his groin. On physical examination there was
swelling just above his testis which apparent with Valsalva maneuver. What is the diagnosis:
a) Direct inguinal hernia
b) Indirect inguinal hernia
c) Femoral Hernia d) Testicular mass e) Hydrocele f) Varicocele
The correct answer is B
737) Gardener has recurrent conjunctivitis. He can’t avoid exposure to environment. In order to
decrease the symptoms in the evening, GP should advise him to:
a) Cold compression
b) Eye irrigation with Vinegar Solution
c) Contact lenses
d) Antihistamines
The correct answer is D
…………………………………………………………………………………………………………………………………………………….
738) 48 year-old male complaining of lower back pain with morning stiffness for 30 minutes
only. On exam he was having spasm centrally on the lower back. What is the appropriate
management :
a) Epidural steroids injection
b) Back brace
c) Facet lysis
d) Physiotherapy
The correct answer is D
…………………………………………………………………………………………………………………………………………………….
739) 41 weeks pregnant lady last biophysical profile showed oligohydramnios. She has no
complaints except mild HTN. What is the appropriate management :
a) Wait
b) Induce labor post 42 wks.
c) Induce labor
d) Do biophysical profile twice weekly
The correct answer is C
740) Full term wide pelvis lady, on delivery station +2, vertex, CTG showed late deceleration,
the most appropriate management:
a) C/S b) Suction c) Forceps Delivery d) Spontaneous Delivery
The correct answer is C
…………………………………………………………………………………………………………………………………………………….
741) Recent study revealed that anti psychotic meds cause the following complication:
a. wt gain b. alopecia c. cirrhosis
the correct answer is A
…………………………………………………………………………………………………………………………………………………….
742) compilication of rapid correction of hypernatremia :
a. brain edema
in hypernatremia : gradually correction to prevent cerebral edema
in hyponatremia : gradually correction to prevent ,myelinolysis, quadriparesis , dysarthria and
coma
……………………………………………………………………………………………………………………………………………………
743) prophylaxis of cholera :
Cholera is an infection of the small intestine that is caused by the bacterium Vibrio cholera.
The main symptoms are profuse watery diarrhea ( rice-watery diarrhea ) vomiting and
dehydration . infection by fecal-oral rout .
treatment : 1- rehydration , 2-antibiotic : young & adult : doxycycline or tetracycline , for
children : SMX-TMP , for pregnant : furazolidone .
prophylaxis : good hygiene and sanitation and oral vaccine , in epidemic public : mass single
dose of vaccine and tetracycline.

744)which one of the anti TB medications cause tinnitus, imbalance..


a. streptomycin
b. isoniazid
c. pyrazinamide
the correct answer is A , streptomycin : cause 8th nerve damage
…………………………………………………………………………………………………………………………………………………….
745) young male had pharyngitis, then cough ,fever, most likely org
a. staph aureus
b. strept pneumonia
the correct answer is B
…………………………………………………………………………………………………………………………………………………….
746)most effective measure to prevent spread of infection among health care workers & pts in
a nursery:
a. wash hand before and after examining each pt
b. wear gown and gloves before entering the nursery
c. wear shoe cover
the correct answer is A

747) 27 yrs old female with perianal pain for 4 days tender erythematous fluctuating swelling
a. Abx
b. local CS
c . Sitz bath
d. evacuation & drainage
the correct answer is D
…………………………………………………………………………………………………………………………………………………….
748) female pt developed extreme fear from zoo, park, sporting events, the fear prevented her
from going out:
a. agoraphobia
b. social phobia
c. schizophrenia
the correct answer is A
…………………………………………………………………………………………………………………………………………………….
749)child starts to smile:
a. at birth
b. 2month
c. 1month
The correct answer is B
…………………………………………………………………………………………………………………………………………………….
750)child recognize 4 colors, 5 words, hops on one foot, consistent with which age:
a. 12 mons
b. 24 mons
c. 36 mons
d. 18 mons
the correct answer is C
751) pt with hx of 5 yrs HTN on thiazide, came to ER midnight screaming holding his Lt foot, o/e
pt afebrile, Lt foot tender erythema, swollen big toe most tender and painful, no other joint
involvement
a. cellulitis
b. gouty arthritis
c. septic arthritis
the correct answer is B
…………………………………………………………………………………………………………………………………………………….
752)23 yrs old married for 3 months, c/o not getting pregnant, they have intercourse 3-4
times/week, normal gynecologic hx, husband 25 yrs old healthy wt would you advice:
a. cont. trying
b. obtain sperm analysis
c. study of tubes patency
the correct answer is A
…………………………………………………………………………………………………………………………………………………….
753) pt with AF what is the most common complication:
a. cerebrovascular events b. v.tach c. AMI d. v.fib
the correct answer is A
…………………………………………………………………………………………………………………………………………………….
754) case scenario : oral + genital ulcer +arthritis
a. behcet disease b. syphilis c. herpes simplex

the correct answer is A


755) 20 yrs old lady, pregnant, exposed to rubella virus since 3 days , never was vaccinated
against rubella mumps or measles , what's the best thing to do:
a. give IG
b. vaccine
c. do nothing
d. terminate the pregnancy
the correct answer is A
If presented 72 hours of exposure and she is not immunized, the better option here is to check
for rubella serology. If rubella IgM is positive, discuss the risk of continuing pregnancy with the
patient and terminate if patient accepts
Immunoglobulin (IG) does not prevent rubella or mumps infection after exposure and is not
recommended for that purpose
IG is not recommended for routine post exposure prophylaxis of rubella in early pregnancy or
any other circumstance.
Administration of IG should be considered only if a pregnant woman who has been exposed to
rubella will not consider termination of pregnancy under any circumstances. In such cases,
intramuscular administration of 20 mL of IG within 72 hours of rubella exposure might reduce --
- but will not eliminate --- the risk for rubella.
…………………………………………………………………………………………………………………………………………………….
756)which of the following not compatible with head engagement:
a. vertex at zero station
b. crowning of the head
c. 3/5 head felt in the abdomen
d. BPD at ischial spines
the correct answer is C ,
When the fetal head is engaged, 2/5 or less of the head is palpable above the pelvic

757) 1st line in Trigeminal Neuralgia management: a. Carbamazepine


758) 58 yrs old female, known case of osteopenia, she's asking you abt the best way to prevent
compression vertebral fracture, what would you advice her:
a. avoid obesity
b. vit. D daily
c. wt. bearing exercise
the correct answer is C
Natural ways to prevent spinal compression fractures include taking calcium supplements,
getting more vitamin D, quitting smoking, preventing falls, and doing weight-bearing and
strength-building exercises.
…………………………………………………………………………………………………………………………………………………….
756) Mother complains of sharp pain on radial styloid when carrying her baby. The pain
increase with extension of the thumb against resistance, Finkelstein test was positive, Dx :
a. Osteoarthritis of radial styloid
b. De Quervain Tenosynovitis
the correct answer is B
…………………………………………………………………………………………………………………………………………………….
757) Contraindicated in acute glaucoma management:
a. Pilocarpine
b. Timolol
c. B-blockers, CA inhibitors, NSAID, Mannitol
Anti cholinergic or sympathomimetic drops are contraindicated
Pilocarpine is para sympathomimetic
All of them can be use in acute glaucoma . may be choice d is correct if mention.

758) True about systolic hypertension:


a. could be caused by mitral Regurgitation
b. More serious than diastolic hypertension
c. systolic>140 and diastolic <90
the correct answer is C
…………………………………………………………………………………………………………………………………………………….
759) Pregnant 38GA, presented in labor, dilated cervix, station +2, late deceleration on CTG,
management:
a. continue spontaneous labor
b. Forceps delivery
c. Vacuum Delivery
d. CS
The correct answer is A
…………………………………………………………………………………………………………………………………………………….
760) Pt, febrile, tender prostate on PR:
a-Acute Prostatitis
…………………………………………………………………………………………………………………………………………………….
761) Proven to prevent some cancers:
a. Ca
b. Folic Acid
c. Vit. D
the correct answer is C
…………………………………………………………………………………………………………………………………………………….
762) Patient with cont. Murmur:
a. PDA b. Coarctation of Aorta
the correct answer is A
763) Old man, post OR, complains of unilateral face swelling:
a. Sjogran syndrome
b. Sarcoidosis
c. Salivary gland tumor
d. Salivary gland stone
the correct answer is D
…………………………………………………………………………………………………………………………………………………….
764) At what level LP done:
a. L2-L3
b. L3-L4
c. L5-S1
The correct answer is B
…………………………………………………………………………………………………………………………………………………….
765) 2months old with scaling lesion on scalp and forehead, Dx:
a. Seborrheic Dermatitis
b. Erythema multiforme
The correct answer is A
…………………………………………………………………………………………………………………………………………………….
766) Pt have high Blood Pressure on multiple visits, so he was diagnosed with hypertension,
what is the Pathophysiology:
a. increased peripheral resistance b. increased salt and water retention
the correct answer is A
…………………………………………………………………………………………………………………………………………………….
767) 13years old with hx of pneumonia and managed with abx 2 weeks back, now he came with
diarrhea, abdominal pain, and +ve WBC in stool, the causative organism is:
a. Clostridium difficile
…………………………………………………………………………………………………………………………………………………….
768) Best Manag. Of Clost. Defficile is:
a. Metronidazole b. Doxycycline
the correct answer is A

769) Prophylaxis of arrhythmia post MI :


a. Quinidine b. Quinine c. Lidocaine d. Procainamide
the correct answer is C , if b-blocker is present choose it
…………………………………………………………………………………………………………………………………………………….
770) Chronic Diarrhea is a feature of:
a. HyperNatremia
b. HyperCalcemia
c. HypoMagnesemia
d. Metabolic Alkalosis
The correct answer is D
…………………………………………………………………………………………………………………………………………………….
771) Pt with have Polyuria and thirst, he had Hx of bipolar on lithium, Dx:
a. Nephrogenic Diabetes Insipidus
b. central Diabetes Insipidus
the correct answer is A
…………………………………………………………………………………………………………………………………………………….
772) In city with population of 15000 people & 105 birth /yr , 4 are still birth ,3 died within
months ,2 died before their 1st birthday , with 750 p come out & 250 come in what is the birth
mortality rate in this city :
a. 4 b. 6 c. 8 d. 9

773) Female with recently inserted IUCD coming with watery brownish vaginal discharge &
abdominal pain what is the most likely Dx :
a. Uterine rupture
b. Ovarian torsion
c. Bacterial vaginosis
d. Ectopic pregnancy
The correct answer is C
…………………………………………………………………………………………………………………………………………………….
774) Pregnant woman with symptoms of hyperthyroidism , TSH low :
a- Propylthioracil b- Radio-active iodine c- Partial thyroidectomy d- ………….
The correct answer A
775) 38 y/o male calling you complaining of fear from going outside & fear from seeing the
front door & don’t sit in the room containing the front door what is most likely Dx:
a. Depression
b. Agoraphobia
c. Malingering
d. Anti-social personality
The correct answer is B
…………………………………………………………………………………………………………………………………………………….
776) 60 y/o male known to have ( BPH) digital rectal examination shows soft prostate with
multiple nodularity & no hard masses , the pt request for ( PSA) for screening for prostatic ca
what will you do :
a. Sit with the pt to discuss the cons & rods in PSA test
b. Do trans-rectal US because it is better than PSA in detection
c. Do multiple biopsies for different sites to detect prostatic ca d. …………………….
The correct answer is A
…………………………………………………………………………………………………………………………………………………….
777) What is the most effective ttt for rosacea :
a. Clindamycin
b. Erythromycin
c. Topical steroids
Answer could be B
Treatment of Rosacea : Topical metronidazole is commonly used as a first-line agent.
Oral tetracycline & topical metronidazole or topical erythromycin or topical clindamycin .
For severe case : isotretinion . surgical treatment for rhinophyma.
Dapsone has been used in severe, refractory rosacea, and dapsone has been particularly
beneficial for patients who cannot take isotretinion
778) common site of anal fissure is :
a- Anterior b- Posterior c- Lateral d- …………………….
The correct answer is B
779) 7 y/o child coming with SOB and wheezing he was sitting in bed , leaning forward , with
drooling & strider what is Dx :
a- Epiglottitis b- Bronchial asthma c- ……… d- ………
The correct answer is A
…………………………………………………………………………………………………………………………………………………….
780) 10 y/o child brought by his parents because they where concern about his weight , he eats
a lot of fast food and French fries , your main concern to manage this pt is :
a- His parents concerning about his weight
b- His BMI > 33
c- Family Hx of heart disease
d- Eating habit ( fast food , French fries )
The answer is B
…………………………………………………………………………………………………………………………………………………….
781) What is an absolute contraindication of OCP :
a. Hx of previous DVT b. Ovarian ca c. Breast ca
The correct answer is A , both hx of DVT and breast cancer are absolute contraindication , but in
hx DVT is more accurate
…………………………………………………………………………………………………………………………………………………….
782) OCP is proven to :
a. Decrease ovarian ca
b. Decrease endometrial ca
c. Increase breast ca
d. Increase risk of ectopic pregnancy
The correct answer is C (OCP is proven to all except (D) )

783) Baby born & discharge with his mother , 3weeks later he started to develop difficulty in
breathing & become cyanotic what is most likely DX :
a- VCD b- Hypoplastic lt ventricle c- Coarctation of aorta d- Subaortic hypertrophy
The correct answer is B
784) 60 y/o male Dx to have acute pancreatitis what is the appropriate nutrition :
a- TPN
b- Regular diet with low sugar
c- High protein ,high ca , low sugar
d- Naso-jujenal tube
The correct answer is D
…………………………………………………………………………………………………………………………………………………….
785) 15 y/o boy brought by his mother with lab values shows microcytic hypochromic anemia
what is most likely Dx :
a- Thalassemia
b- IDA ( iron deficiency anemia )
c- SCD d- …….
The correct answer is B
…………………………………………………………………………………………………………………………………………………….
786) Female with Hx of lt flank pain radiating to groin , symptoms of UTI & N+V what is DX:
a- Appendicitis
b- Diverticulitis
c- Renal colic d- ……..
The correct answer is C

787) Old pt with neck swelling, nodular ,disfiguring , with Hx of muscle weakness , cold
intolerance , hoarseness , what is your management :
a- Levothyroxine
b- Carbamazole
c- Thyroid lobectomy
d- Radio-active iodine
The correct answer A
…………………………………………………………………………………………………………..
788) Pt with thyroid mass , firm ,2x2 cm what is most appropriate for Dx :
a- Neck US
b- FNA
c- Neck CT
d- Surgery
The correct answer is B
…………………………………………………………………………………………………………………………………………………….
789) 56 y/o female complaining of back pain , her mother suffer from vertebral compression Fx
, investigation shows early osteoporosis what is the best ttt :
a- Advice with exercise
b- Vit D , ca supplement , ……. c- ……………. d- ……………
The correct answer is B
If you have alendronate in the choices then choose it
…………………………………………………………………………………………………………………………………………………….
790) What is the most common non-traumatic Fx caused by osteoporosis :
a- Colle's Fx
b- Femoral Fx
c- Vertebral compression Fx
The correct answer is C , because most common non traumatic .

791) 4 or 5 ( not sure ) brought by his parents with weight > 95th percentile , height < 5th
percentile & bowing of both legs what is the appropriate management :
a- Liver & thyroid function tests
b- Lower limb X-ray
c- Pelvis X-ray
d- Thyroid or ( liver not sure ) function test
The correct answer is A
792) At which stage sapration of chromatids occur :
a- Metaphase b- Telophase c- Anaphase d- ……………
The correct answer is A
793) pt known case of ulcerative colitis with erythematous rash in lower limb what is most likely
DX:
a- erythema nodusum

794) Pt known to have ulcerative colitis coming with skin lesion around Tibia which is with
irregular margins what is most likely Dx:
a- Pyoderma gangirenosum
……………………………………………………………………………………………………………………………………………
795) Young pt victim of RTA coming with SOB ( or not breathing not sure ) absent of breath
sound & chest movement in RT side but resonance in percussion what is your next step :
a- Do stat chest X-ray
b- Emergency surgery
Do x-ray first followed by Needle aspiration then chest tube
……………………………………………………………………………………………………………………………………………
796) Which psychiatric disease is treated with electroconvulsive therapy : a- Paranoia b-
The correct answer is : Major depression

797) Pt with bilateral eye discharge , watery , red eyes, corneal ulceration what is the most
common cause :
a- Dust & pollen
b- Hypertension
c- Ultra-violet light & stress
The correct answer is A
……………………………………………………………………………………………………………………………………………
798) 4 y/o baby felt down his mother pulled him by his arm & since then he kept his arm in
pronation position what is your management:
a- Splint ………….
b- Do x-ray for the arm before any intervention
c- Orthopedic surgery
The correct answer is B
……………………………………………………………………………………………………………………………………………
799) diabetic pt on medication found unconscious his blood sugar was 60 what is the most
common to cause this problem:
a- Sufonylurease b- Bigunides
The correct answer is A

800) what is most sensitive indicator for factitious fever :


a- Pulse rate
Factitious fever: Fever produced artificially by a patient. This is done by artificially heating the
thermometer or by self-administered pyrogenic substances. An artificial fever may be
suspected if the pulse rate is much less than expected for the degree of fever noted. This
diagnosis should be considered in all patients in whom there is no other plausible explanation
for the fever. Patients who pretend to have fevers may have serious psychiatric problems.

801) young male with headache involving the whole head & bilateral eye globe that increase
with stress what is most likely DX:
a- Tension headache b- Migraine
Correct answer here is A
……………………………………………………………………………………………………………………………………………
802) pt turns to be erratic , for 4 month he said that’s people in TV knows what people are
thinking about , in last 2 month he claim that he has special power that no one has what is the
most likely DX :
a- Uni-polar depression b- Bipolar …. Mania c- SCZ
The correct answer is C
803) common site of hernia sac in relation to the cord content :
a- Antero-lateral
b- Medial c
- Anywhere
d- Posterior
The correct answer is A

804) what a 4 years child can do :


a- Draw square & triangle b- Say complete sentence c- Tie his shoes
The correct answer is A
…………………………………………………………………………………………………………………………………………
805) You received a call from a father how has a son diagnosed recently with DM-I for six
months, he said that he found his son lying down unconscious in his bedroom, What you will
tell him if he is seeking for advise:
a. Bring him as soon as possible to ER
b. Call the ambulance
c. Give him his usual dose of insulin
d. Give him IM Glucagon
e. Give him Sugar in Fluid per oral
The correct answer is D
……………………………………………………………………………………………………………………………………………
806) factor which determine recurrence of breast cancer :
a- Site & size of breast mass
b- Number of lymph nodes
c- Positive estrogen receptor
d- Positive progesterone receptor
The correct answer is B
……………………………………………………………………………………………………………………………………………
807) teacher in school presented with 3 days Hx of jaundice ( they wrote it ictric as if it will a
differacne) & abdominal pain , N+V (nausea and vomiting) , 4 of school student had the same
illness in lab what is true regarding this pt :
a- Positive for hepatitis A IgG
b- Positive hepatitis A IgM
c- Positive hepatitis B core
d- Positive hepatitis B c anti-body
The correct answer is B

808) regarding weight gain in pregnancy what is true :


a- Pregnant woman should consume an average calorie 300-500 per day
b- Regardless her BMI or body weight she should gain from 1.5 – 3 lb which represent the
baby's growth

Weight gain during pregnancy : - 100 – 300 Kcal / day , 500 Kcal / day in breastfeeding -
Wt. gain : 1 – 1.5 kg / month , 11 – 16 kg gain during pregnancy.
……………………………………………………………………………………………………………………………………………
809) what condition is an absolute contraindication of lactation :
a- Mother with open pulmonary TB for 3 month
b- Herpes zoster in T10 drematome
c- Asymptomatic HIV
The correct answer is C
……………………………………………………………………………………………………………………………………………
810) 70 y/o female say that she play puzzle but for a short period she can't play because as she
develop headache when playing what u will exam for
a. astigmatism b. glaucoma
the correct answer is A
811) what is most sensitive for DX of duodenal ulcer :
a- Epigastric pain starting 30-60 min after the meal
b- Epigastric pain staring immediately after a meal
c- Increasing of pain when lying supine
d- Pain radiating to the back The correct answer is A

812) female at 40 yr with BMI >28 what is your management:


a- Reduce calorie intake to 800 /day
b- In general reduce calorie intake
the correct answer is B
…………………………………………………………………………………………………………………………………………………….
813) prophylaxis for meningitis ttt contact a. cimetidine b. rifampicin
the correct answer is B 8

814) rheumatic fever pt has streptococcal pharyngitis risk to develop another attack
a. 3 times more than normal b. 100% c. 50%
the correct answer is C
815)child with unilateral nasal obstruct with bad odor
a. unilateral adenoid hypertrophy b. FB
the correct answer is B

816) PT. do colectomy for colon cancer routine follow up every


a. 6 months
b. 3 months
c. 9 months
d. 1 years
the correct answer is D
817) for patient with osteoarthritis , what is your initial action :
a. Intraarticular corticosteroid
b. Quadriceps strengthening exercise
c. Climb stairs exercise
The correct answer is B
……………………………………………………………………………………………………………………………………
818) 70-y old female patient with osteoporosis , what is her T score :
a. (-2.5)
b. (-1)
c.(1)
d. (2)
The correct answer is A
……………………………………………………………………………………………………………………………………………
819 ) diabetic patient , diagnosed 2 weeks back came to your clinic at scheduled appointment
supposed to be at 10:00 AM but because you were having another complicated case , he had to
wait for more than an hour , and he was extremely angry , what u will do :
a. be empathetic as this anger is mostly because of the new morbidity diagnosed at this patient
b. you start your talk with him by saying “I was having a hard case “
c. Don’t say anything regarding being late unless he brings it up
d. you start YOUR TALK WITH HIM BY SAYIN “you seem furious”
Correct answer here is B
…………………………………………………………………………………………………………………………………………
820) 1st step in epidemic study is :
a. verifying diagnosis

821) management of mild epistaxis :


a. pinching the fleshy part of nose while tilting head foreword
822) Randomized control trials become stronger if :
a. you follow more than 50% of those in the study
b. systematic assignment predictability by participants
Q is not complete , but with these MCQs , the correct answer is B
……………………………………………………………………………………………………………………………………………
823) child with multiple painful swellings on the dorsum of hands , feet , fingers and toes ,his
CBC showed Hb =7, RBC’s on peripheral smear are crescent shaped , what is your long-term
care :
a. corticosteroids
b. penicillin V
c. antihistaminic
The correct answer is B , this pt. is sickle cell anemia
……………………………………………………………………………………………………………………………………………
824) 54 y old patient , farmer , coming complaining of dry eye , he is smoker for 20 years and
smokes 2 packs/ day , your recommendation :
a. advise him to exercise
b. stop smoking
c. wear sunscreen
The correct answer is B , smoking increase risk of dry eye (international study )
……………………………………………………………………………………………………………………………………………
825) patient is wearing contact lenses for vision correction since ten years , now coming c/o
excessive tearing when exposed to bright light , what will be your advice to him :
a. wear hat
b. wear sunglasses
c. remove the lenses at night
d. saline eye drops 4 times / day
The correct answer is D
826) most common site of malignancy in paranasal sinuses :
a. Maxillary sinus ( correct )
…………………………………………………………………………………………………………………………………………
827) patient with hypertension , what is the best non-pharmacological method to lower the
elevated blood pressure :
a. weight reduction ( correct )
828) newly diagnosed patient with hypertension having Na=147 , K=3 , what is the most likely
cause of his secondary hypertension :
a. hyperaldosteronism
Hypokalemia and hypernatremia are seen with hyperaldosteronism
……………………………………………………………………………………………………………………………………………
829) 50 year old patient , diagnosed with hypertension , he is used to drink one glass of wine
every day , he is also used to get high Na and high K intake , his BMI is 30kg/m , what is the
strongest risk factor for having hypertension in this patient :
a. wine
b. high Na intake
c. high K intake
d. BMI=30
the correct answer is D

830) all of the following tests are necessary to be done before initiating lithium except:
a. liver function tests
……………………………………………………………………………………………………………………………………………
831) in dementia , best drug to use :
a. haloperidol
b. galantamine
the correct answer is B
treatment of dementia is cholinesterase inhibitor ( galantamine, donepezil, rivastigmine,
and tracing )
832) most important point to predict a prognosis of SLE patient: ??
a. degree of renal involvement
b. sex of the patient
c. leucocyte count
the correct answer is A
…………………………………………………………………………………………………………………………………………
833) what is the antibiotic that you will give if you will do appendectomy :
a. doxycycline
b. ceftriaxone
c. metronidazole (most probably/not sure)
d. cefuroxime
The correct answer is B

834) most common site for ectopic pregnancy :


a. fallopian tubes
……………………………………………………………………………………………………………………………………………
835)child with low grade fever , rash , pharyngitis , and tender splenomegaly , +ve EBV , Dx :
a. infectious mononucleosis
……………………………………………………………………………………………………………………………………………
836) best early sign to detect tension pneumothorax :
a. tracheal shift
b. distended neck veins
c. hypotension
The correct answer is C , I'm not sure , I think tracheal shift is the late sign .
……………………………………………………………………………………………………………………………………………
837) First sign of magnisium sulfate toxicity is : a. Loss of deep tendon reflex
838) Patient with mushroom toxicity will present with
a. Constipation b. Hallucination c. Anhydrosis
the correct answer is B

839) Child ate overdose of iron best immediate management


a. Gastric lavage b. Induce vomiting manually c. Emetic drugs d. Ipecac
The correct answer is A
……………………………………………………………………………………………………………………………………………
840) Status epilepticus is :
a. Continuous seizure activity more than 30 min without regaining consciousness
……………………………………………………………………………………………………………………………………………
841) First sign of increased ICP is :
a. contralateral papillary dilatation
b. Ipsilateral papillary dilatation
c. Decrease level of consciousness
the correct answer is C
……………………………………………………………………………………………………………………………………………
842) Patient with history of head trauma unable to move the fork to his mouth , where is the
lesion:
a. cerebellum
b. frontal lobe
c. temporal
the correct answer is A
……………………………………………………………………………………………………………………………………………
843) Patient with lumbar canal stenosis , I forget the presentation , what is the ttt:
a. Physiotherapy b. Lumbar discectomy
Mild to moderate : NSAID + physiotherapy Advance : epidural corticosteroid
Refractory : surgical laminectomy
844) Most common cause of otorrhea :
a. Csf leak
b. Acute otitis media
c. Cerumen leak
The correct answer is B
……………………………………………………………………………………………………………………………………………
845) Most common symptom of acute otitis media :
a. Pain
b. Discharge
c. Tinnitus
d. Vertigo
The correct answer is A
……………………………………………………………………………………………………………………………………………
846) 12 yo boy is mocked at school because he is obese , ate a lot of pill to sleep and never
wake up again , best management is :
a. Refer to mental professional
b. Tell him that most kid grow out before they grow up
c. Advice healthy food
The correct answer is A
…………………………………………………………………………………………………………………………………………
847) 12 yo boy brought by his parent for routine evaluation , his is obese but otherwise healthy
, his parents want to measure his cholesterol level , what is the best indicator of measuring this
child cholesterol :
a. His parent desire
b. Family hx of early CVA
c. High BMI
The correct answer is C

848) Diabetic pt. have neovascularization and vitreous hemorrhage , next step :
a. Refer to ophthalmologist
…………………………………………………………………………………………………………………………………………
849) Histopathology of resected inflamed appendix will show :
a. Neutrophil in muscular wall
b. Lymphoid hyperplasia with multinucleated giant cell
The correct answer is A
……………………………………………………………………………………………………………………………………………
850) A child had bee bite presented after 18 hour with left arm erythema and itching and….
Long scenario , what to do:
a. Antihistaminic
b. Oral steroid
c. Subcutaneous epinephrine
The correct answer is A
………………………………………………………………………………………………………………………………………
851) Old man with urinary incontinence, palpable bladder after voiding , urgency and sense of
incomplete voiding dx;
a. Stress incontinence
b. Overflow
c. Reflex
The correct answer is B
……………………………………………………………………………………………………………………………………………
852) Picture, hyperkeratotic, scaly lesion over the extensor surface of knee and elbow, what to
do to avoid exacerbation:
a. Steroid b. Avoid sun exposure c. Avoid trauma
The correct answer is C
853) Ttt of scabies : a. Permethrin
……………………………………………………………………………………………………………………………………………
854) Which is true about allopurinol :
a. Good if given during acute gout
b. Uricosuric
c. Reduce the chance of uric acid stone
d. Can be antagonize by salicylate
The correct answer is C
……………………………………………………………………………………………………………………………………………
855) man walking in street and saying bad words to stranger , he is not aware of his condition ,
what is the description :
a. flight of idea
b. insertion of idea
c. loosening of association
The correct answer is C
……………………………………………………………………………………………………………………………………………
856) which of the following suggest that thyroid nodule is benign rather that malignant:
a. history of childhood head and neck radiation
b. hard consistency
c. lymphadenopathy
d. presence of multiple nodules
the correct answer is D

857) child presented after sting bite with abd pain , arthralgia , lower extremity and buttock
rash , normal RFT , +ve fecal occult stool , dx :
a. HSP
b. Lyme disease
The correct answer is A
858) best indicator for labor progress is :
a. frequency of contractions
b. strength of contractions
c. descent of the presenting part
the correct answer is C
……………………………………………………………………………………………………………………………………………
859) lactating mother is taking Phenobarbital for seizure prophylaxis , what to advice her :
a. stop breast feeding immediately
b. breast feed after taking the pill by 8 hour
c. start weaning
d. Continue breast feeding and monitor newborn
the correct answer is D
……………………………………………………………………………………………………………………………………………
860) best single way to reduce high blood pressure is :
a. smoke cessation
b. decrease lipid level
c. reduce weight
the correct answer is C
……………………………………………………………………………………………………………………………………………
861) drug of choice for supraventricular tachycardia is : a. adenosine

862) patient with blood group A had blood transfusion group B , the best statement that
describe the result is :
a. type IV hypersensitivity
b. inflammatory reaction
c. type 2 hypersensitivity The correct answer C
863) a nurse gave blood transfusion through CVP , 2 hours later the patient is comatose and
unresponsive , dx :
a. Septic shock
b. Blood group mismatch
Correct options missing here…. Electrolyte imbalance.
864) 100% O2 given for prolonged periods can cause all except:
a. Retrosternal Pain
b. Seizures
c. Depression
d. Ocular Toxicity
The correct answer is C

865) Organophosphorus poisoning, what is the antidote?


a. Atropine
b. Physostigmine
c. Neostigmine
d. Pilocarpine
e. Endrophonium
The correct answer is A
……………………………………………………………………………………………………………………………………………
866) Which of the following pulse character goes with the disease?
a. Collapsing pulse  Severe anemia
b. Pulsus Alternans  Premature ventricular complexes
c. Slow rising Pulse Mitral stenosis
d. Pulsus Bisferens  Mitral Regurgitation
e. Pulsus paradoxus- I think Aortic stenosis or regurgitation
The correct answer is A
867) Entamoeba histolytica cysts are destroyed best by:
a. Boiling
b. Iodine added to water
c. Chlorine added to water
d. Freezing
The correct answer is A
…………………………………………………………………………………………………………………………………………
868) A scenario about an old male with symptoms suggesting parkinsonism such as difficulty
walking, resting tremors and rigidity in addition to hypotension. Then he asks about what is the
most common presenting symptom of this disease
a. Rigidity
b. Tremors
c. Unsteady Gait
d. Hypotension The correct answer is B
……………………………………………………………………………………………………………………………………………
869) Regarding postpartum Psychosis:
a. Recurrences are common in subsequent pregnancies
b. It often progresses to frank schizophrenia
c. It has good prognosis
d. It has insidious onset
e. It usually develops around the 3rd week postpartum
The correct answer is A
……………………………………………………………………………………………………………………………………………
870) What is the most common sequalae of acute pancreatitis ?
a. Abscess b. Pseudocyst c. Bowel obstruction
The correct answer is B Most common complication of acute pancreatitis: pancreatic
Pseudocyst and fistula formation
871) A pregnant female develops lesions on the vulva and vagina and she was diagnosed as
genital herpes, what should be included in her future health care?
a. Cesarean section should be done if the lesions did not disappear before 2 weeks of delivery
date
b. Oral acyclovir to treat herpes
c. Termination of pregnancy because of the risk of fetal malformations
d. Avoidance of sexual intercourse for 1 month after the healing of the lesions
The correct answer is B
HSV in pregnant treated by : oral acyclovir 400 mg TID for 5-7 days . if HSV was present at time
of labor : c – section

872) A picture of 2 warts in the index finger. Diagnosis is  Viral Warts


……………………………………………………………………………………………………………………………………………
873) An old patient with history of cerebrovascular disease & Ischemic heart disease, presents
with a pattern of breathing described as: A period of apnea followed by slow breathing which
accelerates & becomes rapid with hyperpnea & tachycardia then apnea again. What is this type
of breathing?
a. Hippocrates
b. Chyene-stokes breathing
c. Kussmaul breathing
The correct answer is B
1- Chyene-stokes respiration : rapid deep breathing phase followed by period of apnea ,
present with heart failure , stroke, brain trauma , also can be with sleep or high altitude
2-kusmmaul's breathing : rapid and deep breathing . present with metabolic acidosis
particularly in diabetic ketoacidosis

874) 2 years old child with ear pain & bulging tympanic membrane, what is the diagnosis?
a. Otitis media b. Otitis externa c. Otomycosis d. Bullous myringitis
The correct answer is A
875) A patient with mixed 1st & 2nd degree burns in head & neck region, what is the most
appropriate management?
a. Apply silver sulfadiazine cream to all burned areas, cover them and admit to hospital
b. Apply cream to 2nd degree burns and cover them, give IV fluids
c. Debridement of 2nd degree burns and ….
d. Apply silver sulfadiazine then Vaseline ointment to all areas then discharge the patient
The correct answer is A
1sr degree : epidermis + painful ( no blister )
2nd degree : epidermis + partial dermis + painful _ blister
3rd degree : epidermis + full thickness dermis + painless
Rule of nine to estimate BSA : 1- Head and neck : 9% 2- Each upper arm : 9% 3- Each loer limb :
18% 4- Each post. & ant. Trunk : 18% 5- Perineum & genitalia : 1%
-Silver sulfadiazine and mafenide used for all types of burn -supportive : tetanus and I.V
nacrotic can be used for all types of burn - I.V fluid use only in ( > 10% of BSA) in 2nd & 3rd
degree of burn according parkland formula : ( fluid in first 24h = 4 xwt. in kg x BSA .
50% in first 8 hours and another in next 16 hours )
In this patient 1st deree and 2nd degree < 10% ( head and neck 9%) : so, no fluid
……………………………………………………………………………………………………………………………………………
876) Besides IV fluids, what is the most important drug to be given in anaphylaxis?
a. Epinephrine b. Steroids
The correct answer is A
……………………………………………………………………………………………………………………………………………
877) An adult patient in 20s or 30s of age presents by history of 1 month of fever, 5 days of
headache & 2 days of altered sensorium. On examination there is nuchal rigidity, then there is a
table showing investigations which include Hb: 10 g/dl Blood WBC: 18,000 CSF Examination:
WBCs elevated: 77% lymphocytes, 33% Neutrophils. Protein ???. Glucose??
What is the diagnosis?
a. Viral meningoencephalitis b. Tuberculous meningitis
The correct answer is B
878) Regarding chronic fatigue syndrome, which is true?
a. Antibiotics may reduce the symptoms
b. Antidepressants may reduce the symptoms
c. Rest may reduce the symptom
The correct answer is B
chronic fatigue syndrome:
characterizes by profound mental and physical exhaustion. In associated with multiple system
and neurospsychiatric symptoms that last at least 6 mounth. Must be new ( not life long ) ,
must not be relived by rest, and must result in greater than 50% reduction in previous activity.
Presentation with 4 or more of the following : poor memory / concentration, myalgia,
arthalagia, sore throat, tender lymph node, recent onset headach, unrefreshing sleep, excessive
tiredness with exercise. Treatment by : cognitive and excercise therapy .also, diet,
physiotherapy, dietary supplements, antidepressants.
…………………………………………………………………………………………………………………………………………
879) A patient complains of dry eyes, a moisturizing eye drops were prescribed to him 4 times
daily. What is the most appropriate method of application of these eye drops?
a. 1 drop in the lower fornix
b. 2 drops in the lower fornix
c. 1 drop in the upper fornix
d. 2 drops in the upper fornix
Correct answer is A
……………………………………………………………………………………………………………………………………………
880) A patient with penetrating abdominal stab wound. Vitals are: HR 98, BP 140/80, RR 18. A
part of omentum was protruding through the wound. What is the most appropriate next step:
a. FAST Ultrasound
b. DPL (Diagnostic peritoneal lavage)
c. Explore the wound
d. Arrange for a CT
e. Exploratory laparotomy The correct answer is E
881) A patient presents with loin pain radiating to the groin. Renal stones are suspected. What
is the test that has the most specificity & sensitivity in diagnosing this condition?
a. Non contrast spiral CT scan of the abdomen
b. Ultrasound
c. KUB
d. Intravenous pyelography (IVP)
e. Nuclear Scan
The correct answer is A
……………………………………………………………………………………………………………………………………………
882) An old patient presents with history dizziness & falling down 1 day ago accompanied by
history of Epigastric discomfort. He has very high tachycardia I think around 130-140 and BP
100/60. What is the diagnosis?
a. Peptic ulcer
b. GERD
c. Leaking aortic aneurysm
The correct answer is C
……………………………………………………………………………………………………………………………………………
883) A child is about to be given flu vaccine, what allergy should be excluded before giving the
vaccine?
a. Chicken
b. egg
c. Fish
The correct answer is B
…………………………………………………………………………………………………………………………………………
884) Which of the following is proven to reduce the incidence of cancer?
a. Salt b. Mineral water c. Vitamin D d. Fiber
The correct answer is C
885) A female patient with history of cyclic abdominal pain, inability to conceive, heavy menses,
and examination showed tenderness & nodularity in uterosacral ligaments. What is the
diagnosis? a. Endometriosis
……………………………………………………………………………………………………………………………………………
886) A female patient on the 3rd week postpartum. She says to the physician that the
frequently visualizes snakes crawling to her baby’s bed. She knows that it is impossible but she
cannot remove the idea from her head. She says she wakes up around 50 times at night to
check her baby. This problem prevents her from getting good sleep and it started to affect her
marriage. What is this problem she is experiencing?
a. An obsession
b. A hallucination
c. A postpartum psychosis
d. A Delusion
Correct answer is A
The correct answer is a Obsession : persistent, unwanted, and intrusive ideas , thoughts,
impulses or images
……………………………………………………………………………………………………………………………………………
887) Regarding screening for cancer, which of the following is true?
a. Screening for cervical cancer had decreased in recent years
b. Screening for breast cancer had decreased in recent years
c. Screening for Colorectal cancer is inadequate for the high-risk groups
d. Screening for lung cancer has reduced the mortality rate of lung cancer
e. Screening for tobacco use is now adequately done by health professionals
Correct answer is E. Debatable as I could not find references
…………………………………………………………………………………………………………………………………………
888) Which of the following is a side effect of Buprione, a drug used to help smoking cessation:
a. Arrhythmia b. Xerostomia c. Headache d. Seizure
The correct answer is C
889) Regarding postpartum depression, what is the most appropriate intervention to reduce
the symptoms?
a. Include family in the therapy
b. Isolation therapy
c. Add very low doses of imipramine
d. Encourage breastfeeding
The correct answer is A
………………………………………………………………………………………………………………………………………
890) A 60 years old patient with history of heart attack 6 weeks ago, complaining of not getting
enough sleep. Psychiatric evaluation is unremarkable for depression or anxiety, what should be
given to this patient?
a. Amytriptiline b. Buspirone c. Buprione d. Zolpidem
The correct answer is D
……………………………………………………………………………………………………………………………………………
891) A patient is asked to face the wall, bend his waist, and let his hands hang down without
support. This test is used as a screening tool for which of the following?
a. Lower limb asymmetry
b. Rectal prolapsed
c. Scoliosis d. ????
The correct answer is C. This test is called Adam's Forward Bend Test

892) A patient presents with sore throat, cervical lymphadenopathy, mild splenomegaly, EBV
antibodies are +ve. What is the diagnosis?
a. Infectious Mononucleosis (EBV )
……………………………………………………………………………………………………………………………………………
893) Which of the following is an indication for tonsillectomy?
a. Sleep apnea b. Asymptomatic large tonsils c. Peripharyngeal abscess
d. Retropharyngeal abscess
The correct answer is A
894) An old patient presents with knee pain, limitation of movement and quadriceps muscle
atrophy. On examination there is limited range on movement, pain on movement & Crepitus.
What is the diagnosis?
a. Osteoarthritis b. Rheumatoid arthritis c. Ankylosing Spondylitis
The correct answer is A
……………………………………………………………………………………………………………………………………………
895) A patient presents with long time history of knee pain suggestive of osteoarthritis. Now he
complains of unilateral lower limb swelling and on examination there is +ve pedal & tibial
pitting edema. What is the next appropriate investigation?
a. CXR
b. ECG
c. Echocardiography
d. Duplex ultrasound of lower limb
The correct answer is D Osteoarthritis relief by rest . So, immobility pt. can lead to DVT
…………………………………………………………………………………………………………………………………………
896) Alopecia is a side effect of which antiepileptic?
a. Phenytoin b. Carbamazepine c. Na Valproate
The correct answer is C 1-phenytoin : gingival hyperplasia, hirsutism, ataxia
2-carbamazepine : agranulocytosis, hepatotoxicity, aplastic anemia
- Na Valproate : transient hair loss.
…………………………………………………………………………………………………………………………………………
897) Which of the following is a gram –ve rods that grow on charcoal yeast agar?
a. Legionella
…………………………………………………………………………………………………………………………………………
898) A 7 years old child had history of chest infection which was treated with antibiotics. The
patient presented 6 weeks after cessation of antibiotics with abdominal pain, fever and profuse
watery diarrhea for the past month. Which of the following organisms is responsible for the
patient’s condition?
a. Giardia Lamblia b. Clostridium Difficile c. Escherichia coli d. Clostridium Perfringens
The correct answer is B
899) Which of the following features of ulcerative colitis distinguishes it from crohn’s disease
a. Possible malignant transformation
b. Fistula formation
c. Absence of granulomas
d. Colon involvement
The correct answer is B

900) A very very long scenario about a female patient with vaginal discharge “malodorous
watery in character” with pH of 6 & +ve clue cells but there is no branching pseudohyphe. Then
he asks about which of the following drug regimens should NOT be used in this patient:
a. Metronidazole (PO 500 gm for 7 days)
b. Metronidazole (PO 2 large dose tablets for 1 or 2 days)
c. Metronidazole (IV or IM …..)
d. Miconazole ( PO …..)
e. Clindamycin (PO …..)
The correct answer is D. This case a is " bacterial vaginosis" and treatment by : metronidazole
or clindamycin . avoid miconazole because it is antifungal
…………………………………………………………………………………………………………………………………………
901) What is the condition that produces malodorous watery vaginal discharge with +ve clue
cells in wet mount preparation slides:
a. Bacterial vaginosis b. Vaginal Candidiasis c. Trichomonas vaginalis d. Gonorrhea
The correct answer is A
902) Which of the following is the best treatment for Giardiasis: a. Metronidazole
…………………………………………………………………………………………………………………………………………
903) 40 y/o male , presented with large hands , hepatomegaly ….. diagnosis :
a. acromegaly b. gigantism
the correct answer is A
904) A woman with 9weeks History of elevated erythematous wheals overall her body she also
has lip swelling. There is no History of recent travel or food allergy or drug ingestion, Diagnosis:
a. Chronic Angioedema & Urticaria
b. Contact Dermatitis
c. Solar Urticaria
d. Cholinergic Urticaria
e. Cold Urticaria
The correct answer is A
Chronic Urticaria : if more than 6 months

905) A 17 years old male with history of mild intermittent asthma. Attacks occur once or twice
weekly in the morning and no attacks at night. What should be the initial drug to give?
a. Inhaled short acting B2 agonist as needed
b. Inhaled high dose corticosteroid as needed
The correct answer is A
…………………………………………………………………………………………………………………………………………
906) A child presented with sore throat. Culture from the throat revealed +ve meningiococci.
The patient is now asymptomatic. Which of the following should be done?
a. Reassurance
b. Rifampicin oral for 7 days
c. IM ceftriaxone 1 dose
d. Ceftriaxone oral
The correct answer is C
……………………………………………………………………………………………………………………………………………
907) very obese girl , what's the first step to reduce weight :
a. increase water intake
b. decrease calorie intake
The correct answer is B
908) Which of the following increases the quality of the randomized controlled study & make it
stronger:
a. Systemic Assignment predictability by participants
b. Open Allocation
c. Including only the participants who received the full intervention
d. Following at least 50 % of the participants
e. Giving similar intervention to similar groups
The correct answer is A
……………………………………………………………………………………………………………………………………………
909) A scenario about an infant who presented with difficulty breathing and sweating,
examination shows hyperactive precordium with loud S2 and pansystolic murmur grade 3/6
best heard in the left 3rd parasternal region. What is the Diagnosis?
a. VSD b. Mitral Regurgitation c. PDA d. Large ASD
The correct answer is A

910) course of facial nerve when comes out from stylomastoid …


a. Deep to parotid gland & lateral to external carotid & retromandibular vein
…………………………………………………………………………………………………………………………………………
911) old female with hemorrhoids for 10 years , no complication , your action ?
a. observe b. surgery c. increase fiber diet
the correct answer is C
…………………………………………………………………………………………………………………………………………
912) female about 30 y c/o abdominal pain related to menses (scenario going with
endometriosis)…………next step in dx:
a. Laparoscopy b. U/S c. CT
The correct answer is a Confirm diagnosis by laparoscopy ,
US & MRI can use but if normal can not exclude endometriosis .
913) female about 30y with breast cancer (given cbc –chem. And reveal low hb and
hematocrit….) what is the next step in management:
a. Staging b. Lumpectomy c. Mastectomy d . Chemotherapy
Answer A
……………………………………………………………………………………………………………………………………………
914) pt with open angle glaucoma and k/c of COPD and DM ttt:
a. Timolol b. betaxolol c. Acetazolamide
the correct answer is C
…………………………………………………………………………………………………………………………………………………....
915) Infant with runny nose and fever after that develop dyspnea ,wheezing with working
accessory muscles and chest retractions with cyanosis, temp 38 dx:
a. Viral pneumonia
b. Bacterial pneumonia
c. Bronchiolitis
The correct answer is C

916) which drug increase incidence of reflux oesophagitis:


a. Theophylline
b. Amoxicilline
c. Metoclopramide
d. Rantidine
e. Lansoprazole
The correct answer is A

917) most effective ttt of cluster headache:


a. Ergotamine nebulizer b. S/C Sumatriptan c. 100% O2 d. IV Verapamil
The correct answer is C
918) old pt. with HTN and migraine ttt:
a. B blockers b. ACE I c. Ca blockers
The correct answer is A

919) 34y female with HIV pap smear negative, about cervical cancer screening :
a. After 3m if negative repeat after 6m
b. After 6months then annually
c. After 1y………………………………….annually
The correct answer is B (I'm not sure ), may be c .
Screening should begin no later than age 21. Screening should begin earlier than age 21 if
the patient is sexually active. In this case, it should start 3 years after initiation of vaginal
intercourse. Once initiated, screening should be performed annually if a traditional,
glassslide-based technique is used. If liquid medium is used, Pap smear screening may be
performed every other year. After age 30, for women who have had 3 consecutive, normal
Pap smears, screening frequency may be reduced to every two to three years. Women who
are HIV positive, immunocompromised due to disease or medication, or are DES daughters,
should continue annual screening. Screening may stop following a total hysterectomy
(including the cervix), if the the patient is at low risk, and has had three consecutive normal Pap
smears within the last 10 years. High risk patients, including those with a history of cervical
cancer, DES exposure in-utero, HIV positive, immunocompromised from medication, and those
tested positive for HPV, should continue to be screened indefinitely. Screening may stop
after age 70, if the patient is low risk, and has had three normal Pap smears over the last 10
years. Screening may be omitted in the case of women with life-threatening or other serious
illness
…………………………………………………………………………………………………………………………………
920) lactating women 10 days after delivery developed fever ,malaise, chills tender Lt breast
with hotness and small nodule in upper outer quadrant with axillary LN .Leukocyte count was
14 x10/L dx:
a. Inflammatory breast cancer
b. Breast abscess
c. Fibrocystic disease
Correct answer here is B
921) newborn with fracture mid clavicle what is true:
a. Most cases cause serious complication
b. Arm sling or figure 8 sling used
c. Most patient heal without complications
The correct answer is C -most clavicles fracture in newborn no need to treatment apart from
careful handling. If the fracture is displaced and baby in pain, simple sling is required

922) 70y male with osteoporosis the T score of bone densometry would be :
a. 3.5 b. -2.5 c. -1 d. 2 e. 3.5
The correct answer is B -
T score > (- 1) : normal BMD -
T score (-1 to -2.5) : osteopenia -
T score < ( -2.5 ) : osteoporosis (I think 1 of the answers should be -3.5 & it would be the right
answer)
………………………………………………………………………………………………………………………………………
923) what is the most complication after hysterectomy:
a. Ureteral injury b. Pulmonary embolism c. Haemorrhage
The correct answer is C
…………………………………………………………………………………………………………………………………………………...
924) young female with BMI 18 , fine hair allover body , feeling of she is fat ,doesn’t eat well
with excessive exercise………………..
a. Anorexia nervosa? b. Body dysmorphic disorder c. Bulimia nervosa
The correct answer is A
…………………………………………………………………………………………………………………………………………………....
925) what's advantage of mature human milk over cow's milk:
a. More protein
b. More Iron content
c. More calories
d. More fat
The correct answer is C
…………………………………………………………………………………………………………………………………………
926) young male c/o of deformity of jaw. past h/o of nasoplasty and blepharoplasty O/E
nothing abnormal ………..dx:
a. Body dysmorphic syndrome

927) the most common cause of non-traumatic subarachnoid Haemorrhage:


a. Rupture aneurysm b. Vessels abnormality c. Hypertension
The correct answer is A -
Most common causes of subarachnoid hemorrhage are trauma and berry aneurysm
…………………………………………………………………………………………………………………………………………………....
928) Pt. with chest pain and s.o.b decreased by leaning forward . O/E friction rub and increased
JVP >>>> (a case of pericardial effusion) ECG will show:
a. ST changes
b. Low voltage
c. Increase PR interval answer A
ECG changes during pericarditis: diffuse ST elevation, PR depression followed by t-wave
intersion , and low voltage.
…………………………………………………………………………………………………………………………………………………....
929) The causative organism of pseudomembranous colitis is: a. Clostridium difficile
…………………………………………………………………………………………………………………………………………
930) scenario about arthritis (I couldn't remember the details ) but Joint aspirate reveal Gram
negative diplococci …..dx:
a. Nisseria gonorrhea
N. gonorrhoeae : gram negative diplococci and can also cause conjunctivitis, pharyngitis,
proctitis or urethritis, prostatitis and orchitis.
……………………………………………………………………………………………………………………………………………
931) Baby with red macule & dilated capillary on the rt side of the face
Sturge-Weber Syndrome or Nevus Flammeus (one of them will come)
Don’t choose milia or cavernous haemangioma
932) A 25 year old secondary school teacher that every time enters the class starts sweating
and having palpitation, she is a fired to give wrong information and be unparsed. What is the
diagnosis:
a. Specific Phobia
b. Social Phobia
The correct answer is B
……………………………………………………………………………………………………………………………………………….....
933) At which chromosome is the cystic fibrosis gene:
a. Long arm chromosome 7
b. Short arm chromosome 7
c. Long arm chromosome 8
d. Short arm chromosome 8
e. Long arm chromosome 17
The correct answer is A
……………………………………………………………………………………………………………………………………………….....
934) Regarding hepatocellular carcinoma (Hepatoma) Which is true:
a. More common in females
b. .. c. ..
d. The most common cancer in Africa and Asia
e. Increase risk in chronic liver disease
The correct answer is E , chronic disease especially Hepatitis B
…………………………………………………………………………………………………………………………………………
935) A patient is having a 2 year history of low interest in live, he doesn’t sleep well and can’t
find joy in life, What is the most likely diagnosis:
a. Dysthymia
b. Major depressive disorder
c. Bipolar disorder
The correct answer is A
936) A case of a patient with thin cervix and little amount of cervical mucus, how would you
treat her: ????? a. Estrogen injections

937) What is the mechanism of OCD drugs:


a. Increase availability of Serotonin
b. Decrease production of Serotonin
c. Increase production of Serotonin
d. .. Serotonin e. .. Serotonin
The correct answer is A
…………………………………………………………………………………………………………………………………………
938) What is the MOST accurate test to detect early pregnancy:
a. ..Serum BHCG b. ..
c. Urine pregnancy test
d. Ultrasound e. ..
The correct answer is A
……………………………………………………………………………………………………………………………………………….....
939) The most common cause on chronic interrupted rectal bleeding is:
a. Diverticulosis b. Hemorrhoids
The correct answer is B
……………………………………………………………………………………………………………………………………………….....
940) A child is having a croup early morning, the most common cause is:
a. Post nasal drip
941) A burn patient is treated with Silver Sulfadiazine, the toxicity of this drug can cause:
a. Leukocytosis
b. Neutropenia
c. Electrolyte disbalance
d. Hypokalemia e. ..
The correct answer is B
941) A case scenario about a patient who had appendectomy, after that he has abdominal pain
and constipation and absent bowel sound, the most likely cause is:
a. Ileus paralytic
Q is not complete
……………………………………………………………………………………………………………………………………………….....
942) A case scenario describing a patient who had a right chest trauma than developed tension
pneumothorax, the immediate management is:
a. Insert a needle in the second intercostal space
b. Insert a needle in the fifth intercostal space
c. Give IV antibiotics d. ..
e. Insert a tube in the fifth intercostal space
The correct answer is A
……………………………………………………………………………………………………………………………………………….....
943) A patient that have a penile ulcer that healed after that he developed a palm and sole
rash, the most likely cause is:
a. Syphilis
Q is not complete
944) A case scenario about a patient who has on and off episodes of abdominal pain and was
found to have multiple gallstones, the largest is 1 cm and they are not blocking the duct, What
will you do:
a. Give pain killers medication
b. Remove gallbladder by surgery
Q is not complete , but with these MCQs , the correct answer is B
……………………………………………………………………………………………………………………………………………….....
945) Most common symptoms of soft tissue sarcoma :
a. Paralysis b. On growing mass c. Pain d. ..
The correct answer is B
……………………………………………………………………………………………………………………………………………….....
946) Patient use illegal drug abuse and the blood show RNA virus. Which hepatitis
a. A
b. B (DNA)
c. C
d. E
The correct answer is C , RNA virus transmitted by blood ic HCV
……………………………………………………………………………………………………………………………………………….....
947) A case scenario about bronchial carcinoma, which is true: ????
a. The most common cancer in females
b. Squamous cell carcinoma spreads faster
c. Adenocarcinoma is usually in the upper part
d. Elevation of the diaphragm on the x-ray means that the carcinoma has metastasize outside
the chest
e. Bronchoscopy should be done
The correct answer is E

948) old female with pubic itching with bloody discharge, then she developed pea shaped
swelling in her labia, most likely:
a. Bartholin cyst
b. Bartholin gland carcinoma
c. Bartholin abscess d. ..
The correct answer is A
……………………………………………………………………………………………………………………………………………….....
949) Patient has decrease visual acuity bilateral , but more in rt side , visual field is not affected
, in fundus there is irregular pigmentations and early cataract formation . what you will do
a. Refer to ophthalmologist for laser therapy
b. Refer to ophthalmologist for cataract surgery c. ..
The correct answer is A
950) Inflammatory bowel disease is idiopathic but one of following is possible underlying cause:
a. Immunological
……………………………………………………………………………………………………………………………………………….....
951) pregnant lady delivered Anencephaly stillbirth occurrence of neural tube defect in next
pregnancy
a. 8% b. 2% c. 10% d. 20%
The correct answer is B , ( 2 – 4 % )
…………………………………………………………………………………………………………………………………………
952) typist thenar atrophy ,+ve phalen sign
a. Median nerve b. Ulnar c. Brachial
The correct answer is A
953) painful pile
a. Excision drainage
b. Sitz bath and steroid supp
c. Ab
d. Fiber food and analgesics
The correct answer is A
…………………………………………………………………………………………………………………………………………
954) 39 years old HIV patient with TB receive 4 drugs of tt after one month
a. Continue 4 drugs for 1 years
b. Continue isoniazid for 9 months
c. Continue isoniazid for 1 year
The correct answer is B
……………………………………………………………………………………………………………………………………………
955) Malaria ?
- the most common cases is caused by Plasmodium falciparum.
956) female not married with normal investigation except FBS=142. RBS196. so ttt
a. give insulin subcutaneous
b. advice not become married
c. barrier contraceptive is good
d. BMI control
The correct answer is D
…………………………………………………………………………………………………………………………………………………
957) Entamoeba histolytica cysts are destroyed best by:
a. Boiling
b. Iodine added to water
c. Chlorine added to water
d. Freezing
The correct answer is A
958)A scenario about an old male with symptoms suggesting parkinsonism such as difficulty
walking, resting tremors and rigidity in addition to hypotension. Then he asks about what is the
most common presenting symptom of this disease
a. Rigidity
b. Tremors
c. Unsteady Gait
d. Hypotension E. ??? The correct answer is B
…………………………………………………………………………………………………………………………………………

959) Regarding postpartum Psychosis:


a. Recurrences are common in subsequent pregnancies
b. It often progresses to frank schizophrenia
c. It has good prognosis
d. It has insidious onset
e. It usually develops around the 3rd week postpartum
The correct answer is A
…………………………………………………………………………………………………………………………………………
960) Gastrectomy post-op 1 day. He have temperature 38.8 & pulse 112. What is the most
common cause ?
a. wound infection.
b. inflammatory mediator in the circulation.
c. UTI
d. normal
the correct answer is B
most common causes of post-operation fever : ( 6 w )
1- Wound
2- Wine = pneumonia or atlectasia
3- Water = UTI
4- What happen = transfusion
5- Walk = DVT
6- Wonder drug = aspirin, erythromycin , isoniazid
Immediately ( 24 h ) : post-op inflammation , wound infection , transfusion , streptococcus
Acute ( 3 – 7 days ) : pneumonia
…………………………………………………………………………………………………………………………………………………….

961 ) Polyartheralgia rhumatica. What is the thing that suggest it rather than increased ESR &
C-reactive protein:
a. proximal muscle weakness
b. proximal muscle tenderness
the correct answer is B
Muscle weakness is not a feature of PMR.
……………………………………………………………………………………………………………………………………………
962) The cause of insulin resistance in obese is:
a. increase insulin receptors kinase activity
b. increase number of insulin receptor
c. increase circulation of anti-insulin
d. decrease insulin production from the pancreas
e. decrease post-receptor action
the correct answer is E

963) Pt came with total bilirubin: 5.8 & direct bilirubin: 0.4
a. Guillain–Barré syndrome
b. Dubin-johnson syndrome
c. Biliary sclerosis
d. Criggler-Najjar syndrome
e. Gilbert Syndrome
the correct answer is E
……………………………………………………………………………………………………………………………………………………

964) Pt complaining of hypotension & bradycardia. Electrolytes show: decrease Na, increase K,
increase Cl, increase Urea. So the cause of this is:
a. hyponatremia
b. hyperkalemia
c. hyperchloremia
d. uremia
the correct answer is B
965) Scenario about premenstrual depression syndrome ???? Premenstrual syndrome : define
as a symptoms complex of physiological emotional symptoms severe enough to interfere with
everyday life and occur cyclical during luteal phase of menses
……………………………………………………………………………………………………………
966) After doing CPR on child and the showing asystole:
a. Atropine
b. Adrenaline
c. Lidocaine
The correct answer is B.

You might also like